BALLBUSTERPEDS

Pataasin ang iyong marka sa homework at exams ngayon gamit ang Quizwiz!

Stage B symptoms in Hodgkins Lymphoma are associated with a POOR prognosis. What are the symptoms?

- fever, weight loss, night sweats

Bilateral cataract with direct van den Bergh's test positive in a 3-month old neonate who is exclusively breastfed is suffering from: a. von Gierke disease b. Toxoplasmosis c. Galactosemia d. Fructosemia

...

Brachycephaly occurs due to fusion a. Coronal suture b. Lambdoid suture c. Sagittal suture d. Parietal suture

...

Calculate the percentage of burns on the head, neck and face in child of 1 year: a. 15% b. 19% c. 10% d. 13%

...

Cephalhematoma usually disappears within ........... weeks: a. 5-7 b. 2-12 c. 1-2 d. 3-5

...

Cerebral palsy is defined as: a. A non progressive neuromuscular disorder of cerebral origin b. Progressive neuromuscular disorder of cerebral origin c. Progressive neuromuscular disorder of cerebral origin d. Non progressive neuromuscular disorder of peripheral origin

...

Characteristics Feature of lactase deficiency is a. Acidity of stool b. Abnormal muscle cells c. Abdominal pain, distention and diarrhoea after milk ingestion d. Decreased serum lactic acid

...

Child born to diabetic mother can have all except: a. Small left colon syndrome b.Hypoglycemia c. Hypocalcemia d. Mental retardation

...

Chronic Iridocyclitis is present in: a. Pauciarticular rhematoid factor +ve b.Juvenile systemic Rh -ve arthritis c. Polyarticular Rh factor +ve d. Pauciarticular type I

...

Clinical manifestations of cystic fibrosis do not include: a. Malabsorption b.Cor pulmonale c. Bronchiectasis d. Hypertension

...

Combination of retinitis pigmentosa and icthyosis is seen in a. Down syndrome b. Turner syndrome c. Refsum's disease d. Mohr's syndrome

...

Compared to oral digitalizing dose, parentral dose of digoxin should be: a. 1/3 b. 1/4 c. 1/2 d. 2/3

...

Congenital heart disease, which causes death in the first week of life, is: a. Ventricular septal defect b. Tetralogy of fallot c. Ebstein's anomaly d. Hypoplastic left ventricle syndrome

...

Convulsions in a child with dehydration and vomiting can be due to: a. Decreased serum magnesium b. Decreased serum sodium c. Decreased serum chloride d. Decreased serum potassium

...

Convulsions in infants can be caused by: a. Hypocalcemia b. Anoxia c. Hypoglycemia d. All of the above

...

Craniopharyngioma is most commonly located in: a.Suprasellar b. Intraventricular c. Intracerebral d. Infrasellar

...

Croup is not associated with: a. Adenovirus b. Enterovirus c. Influenza virus d. RSV

...

Croup syndrome is usually caused by: a. Coxsackie B virus b. Rhinovirus c. Coxsackie A virus d. Parainfluenza viruses

...

Dandy-Walker syndrome is because of obstruction of: a. Arachnoid granulations b. Foramen of Magendie and Luschka c. Foramen of Munro d. Foramen of Morgagni

...

Death in diaphragmatic hernia is due to: a. Hypoplastic lung b. Cardiac malformation c. Intestinal obstruction d. Infections

...

Decreased serum alpha-fetoprotein level is seen in: a. Exomphalos b. Fetal death c. Meningomyelocele d. Down syndrome

...

Delayed Eruption of teeth is seen in: a. Down Syndrome b. Hypopituitarism c. Hypothyroidism d. All of the above

...

Diarrhoea in infants is most commonly caused by which of the following viruses: a. Rotavirus b. Rhabdovirus c. Myxovirus d. Adenovirus

...

Drug of choice for an attack of familial periodic paralysis: a. Adrenaline b. Potassium chloride c. ACTH d. Calcium chloride

...

Drug of choice for neonatal convulsions: a. Carbamazepine b. Valproate c. Phenobarbitone d. Phenytoin

...

Drug of choice in community acquired neonatal pneumonia is: a. etronidazole + Amikacin b.Ampicillin+ Chloramphenicol c. Cefotaxime+ amikacin d. Ampicillin+ Gentamicin

...

Drug of choice in infantile myoclonic seizures: a. ACTH b. Clonazepam c. Phenytoin d. Phenobarbitone

...

Dyslexia is characterized by all except: a. Mental retardation b. Inability to interpret written language c. Male preponderance d. Retarding reading

...

Earliest change in raised intracranial tension in a child is: a. Posterior clinoid erosion b. Erosion of sella c. Silver beaten appearance d. Sutural separation

...

Earliest manifestation of rickets is a. Harrison's groove b. Craniotabes c. Pigeon chest d. Rachitic rosary

...

Early sign of rickets at 6 months of age is: a. Craniotabes b. Bent tibia c. Double malleoli d. Knock knees

...

Emergency treatment of TGV: a. Balloon septostomy b. Digoxin c. Oxygen d. Ventilation

...

Empyema in a child less than 2 years is most commonly caused by: a. Klebsiella b. Staphylococcus c. Escherichia coli d. Pneumococcus

...

Energy requirement of an infant is: a. 50-60 kcal/kg b.60-70 Kcal/kg c. 80-90 Kcal/kg d. 110-120 Kcal/kg

...

Epiphyseal dysgenesis ia a pathognomic feature of which of the following conditions: a.Hyperrthyroidism b.Hypothyroidism c. Hyperparathyroidism d. Hypoparathyroidism

...

Erythromycin is not the drug of choice in which of the following childhood infections: a. Chlamydia trachomatis b. Pneumocystis carinii c. Bordetella pertusis d. Mycoplasma pneumoniae

...

Failure to pass meconium within 48 hrs of birth in a newborn with no obvious external abnormality should be lead to suspicion of: a. Anal atresia b. Congenital pouch colon c. Congenital aganglionosis d. econium ileus

...

False regarding Down Syndrome is: a. Duodenl atresia b. Decreased femur length c. Decreased humerous length d. Decreased nuchal fold thickness

...

False regarding Hemolytic- uremic syndrome is: a. Viral prodrome b. Burr cells are seen c. ARF d. Always fatal

...

False regarding Henoch-Schonlein purpura: a. Nephritis b. Centrifugal rash c. Thrombocytopenia d. Arthritis

...

False regarding Pierre Robin syndrome is: a. Mandibular hypoplasia b. Coloboma irides c. Cleft Palate d. Respiratory distress

...

False regarding Reye's syndrome is: a. Frequently complicates viral infection b. Deep jaundice is present c. Prothrombin time is prolonged d. Disease may be precipitated by salicylates

...

False regarding Reye's syndrome: a. Bilirubin of more than 3% b. Microfatty changes in liver without inflammatory changes c. Increased prothrombin time d. Cerebral edema

...

False regarding breath holding a. Attacks of cyanosis can occur b.Child may lose consciousness c. Antiepileptic treatment is necessary d. Occurs between 6 months to 5 years of age.

...

False regarding breath holding spells in children is a.It must be immediately attended to prevent hypoxia and onset of seizure b. Parents are advised to ignore and not to reinforce these attacks c. It does not contribute to increased risk of seizure disorder d. It is fairly common in the first two years of life

...

False regarding cartilage hair hypoplasia syndrome: a. T cell dysfunction b. Lymphocytosis c. Sparse hair d. Short limb dwarfism

...

What time defines delayed passage of meconium? Differential for this condition?

48 hours after birth. meconium ileus and Hirschsprung

Fines motors: Wiggles thumb, Copies cross

48 months

Gross motors: Hops on one foot

48 months

Language: Knows colors, Asks questions

48 months

Personal/Social: Dresses self

48 months

Clinical jaundice develops at a bilirubin level > __ mg/dl

5

Enuresis is defined as urinary incontinence > age __

5

Describe the burn 'rule of 9's' in children.

18% = head and neck, 9% = each upper extremity, 18% = anterior portion of trunk, 18% = posterior portion of trunk, 14% each lower extremity, 1% = perineum/genitalia

Fines motors: Scribbles, Turns pages of book

18-21 months

Gross motors: Runs well, Kicks large ball, Walks backward

18-21 months

Language: Follows simple commands, 20-50 words

18-21 months

Personal/Social: Drinks well from a cup, Feeds self, Uses a spoon well

18-21 months

What is the incubation period of the influenza virus?

18-72H

Regarding lead screening. What year houses started being built without lead paint?

1979 and onwards. If a house is built before 1978, the child in the family is at risk for lead poisoning.

What are normal bowel habits in the 1st week of life? 1st 3 motnhs/ By 2 yeras of age?

1st week of life, infants pass 4 soft or liquid bowel movements per day 1st 3 months, breastfed infants 3 soft bowel movements/day. Some may have a BM after each feeding, some only once a week. This is normal. Formula fed infants have 2-3 bowel movements/day By 2 years of age, child has 1-2 formed BM/day BM's should be soft, but formed

A burn involving part of the dermis that can be superficial or deep is a burn of what degree?

2

T or F. If strabismus is left untreated after age ___, ambylopia will result.

2

What combination of major and minor criteria must you have to be diagnosed with Rhematic Fever?

2 Major or 1 Major and 2 minor

At what age is stable alignment of the eye usually present?

2 months

How many bowel movements per week are considered "abnormal" for a diagnosis of constipation?

2 or fewer bowel movements per week are considered abnormal. More common in women and advanced age.

When does body weight quadruple?

2 years

When does AAP recommended screening for obesity? Over what percentile is characteristic for obesity?

2, 85

Fine motor: tracks object past midline, hands open

2-3 months

Gross motor: Lifts head

2-3 months

Language: vocalizes in play, Coos

2-3 months

Personal/Social: smiles responsively

2-3 months

Wilms tumor, age?

2-5 years

False regarding childhood autism is: a. Isolated area of talent may be remarkable developed b. Parental attitude and behavior do not attribute to it c. Emergence of speech and social smile is often delayed d. It represents a type of childhood neurotic disorder

...

False regarding childhood malignancies: a. AML is the the most common type of leukemia. b. Neuroblastoma ocurs in first 4 years of life c. Lymphoma commonly presents with CNS symptoms d. 66% of leukemia have splenomegaly

...

False regarding cystic fibrosis is a. Increased sweat Na+ concentration b. Dry Mouth c. Steatorrhea d. Sterility

...

False regarding febrile convulsions: a. Aspirin must be given during spike of fever b. Temperature >38 degree centigrade c. Occurs from six months to 5 years d. Seizures occur early in disease

...

False regarding fetal alcohol syndrome is: a. Growth retardation b. Micrognathia c. Brachycephaly d. Altered palmer crease pattern

...

False regarding primary complex in children: a. Effusion seen b. Mediastinal lymphadenopathy c. Connecting lymphatic channels d. Ghon's focus in lung

...

False regarding pseudotumor cerebri: a. CT scan shows ring enhancing lesion b. Ventricular system is of normal size or small c. No focal neurological deficit d. ICT is raised

...

False regarding tricuspid atresia is: a. Split S2 b. Pulmonary oligemia c. Patent foramen ovale d. Left axis deviation

...

False regarding typhoid is: a. Recurrence is common b. Localizes in gall bladder c. Chronic carrier state lasts for 3-4 months d. Fecal carrier are more than urinary carrier

...

Feature of petit mal epilepsy in EEG is: a. Spikes and dome b. Continuous Stunted spikes c. Waves at the rate of 10/seconds d. Continuous Tall spikes

...

Fetal hemoglobin equals adult hemoglobin at: a. 2 months b. Birth c. 4 months d. 6 months

...

Fever stops and rash begins is diagnostic of: a. Fifth disease b. Roseola infantum c. Measles d. Toxic shock syndrome

...

First clinical manifestation of abetalipoproteinemia is: a. Convulsions b. Tremors c. Steatorrhea d. Hypohydrosis

...

First permanent tooth to erupt is: a.First premolar b.First molar c. First incisor d. Second premolar

...

Full-term, small -for-date infants (IUGR) are predisposed to which of the following condition: a. PDA b. Hypercalcemia c. CNS infections d. Hypoglycemia

...

Genotype in testicular feminization syndrome is: a. 46XY b. 45XO c. 47XXY d. 46 XX

...

Ghon's focus is: a. Primary complex b. Miliary TB c. TB lymph nodes d. Postprimary TB

...

Hemolytic anaemia in G6PD deficiency is not caused by: a. Co-trimoxazole b. Nitrofurantoin c. Rifampicin d. Nalidixic acid

...

Hereditary retinoblastomas develop the following chromosomal deletion a. 13q14 b. 13p14 c. 14p13 d. 14q13

...

Human colostrum has a relatively lower concentration of: a. Sodium b. Fat c. Protein d. IgA

...

Human colostrum in comparison with mature human milk contains more: a. Protein b. Fat c. Water d. Lactose

...

Hyaline menbrane seen in lung is composed of: a. Polysaccharide b. Mucoprotein c. Globulin d. Fibrin

...

Hypokalemia in an infant is not due to: a. Diarrhoea b. Thiazide therapy c. ARF d. Beta-2 agonists

...

IUGR can be caused all except: a. Captopril b. Caffeine c. Atenolol d. Phenothiazine

...

IUGR is caused by all except: a. Diabetes b. Chronic renal failure c. Alcohol d. Smoking

...

IUGR is characterized by all except: a. Polycythemia b. Meconium aspiration syndrome c. HMD d. Hypocalcemia

...

IUGR is defined as: a. <7 percentile of expected weight b. <8 percentile of expected weight c. <10 percentile of expected weight d. 5 percentile of expected weight (Al 1993)

...

In 'asymmetrical' IUGR which organ is not affected: a. Subcutaneous Fat b. Muscle c. Liver d. Brain

...

In G6PD deficiency, hemolytic anaemia does not occur with: a. Estrogen b. Nitrofurantoin c. Primaquine d. Salicylates

...

In India most common cause of intracerebral calcification in children is: a. Toxoplasmosis b. AV malformation c. Tuberculoma d. Hyperparathyroidism

...

In ORS the function of glucose is to: a. Increase Na + absorption by co-transport b. Increase Na +K+ pump activity c. Increase osmolality of ORS d. Give sweet taste of ORS

...

In X-ray batwing appearance is due to: a. Pulmonary embolism b. Mitral stenosis c. Cardiogenic pulmonary edema d. Bronchial asthma

...

In a 2 year old child, seizure with chorioretinitis is suggestive of: a. Hypoxia b. Tay- Sachs disease c. Congenital toxoplasmosis d. Galactosemia

...

In a child with attack of severe acute asthma, the treatment of choice is: a. IV hydrocortisone b. SC Adrenaline c. IV theophylline d. Nebulised salbutamol

...

In a newborn, Harlequins skin change is due to a. Polycythemia b. Septicemia c. Autonomic dysfunction d. Icthyosis

...

In a term newborn, hypoglycemia occurs when the serum blood glucose level is below: a. 30 mg/dl b. 40 mg/dl c. 50 mg/dl d. 55 mg/dl

...

In acute polymyelitis the most common cause of death is: a. Intercostal muscle paralysis b.Cardiac Arrest c. Respiratory failure d. Convulsions

...

In an AIDS patient, diarrheal syndrome can be due to: a. Cryptosporidium b. Adenovirus c. Rotavirus d. Escheria coli

...

In an infant congestive cardiac failure is diagnosed by: a. Basal crypts b. Elevated JVP c. Pedal edema d. Liver enlargement

...

In an infant with galactosemia which of the following should be avoided: a. Egg b. Rice c. Milk d. Gluten

...

In bed wetting the drug of choice is a. Trazodone b. Tranylcypromine c. Imipramine d. Selegiline

...

In children hydrocephalus is commonly caused by: a. Brain tumours b. Postinflammatory obstruction c. Perinatal injury d. Congenital anomaly

...

In children lactase deficiency is characterized by: a. Low lactic acid level in blood b. High pH of stool c. Abdominal bloating, cramps and diarrhoea d. Dumping syndrome

...

In children most common posterior fossa tumor is a. Meniningioma b. Astrocytoma c. Glioblastoma multiforme d. Medulloblastoma

...

In congenital rubella most commonly ocurring defect is: a. Blindness b. Mental retardation c. Deafness d. Microcephaly

...

In cystic fibrosis most common organism is Pseudomonas aeruginosa. Next most common organism is: a. Staphylococcus aureus b. Klebsiella c. Pneumococci d. Streptococcus pyogens

...

In human milk the percentage of lactose is: a.4.5 gm% b. 7.2 gm% c. 8.0 gm% d. 6.7 gm%

...

In infants which of the following drug is contraindicatd: a. Ciprofloxacin b. Chloroquine c. Theophylline d. Barbiturate

...

In kernicterus which is increased: a. Conjugated bilirubin b. Unconjugated bilirubin c. None d. Both

...

In kernicterus, staining of brain is more intense in: a. Corpus callosum b.Cerebral cortex c. Basal ganglion d. Spinal cord

...

In meningitis subdural sffusion is most commonly due to: a. Meningococcal b. Pneumococcal c. Haemophilus d. Tuberculous

...

In pertussis antibiotic of choice is: a. Penicillin b. Amphicillin c. Erythromycin d. Gentamycin

...

In physiological jaundice in term infants, serum bilirubin does not generally exceed: a. 20 % b. 5 % c. 15 % d. 10 %

...

In pulse polio program, target age group is: a. Under 3 years b. Under 10 years c. Under 5 year d. Under 1 year

...

In the cases of CNS relapse ALL chemotherapy would consist of intrathecal: a. Methotrexate b. Prednisolone c. Adrimycin d. Methotrexate+ Cystosine arabinoside

...

In the treatment of severe malnutrition, calories required/ kg body weight in phase III is: a. 130 b. 300 c. 180 d. 125

...

In tuberculous meningitis, the CSF finding includes: a. Low sugar + high protein and lymphoma b. High sugar + high chloride c. Low sugar + high protein and lymphocytes d. High sugar + low protein

...

In which of the following conditions, combination chemotherapy is not indicated: a. Immunologically suppressed patients b. Primary complex c. Acute epiglottitis d. Acute laryngotracheobronchitis

...

Incubation period of pertussis is........... days: a. 3 b. 7-10 c. 28 d. 15-28

...

Infantile myocarditis and pericarditis is not caused by: a. Echovirus b. Coxsackie A c. Coxsackie B d. Rotavirus

...

Infants born to mothers of advance age has greater risk of: a. Down's syndrome b. Glycogenesis typeII c. Marfan's syndrome d. Trisomy 18

...

Infective endocardititis is least common with: a.Aortic regurgitation b. Small ventricular septal defect c. ASD d. VSD

...

Jaundice in newborn is physiological when: a. Jaundice persists for more than 4 days in a term infant b. Infant is visibly jaundiced in first 24 hour of birth c. Total bilirubin concentration in serum increases by 1 mg/dl per day d. Total bilirubin concentration in serum is above 15 mg/dl

...

Karyotype in Klinefelter syndrome: a. XY b. XXY c. XO d. XX

...

Kernicterus is caused by: a. Bilirubin monoglucuronide b. Albumin bound bilirubin c. Unbound bilirubin d. Bilirubin diglucuronide

...

Kwashiorkor is not characterized by: a. Fatty infiltration of pancreas b. Fatty liver c. Patchy depigmented hair d. Edema

...

L Asparaginase is particularly used in: a. CLL b. ALL c. CML d. AML

...

Meconium is excreted by a newborn till: a. 6 days b. 3 days c. 5 days d. 2 days

...

Meningitis in preschool children is most commonly due to: a. Staphylococcus b. Streptococcus c. Pneumococcus d. Haemophilus influenza

...

Microcytic hypochromic anaemia ia not seen in: a. Fanconi anaemia b.Iron deficiency anaemia c. Thalassemia major d. Lead poisoning

...

Minimum level of bilirubin to start phototherapy in a 3- day-old full term baby with jaundice is: a.15 mg% b.18 mg% c. 12 mg% d. 9 mg%

...

Most common cause of GI hemorrhage in children is: a. Mallory Weiss syndrome b. Portal hypertension c. Peptic ulcer d. Erosive damage

...

Most common cause of congenital hypothyroidism is: a. Deficiency of deoximinase b. Defective release c. Thyroid dysgenesis d. Pendre syndrome

...

Most common cause of convulsions in a child with fever is: a. Hypothyroidism b. Febrile convulsions c. Meningitis d. Epilepsy

...

Most common cause of nocturnal enuresis in children is a. NIDDM b. Spina bifida c. UTI d. Psychological stress

...

Most common cause of systemic hypertension in children is: a. Congenital adrenal hyperplasia b. Acute glomerulonephritis c. Coarctation of aorta d. Nephrotic syndrome

...

Most common childhood tumor is a. Lymphangioma b. Meningioma c. Ependymoma d. Glioma

...

Most common renal lesion in children is a. Diffuse glomerulonephritis b. Focal glomerulonephritis c. Membranoproliferative glomerulonephritis d. Lipoid nephrosis

...

Most common site of childhood rhabdomyosarcoma is: a. Trunk b. Genitourinary system c. Extremities d. Head and neck

...

Most common solid tumor in children and infants is: a. Neurofibroma b. Neuroblastoma c. Sacrococcygeal teratoma d. Wilm's tumor

...

Most common type of cerebral palsy is: a. Spastic b. Atonic c. Extrapyramidal d. None

...

Most indicative sign of pneumonia in 12 months old child with cough and fever is: a. Temperature 39.5°C and respiratory rate is 60/min b. Intercostal retraction and fever c. Temperature 39.5°C d. Grunting and respiration rate is 60/min

...

Murmur heard in Eisenmenger syndrome is: a. Carey Coomb murmur b. Austin- Flint murmur c. Grahm steell murmur d. None

...

NOt a clinical feature of VSD: a. A loud pan systolic murmur is present at the right sternal border b. Infective endocarditis is also seen c. Congestive cardiac failure is common d. second heart sound is split and variable with respiration

...

NOt a feature of neonatal tetanus: a. Mortality is 95% b. Refusal to feed is common initial symptom c. Usually occurs in first 8 days of life d. It is caused by Clostridium tetani

...

NOt a usual feature of ascariasis: a. Intussusception b. Loeffler's syndrome c. Anemia d. Abdominal pain

...

Nadas' criteria are used for: a. Malnutrition b. Dehydration c. Mental retardation d. Presence of heart disease

...

Neonatal jaundice seen after 48 hours of birth with bilirubin < 5% is due to : a. Physiological jaundice b. Neonatal hepatitis c. Hemolytic disease of newborn d. ABO incompatibility

...

Neonatal sepsis and meningitis is most commonly caused by: a. Group B Streptococcus b. Staphylococcus aureus c. Streptococcus agalactiae d. Enterococcus faecalis

...

Neonate with recurrent hypoglycemia attacks and hepatomegaly is likely to have: a. Von Gierke disease b. Neonatal hepaitis c. Galactosemia d. Neonatal diabetes

...

Neuroblastoma in children is characterized by all except: a. Proptosis b. Hypertension c. Cafe au lait spots d. Hepatomegaly

...

Normal caloric requirement of a 5-year child is: a.2000 calories b.1500 calories c. 800 calories d. 1000 calories

...

Normal calorie requirement for a 9-year-old child is: a. 2000 calories b. 800 calories c. 1700 calories d. 1000 calories

...

Normal reflex patterns at birth include: a. Sucking and rooting reflex b. Moro's reflex c. Grasp reflex d. All of the above

...

Not a cause of floppy baby: a. Prader Willi syndrome b. Little's disease c. Werdnig- Hofman syndrome d. Down syndrome

...

Not a characteristic feature of Down's syndrome: a. Mental retardation b. Protruding tongue c. Simian crease d. Prominent occiput

...

Not a characteristic feature of cretinism: a. Macroglossia b. Dwarfism c. Hyperpyrexia d. Mental retardation

...

Not a characteristic feature of fragile X syndrome: a. Large testis b. Large nose c. Large ear d. Large face

...

Not a clinical feature of Dandy-Walker syndrome: a. Arachnoid cyst b. Agenesis of cerebellar vermis c. Hydrocephalus d. Posterior fossa cyst

...

Not a clinical feature of scurvy in children: a. Retrobulbar hemorrhage b. Intracerebral hemorrhage c. Perifollicular hemorrhage d. Hemorrhage into periosteum of long bones

...

Not a complication of chickenpox: a. Pneumonia b. Reye's syndrome c. Enteritis d. Meningitis

...

Not a complication of pertussis: a. Pneumonia b. Convulsions c. Cerebellar ataxia d. Subconjunctival hemorrhage

...

Not a constituent of ORS: a. Glucose without salt b. Glucose with salt c. Sodium d. Potassium

...

Not a feature of Down Syndrome: a. Respiratory tract infections b. Pigmented birth marks c. Hypotonia d. Clinodactyly

...

Not a feature of Laurence- Moon-Biedl syndrome: a. Retinitis pigmentosa b. Polydactyly c. Diabetes insipidus d. Mental retardation

...

Not a feature of neonatal necrotizing enterocolitis a. Pneumoperitoneum b. Metabolic acidosis c. Increased bowel sound d. Abdominal distension

...

Not a radiographic feature of rickets: a. Increase in width of growth plate b. Rickety rosary c. Subperiosteal bleeding d. Decrease in bone density

...

Not a recognized feature of Wilson's disease: a. Increased copper content of liver b. Increased ceruloplasmin c. Histopathological features of chronic active hepatitis d. Psychological disturbances

...

Not a recognized sign and symptom of raised ICT in a 9-month-old infant: a. Papilledema b. Diplopoa c. Increase in head size d. Bulging fontanel

...

Not a sign of hydrocephalus in a neonate: a. Depressed fontanelle b. Crack pot sign c. Sunset sign d. Enlarged head

...

Not associated with a large fontanel: a. Vitamin D deficiency rickets b. Down's syndrome c. Hypogonadism d. Osteogenesis imperfecta

...

Not caused by RSV: a. Pneumonia in elderly b. Cold in children c. Coryza in adult d. ARDS

...

Not seen in Kwashiorkor: a. Edema b. Fatty infiltration of liver c. Depigmented areas in skin d. Fatty infiltration of pancreas

...

Not seen in a child with nephrotic syndrome: a. Hyperlipidemia b. Increased alpha globulins c.Uremia d. Hypoproteinemia

...

Not seen in myotonic dystrophy: a. Cardiac defect b. Cataract c. Respiratory failure d. Enlarged testes

...

Notching of 4-9th ribs with double bulging is found in: a. Diaphragmatic hernia b. Coarctation of aorta c. Aortic dissection d. Aortic aneurysm

...

Obesity is associated with: a. Laurence Moon Biedl Syndrome b. Prader Willi syndrome c. Carpenter's syndrome d. All of the above

...

One gram of haemoglobin librates how many milligram of bilirubin: a.55 b. 34 c. 10 d. 40

...

One-year-old child presenting with abdominal mass and calcification on X-rays is suggestive of: a. Wilm's tumour b. Neuroblastoma c. Rhabdomyosarcoma d. Teratoma

...

Osler's nodes are typically seen in which 1 of the following: a. Chronic candida endocarditis b. Acute staphylococcal endocarditis c. Pseudomonas endocarditis d. Libman- Sack's endocarditis

...

Persistent Moro's reflex at 12 weeks indicates: a. Normal child b. Brain damage c. Irritable child d. Hungry child

...

Phenylketouria is diagnosed by: a. Guthrie test b. Vanillylmandelic acid in urine c. Gerhard's test d. Ferric chloride test

...

Phototherapy reduces un-conjugated bilirubin to conjugated bilirubin by: a. Structural isomerization b. Photo-oxidation c. E-isomerization d. All of the above

...

Pleomorphic rash is characteristic of: a. Erythema subitum b. Erythema infectiosum c. Chickenpox d. Smallpox

...

Preterm infant with PDA should be treated with: a. Corticosteroids b. Indomethacin c. Aspirin d. Prostaglandin E2

...

Prior to discharge of a patient of haemophilus influenzae meningitis, which of the following essential investigations should be done: a. Development screening test b. ECG c. ABER d. EEG

...

Pseudopancreatic cyst in a child is commonly because of: a. Drug induced pancreatitis b. Choledochal cyst c. Traumatic pancreatitis d. Annular pancreatitis

...

Pure gonadal dysgenesis will be diagnosed in the presence of: a. Bilateral Streak gonads b. Bilateral Dysgenetic gonads c. One side streak and other dysgenetic gonads d. One side streak and other normal looking gonad

...

Retinoblastoma is bilateral in: a. 100% of cases b. 1% of cases c. 50% of cases d. 30% of cases

...

Retrolental fibroplasia is because of: a. Hyperoxia b. Hypoxia c. Hypocapnia d. Increased CO2

...

Risk of kernicterus is not increased: a. High level of serum albumin b. Low level of serum albumin c. Prematurity d. Acidosis

...

Risk of neonatal chickenpox is the maximum, if maternal infection occurs: a. Within 6 weeks of delivery b. Within 4 days of delivery c. During the first trimester d. During the second trimester

...

Roseola infantum is caused by: a. Herpes virus type 6 b. Parvovirus c. Coxsackie virue d. Escherichia coli

...

Rotavirus immunity to children occurs above: a. 5 years b. 10 years c. 3 years d. 7 years

...

Rotaviruses are responsible for: a. Acute non-bacterial gastroenteritis b. Infantile diarrhea c. Teratogenic effects d. Respiratory tract infection

...

Rubella maximally infects the fetus if the mother contacts the disease during ........................... weeks of pregnancy: a. 5-6 b. 12-13 c. 20 d. 9-12

...

SSPE (Subacute sclerosing panencephalitis) is associated with: a. Mumps b. Chickenpox c. Herpes d. Measles

...

Scratch tests are best read within: a. 4 hours b. 1 hour c. 10 minutes d. 15-20 minutes

...

Secondary hypertension in children is most commonly caused by: a. Adrenal tumors b. Systemic vasculitis c.Renal artery stenosis d.Renal disease

...

Sexual maturity rate (SMR) in girls does not include: a. Breast development b. Onset of menarche c. Weight velocity d. Pubic hair development

...

Social smile appears by: a. 2 months b. 3 months c. 5 months d. 9 months

...

Spasticity is not associated with: a. LMN type paralysis b. Quadriplegia c. Clonus d. Clasp Knife rigidity

...

Spaulding sign is an indication of: a. Dead fetus b. Macerated fetus c. Battered fetus d. Putrefied fetus

...

Tetanus is characterized by: a. Incubation period is 2-3 days b. Spasm of masseter occurs first c. Accompanied by loss of consciousness d. Can be cultured from wound

...

The characteristic feature of lactase deficiency in children is: a. Low lactic acid level in blood b. High pH of stool c. Abdominal distention, cramps and diarrhoea d. Dumping syndrome

...

The characteristics of caput succedaneum do not include: a. A diffuse edematous swelling of the soft tissues of scalp b. Crosses the suture line c. Does not disappear within 2-3 days d. Crosses midline

...

The clinical sign of hyaline membrane disease generally first appears: a. In the first 6 hours of life b. After 48 hr of life c. Between 12-24 hr of life d. Between 36-48 hr of life

...

The correct dose of chloroquine in 4-6 years of age group is: a. 500 mg b. 520 mg c. 400 mg d. 150 mg

...

The diagnosis of rheumatic fever is best confirmed by: a. ASO titer b. ECG changes c. Raised ESR d. Throat swab culture

...

The earliest immunoglobulin to be synthesized by the fetus is: a.IgA b.IgG c.IgE d. IgM

...

The goal of incubator care is to maintain the core temperature of infant at: a. 36.6-37.5 degree C b. 30.6-31.5 degree C c. 39.6-40.5 degree C d. None

...

The incubation period of hepatitis A is: a. 6 weeks to 6 months b. 2 days to 2 weeks c. 4 weeks d. 2 weeks to 2 months

...

The major toxicity of acetaminophen overdose involves: a. Heart b. Acid base metabolism c. Liver d. CNS

...

The national schedule of immunization in a child begins at birth and end at: a. 3rd year b. 4th year c. 5th year d. 10th year

...

The number of fontanelles present in a newborn child is: a. 1 b. 2 c. 5 d. 6

...

The syndrome of inappropriate antidiuretic hormone is characterized by the following: a. Hyponatremia and urine sodiumexcretion >20mEq/liter b. Hypernatremia and urine sodium excretion <20mEq/liter c. Hyponatremia and hyperkalemia d. Hypernatraemia and hypokalemia

...

The treatment of choice for primary grade-V vesicoureteral reflux involving both kidneys in a 6-month-old boy is: a. Antibiotics prophylaxis b. Ureteric reimplantation c. Cystoscopy followed by subureteric injection of Teflon d. Bilateral ureterostomy

...

There is a high risk of renal dysplasia in: a. Posterior urethral valves b. Bladder extrophy c. Anorectal malformation d. Neonatal sepsis

...

Transient myeloproliferative disorder of the newborn is seen in association with: a. Turner syndrome b. Down Syndrome c. Neurofibromatosis d. Ataxia telangiectasia

...

Triad of mental retardation, seizure and sebaceous adenoma is seen in: a. Tuberculous sclerosis b. Hypothyroidism c. Congenital syphillis d. Toxoplasmosis

...

True about dietary treatment of galactosemia is: a. must be continued life long b. Ensures normal intellectual development if started in the first week of life c. Can be relaxed after puberty d. Involves delayed weaning

...

True regarding Hemorrhagic disease of newborn: a.Prolonged Bleeding time b. Defective platelet count c. Prolonged thrombin time d. Prolonged prothrombin time

...

True regarding a 2-month-old child is: a. Sustain head level with body when placed in ventral suspension b. Hold head and chest off a flat surface with extended elbows c. Lift head and chest off a flat surface with extended elbows d. Show a positive parachute protective reflec

...

True regarding about juvenile CML is: a. Massive Splenomegaly b. High HbF content c. High alkaline phosphatase d. Philadelphia chromosome

...

True regarding canals of lambert in the alveolar spaces of lung: a. Prevent atelectasis b. Bronchoalveolar connections c. None d. Both

...

True regarding childhood brain tumor is: a. Rare form of malignancy b. Most tumors are below tentorium c. Hemiparesis is frequent form of presentation d. Papilledema is infrequent

...

True regarding diaphragmatic hernia is: a. More common in right side b. Vomitting occurs early c. Chest X-ray is not diagnostic d. Pulmonary hyperplasia is seen

...

True regarding ectopia cordis is: a. Is seen between the ribs b. The heart is seen projecting from a defect in the lower thorax and the upper abdominal wall c. Projects from the whole of defect arising from the abdominal wall d. Projects from the upper thoracic region

...

True regarding laryngeal papilloma is: a. Seen in adults b. Multiple c. Cautery is treatment d. Single

...

True regarding nutritional rickets is: a. Decreased cortical bone b. Decreased calcification of mature bone c. Decreased calcification of growing bone d. Decreased production of steroid

...

True regarding transient tachypnea of newborn: a. It often leads to chronic lung disease b. In premature babies it is often fatal c. Onset of respiratory distress is immediately after birth and it rarely lasts beyond 72 hours d. It is the commonest respiratory disorder caused by absence of surfactant

...

Upper and lower portion of body disproportion with growth retardation is characteristic of: a. Congenital hyperthyroidism b. Silver Russell syndrome c. Turner Syndrome d. Down Syndrome

...

Upper segment lower segment ratio at 3 years of age in a normal child is: a. 1.7 b. 1.1 c. 1.3 d. 1.0

...

Watson- Schwartz test is used to diagnose: a. Wilson's disease b. Hemochromatosis c. Acute intermittent porphyria d. All of the above

...

Which of the following do not regress a. Salmon patch b. Strawberry angioma c. Port wine stain d. Lymphangiomyomatosis circumscripta

...

Which of the following does not predispose to malignancy: a. Down's syndrome b. Cri du chat syndrome c. Klinefelter syndrome d. Turner's syndrome

...

Which of the following fatty acids is necessary during 0-6 months of age: a. Arachidonic acid b. Linolenic acid c. Linoleic acid d. Palmitic acid

...

Which of the following immunoglobulins is most affected in kwashiorkor: a. IgM b. IgE c. IgA d. IgD

...

Which of the following is a preventable cause of mental retardation in children: a. Cerebral palsy b. Hypothyroidism c. Down syndrome d. All of the above

...

Which of the following is affected by lactase deficiency: a. Soya bean b. Meat c. Cereals d. Milk ingestion

...

Which of the following is caused by congenital 17 hydroxylase deficiency: a. Hyperkalemia b. Hermophorditism c. Hypertension d. Virilism

...

Which of the following is false regarding Turner syndrome: a. Hypergonadotropic hypogonadism b. Horseshoe Kidney c. Coarctation of aorta d. Breast development

...

Which of the following is necessary for diagnosis of rheumatic fever: a. Prior history of rheumatic fever b. Recent sore throat infection c. One major and two major criteria d. Erythema marginatum

...

Which of the following is not a tumor of first decade of life a. Ameloblastoma b. Rhabdomyosarcoma c. Neuroblastoma d. Retinoblastoma

...

Which of the following is not an indication for antibiotics in diarrhoea: a. Organism with darting mortality b. Septic focus c. Severe dehydration d. Bloody diarrhoea

...

Which of the following is the most common age group affected by infantile colic: a. 4-6 months b. 0-3 months c. 6-9 months d. 9-12 months

...

Which of the following is the most common cause of abdominal mass in neonates? a. Neuroblastomas b. Wilm's tumor c. Distended bladder d. Multicystic dysplastic kidneys

...

Which of the following is the most common cause of fecal soiling (encopresis) in a child: a. Chronic constipation b. Hirschsprung's disease c. Gaetroparesis d. Gastrocolic reflex

...

Which of the following is true of Hyaline membrane disease of newborn: a. Maternal steroid exposure increases severity of disease b. Surfactant increases the surface tension of alveoli c. Phosphotidyl glycerol estimation is a reliable method of diagnosis d. Prematurity provides relative protection to occurrence

...

Which of the following is true regarding TOF: a. Spells associated with hypoxia and cyanosis b. Usually presents after 7 years c. No supportive treatment is required d. Murmur intensity during spells

...

Which of the following is true regarding a 1-year- old child with PDA: a. Indomethacin may help in closure b. Chances of spontaneous closure is high c. Symptoms similar to aorto-pulmonary window d. Endocarditis is rare

...

What age is ALL associated in kids? Diagnosis? Clinically can these patients present with leukocytes less than 10K?

2-5 years. Bone marrow biopsy. Yes this is half of all presentations. (be cautious!) Note: do a lymph node biopsy if bone marrow is equivocalHow

Motor findings expected a 1 year of age?

2-finger pincer grasp and walking independently.

Levels less than ___ mg/dl suggest deficiency of Vitamin A.

20

Fines motors: Turns doorknobs, Builds towers of six to seven blocks

24 months

Gross motors: Throws ball overhead, Jumps, walks up and down (2ft per step)

24 months

Language: Points to body parts

24 months

Which of the following is true regarding coarctation of aorta: a. Right ventricular hypertrophy b. Upper rib notching due to erosion by dilated collateral vessels c. Most common distal to the origin of the left subclavian artery d. Most common presentation at 15-20 years of age

...

Which of the following is true regarding erysipelas a. Anaerobic infection b. Subcutaneous inflammation due to streptococcus c. Lymphangitis due to streptococcus d. Lymphangitis due to staphylococcus

...

Which of the following is true regarding phocomella: a. It is defect in long bones b. Absence of brain c. It is defect in short bones d. Reduplication of bones

...

Which of the following is true regarding tuberculoma in childhood: a. No macrophages are seen b. Always associated with extracerebral tuberculosis c. Most common in brainstem d. always associted with meningitis

...

Which of the following maternal antibody causes heart block in newborn? a. Anti Ds DNA b. Anti Jo c. Anti-Ro antibodies d. Anti-histone antibodies

...

Personal/Social: Washes and dries hands, Little spilling during self-feeding, know first and last name

24 months

Which of the following should not be done if a women having tuberculosis delivers a baby: a. Disposal of sputum by burning b. Give ATT c. Stop breastfeeding d. Give baby BCG

...

Personal/Social: Shows displeasure through vocalization, Enjoy looking around; laughs

4-5 months

When does body weight double?

4-5 months

social smile

4-6 weeks, 1st social interaction, if not by then most likely a CNS problem

2/3 of kids have 1 episode of acute otitis media. In what percentage of kids does it resolve spontaneously?

40-60%?

What levels would warrant dimercaptosuccinic acid? how about the addition of calcium disodium edetate to that drug?

45-69. > 70.

Turner Syndrome

45X, phenotypic female, short stature, wide chest, low set years, excess nuchal skin, webbed neck, lymphedema of hands and feet, coarctation of aorta, low estrogen, osteoporosis

While working in the neonatal ICU your team delivers a premature infant at 27 weeks of gestation and weighing 1,500 grams. How soon will you request fundus examination by an ophthalmologist? a. Immediately b. 3-4 weeks after delivery c. At 34 weeks gestational period d. At 40 weeks gestation period

...

X-ray appearance in hyaline membrane disease is: a. Homogenous ground glass appearance b. Air bronchogram c. Reticulonodular shadow d. Normal X-ray

...

Y chromosome is always present in which of the following conditions: a. Gonadal dysgenesis b. True hermaphroditism c. Klinefelter syndrome d. Turner syndrome

...

Symptoms of scurvy caused by vitamin C deficiency occur when plasma conc of ascorbic acid is less than ___ mg/dl

.2

Fine motor: Clenched fist, Eye contact

0-2 months

Gross motor: Turns head side to side

0-2 months

Language: Cries, Startles at loud noise,

0-2 months

Personal/Social: Recognizes human face

0-2 months

A burn involving just the dermis, is a burn of what degree?

1

How long does transient synovitis last for?

1 to 4 weeks at most.

When does body weight triple?

1 year

The diagnosis of Juvenile R. Arthritis is made on specific criteria. What is the criteria?

1. Age of onset < 16 (many affected b/t ages of 1 and 3) 2. Arthritis in 1 or more joints. Not swelling or effusion w/ 2 or more of the following: • Limited ROM, tenderness, pain on motion, increased heat 3. Duration of disease 6 weeks or longer 4. Exclusion of other d/os

What are the signs of an acute herpes simplex infection? What are the signs of a latent herpes simplex infection?

Acute infection = development of multinucleated giant cells. Latent infection triggered by fever, trauma and exposure to UV light.

What will you see in the labs of a pt with Juvenile R. Arthritis?

Anemia and leukocytosis. Examination of the synovial fluid reveals elevated WBC count. ESR and CRP are increases or normal w/ systemic type. ANA may e increased in the pauciarticular type and indicates a tendency for uveitis.

What are the causative bugs in bacterial sinusitis? What are the causative bugs in viral sinusitis? What are the causative bugs in fungal sinusitis?

Bacterial= s. pneumo, h. influenzae, m. catarrhalis, anaerobes, p. aureginosa, s.aureus Viral = rhinovirus, parainfluenzae, influenza Fungal = rhizopus, rhizomucor, mucor, asperigillus

What are the complications of erythema infectiosum (5th dz)?

Complications: Infection during pregnancy inc. risk of miscarriage. Aplastic crisis can also develop in pts infected w/ the virus. Treatment of the aplastic crisis includes IV IgM.

What is the treatment for RA?

Control of inflammation with NSAIDs, immunosuppressive drugs and steroids. Methotrexate or leflunomide may be used as second-line agents, early on if there is no improvement. PT is vital in maintaining joint function.

In children with UTI, what is the treatment if culture is positive for EColi? What about enterococcus?

Ecoli: Cefdinir. TMP-SMX and Augmentin alternatives but resistance Enterococcus: Amox. Nitrofurantoin is alternative

When are erythropoietin levels lowest? When does it rise to max values?

Erythropoietin levels lowest at one month and highest at 2 months. RBC production is at min during 2nd week, and rises to max values at 3 months.

What is the treatment for andorgenetic alopciea in men/

Finasteride (Propecia): 1 mg daily. Indicated for men only, for vertex or frontal balding. Minoxidil (Rogaine 5%) most effective for vertex balding Ketoconazole 2% shampoo (nizoral) Pyrithione Zinc (1%): shampoo 2-4 times weekly Hair transplant

Patients with acute rheumatic fever may have these positive titers

Includes positive antistrepolysin-O (ASO), anti-DNase or antihyalurodinase titers.

What lab findings will you see in factor XI d/o and what is the mainstay of treatment?

Lab features - prolonged PTT, normal PT and decreased factor XI activity. FFP is the mainstay of tx.

What is the treatment for androgenetic alopecia in women?

Minoxidil (Rogaine) 2%- apply BID to dry scalp. Risk of hypertrichosis of the face (abnormal amnt of hair growth on body). Retin A may augment effect. Oral contraceptives- less effective than Minoxidil Spironolactone- results in modest reduction in hair loss Flutamide (Eulexin)- Liver function abnormalities occur in up to 1/3 of pts.

What antibiotics are indicated for vesicourethral reflux in neonates <8 wks? What about older children?

Neonates and infants < 8 weeks: Amoxicillin Older children: Trimethoprim-sulfamethoxazole, nitrofurantoin, and penicillins

What is the treatment for croup?

Oxygen, Racemic epinephrine, Glucocorticoids. Artificial airway may be used in severe cases.

What is the most common location of Ewing's sarcoma?

Pelvis is most common location, followed by femur, tibia and humerus

How do you treat osteosarcoma?

TX: consists of chemotherapy and surgical treatment. • Chemotherapeutic agents = methotrexate, doxorubicin, cisplatin and ifosfamide. • Surgical treatment consists of limb-sparing surgery.

What is the treatment of coarctation of the aorta?

Treatment: Signs of HF must be treated aggressively. Prostaglandin E can be used to dilate the PDA. Repair via balloon angioplasty or surgical anastomosis.

The diagnosis of HSV can be made by cell culture, tzanck smear, antigen detection and PCR. What will tzanck smear show? What will PCR show?

Tzanck smear shows multincucleated giant cells. PCR is the test of choice for dx of HSV encephalitis.

What are the clinical manifestions of juvenile r. arthritis?

polyarticular, oligoarthritic or systemic. Specific symptoms include morning stiffness, night pain, refusal to bear weight and joint deformity. Systemic symptoms include fatigue, anorexia, low grade fever, rash and h/s-megaly.

Bordetella Pertussis INFO

5 doses of DTap required from 2 months to 6 years with a Tdap booster from 11 to 18 years and during pregnancy. Bacterial culture and/or PCR of nasopharyngeal secretions to confirm infx lymphocyte-predominant leukocytosis **unique to bacteria macrolide antibiotics = gold standard

Fines motors: Holds a pencil correctly, print first name, count 10 objects

5 years

Gross motors: Skips using, alternate feet

5 years

Language: Easily carries on a conversation, May count or recite part of the alphabet, Asks meaning of words

5 years

Personal/Social: Brushes teeth, without help, tie shoes

5 years

What are the best tests to diagnose Glomerulonephritis?

: Serologic markers (ANCAs) anti-GMBS, Urinalysis (dysmorphic RBCs and RBC casts are common), proteinuria noted, Biopsy may need. *Antistreptolysin-O titer inc. in 80 - 89% of pts.* Serum component (C3) levels are decreased.

How is SJS dx?

: Skin bx shows full thickness epidermal necrosis with sparing of the dermis. Disruption is in the dermal epidermal junction.

What is the treatment for hirschprung disease?

: Surgical resection of affected bowel

What is the treatment for foreign body aspiration?

: Varies with age of patient. Bronchoscopy is both diagnostic and therapeutic. < 1 = face down, forceful back blows given > 1 = Heimlich maneuver Avoid blind finger sweeps

What is the murmur of a VSD?

: holocystolic murmur best heard at middle to LLSB.

What are adverse effects of inhaled corticosteroids used in the treatment of asthma?

: oral candidiasis, dysphonia, HPA growth suppression, bone loss, skin thinning, cataract formation

What will you see on labs of a pt with HSP?

: platelets normal, coags normal, UA may show hem. ESR, CRP, and WBC elevated. Should test BUN and Cr for renal involvement

What are common agents that can cause acute pharyngotonsilitis?

: strep pyogenes, adenovirus, rhinovirus, enterovirus, influenza A and B, EBV, RSV

Minimal chang disease. Age of presentation?

< 10 years.

What is the galveston formula for fluid resucitation in children? (Burns)

% dextrose in lactated ringer's solution. Total volume = 5000 mL/m of BSA burned plus 2000 mL/m during the 1st 24 hours. ½ of total is given the 1st 8 hours and the rest given over the next 16 hours. Dextrose is added to prevent hypoglycemia.

Philadelphia chromosome indicates a poor prognosis in leukemia. What translocation causes it?

(9;22)

Chronic hepatitis may result from viral infection of Hep B, C or D bust most commonly by inherited disorders such as..

(Wilson disease, a-antitrypsin deficiency), autoimmune deficiency of liver etc.

In children which of the following is the most common complication of mumps a. Aseptic meningitis b. Otitis media c. Myocarditis d. Orchitis

(a) Aseptic meningitis

Cirrhosis is not seen in a. Cystinuria b. Alpha-1 antitrypsin deficiency c. Wilson disease d. Galactosemia

(a) Cystinuria

Best treatment for a neonate born to hepatitis B mother is a. Immunoglobulin (HBIG) and hepatitis B vaccine b. Hepatitis B vaccine c. Immunoglobulin d. Isolation

(a) Immunoglobulin (HBIG) and hepatitis B vaccine

Ossification center which develops before birth is a. Lower end of femur b. Lower end of tibia c. Talus d. Coracoid

(a) Lower end of femur

Which of the following is not treated by steroids a. MPGN b. Membranous glomerulonephritis c. Focal segmental glomerulosclerosis d. Lupus glomerulonephritis

(a) MPGN

Which of the following maternal disorders can lead to microcephaly a. Rubella b. SLE c. Hurler syndrome d. Hepatitis A

(a) Rubella

Long standing unconjugated hyperbilirubinemia of newborn is not caused by: a. Criggler najjar syndrome b. Cretinism c. Septicemia d. Breast milk jaundice

(a) Septicemia

In children, Cushing syndrome is best diagnosed by a. Steroid assay b. Dexamethasone suppression test c. CT scan d. ACTH level

(a) Steroid assay

Calculate the APGAR score of a newborn with heart rate 110, good cry, Blue extremities, active movement, grimace a. 9 b. 8 c. 6 d. 4

(b)

A child can climb stairs with altenate limbs at a. 4 years of age b. 3 years of age c. 2 years of age d. 1 year of age

(b) 3 years of age

False regarding Wilms tumor is a. Derived from nepal blastema b. Always associated with hematuria c. Fever may be present d. Abdominal mass present

(b) Always associated with hematuria

Most common cause of convulsion in a newborn on first day of life is a. Head injury b. Anoxia c. Hypoglycemia d. Hypocalcemia

(b) Anoxia

Congenital hypothyroidism is diagnosed earliest by which of the following: a. Decreased T3 level in blood b. Increased TSH level in blood c. Incraesed radioactive iodine uptake d. Protein boudn iodine estimation in blood

(b) Increased TSH in blood

Which of the following is false regarding measles a. Koplik spots b. Incubration period 20-22 days c. Sequence of appearance of rash is characteristic d. Caused by paramyxoviruses

(b) Incubration period 20-22 days

Which of the following is the most common renal lesion in children and young adults a. Hemolytic uremic syndrome b. Minimal change nephrotic syndrome c. MPGN d. Interstitial nephritis

(b) Minimal change nephrotic syndrome

Not a causative agent of lung abscess a. Entamoeba histolytica b. Pneumococcus c. Staphylococcus d. Klebsiella

(b) Pneumococcus

Not included in APGAR score in neonates a. Color b. Muscle tone c. Respiratory rate d. Heart rate

(b) Respiratory rate

In Down syndrome, defect is a. Trisomy 18 b. Trisomy 21 c. Trisomy 13 d. 45 XO

(b) Trisomy 21

Which of the following is false regarding Gaucher's disease in children a. Spleen is always markedly enlarged. b. X linked recessive trait c. Neuropsychological defects in the child d. Deficiency of glucosylceramide beta-glucosidase

(b) X linked recessive trait

Isolation period of mumps is until: a. Headache subsides. b. Swelling subsides c. Pain subsides d. Fever subsides

(b) swelling subsides

Breastfeeding should be started earliest by___hours a. More than 24 b. More than 48 c. 2-4 d. More than 72

(c) 2-4

Myaer-Rokitansky-Kuster-Hauser Syndrome is due to a. 45 XO b. 47 XXY c. 46 XX d. Trisomy 18

(c) 46 XX

Which of the following is true regarding microcephaly ? a. It is a metabolic disease b. It a familial disorder c. Autosomal recessive disease d. Result of mild encephalitis

(c) Autosomal recessive disease

Blast cells of acute lymphoblastic leukemia in childhood have a. Excessive mitochondria b. Antibodies on WBC c. CALLA +ve d. Surface antigen

(c) CALLA + ve

Hair on end appearance on skull X-ray and greatly increased diploe is see in a. Meningioma b. Multiple myeloma c. Congenital hemolytic anemia d. Paget's disease

(c) Congenital hemolytic anemia

Which of the following is not an X-linked disorder a. Ocular albinism b. Hemophilia A c. Congential adrenal hyperplasia d. Testicular feminization

(c) Congential adrenal hyperplasia

A newborn child with respiratory distress and Apgar score of 3 in one minute is resuscitated with IPPR. On examination he has cyanosis, apex beat shifted to the right side and scaphoid abdomen. The diagnosis is a. TOF b. Dextrocardia c. Diaphragmatic hernia d. Pnuemothorax

(c) Diaphragmatic hernia

Which of the following drug does not cause hemolytic anemia in G6PD deficiency a. Nitrofurantoin b. Dapsone c. INH d. Primaquine

(c) INH

In hypothyroidism which of the followisng appears first a. Decreased BMR b. Decreased T3 c. Increased TSH d. Delayed ankle jerk

(c) Increased TSH

Fatty liver does not occur in a. Alcoholic liver disease b. Diabetes mellitus c. Marasmus d. Kwashiorkor

(c) Marasmus

Cardiac anomaly commonly seen in congenital rubella is a. TOF b. ASD c. PDA d. VSD

(c) PDA

Wilms tumor is not associated with a. Beckwith syndrome b.. Aniridia c. Polycystic kidney d. Hemihypertrophy

(c) Polycystic kidney

Koplik's spots are pathognomic of a. Chickenpox b. Typhoid c. Rubeola d. Rubella

(c) Rubeola

Which of the following is not associated with prolonged unconjugated hyperbilirubinemia in a newborn a. Breast milk jaundice b. Hypothyroidism c. Septicemia d. Crigler najjar syndrome

(c) Septicemia

Pneumatoceles are commonly associated with: a. Viral pneumonia b. Klebsiella pneumonia c. Staphylococcal pneumonia d. Pneumococcal pneumonia

(c) Staphylococcal pneumonia

A child presents with increased oral secretion and choking. He has a. Cyanotic heart disease b. Pyloric stenosis c. Tracheoesophageal fistula d. Achalasia cardia

(c) Tracheoesophageal fistula

Which of the following is false regarding measles a. Swelling persist for 3-7 days b. Incubation period is of 2-3 weeks c. Unilateral swelling is common d. Painful parotid swelling

(c) Unilateral swelling is common

Glucose-6-phosphate dehydrogenase deficiency is a. AR b. AD c. X-linked d. Y-linked

(c) X-linked

In a child with bronchial asthma, steroid should be given a. Orally b. Subcutaneously c. Aerosol d. Intravenously

(d) Aerosol

Thiamine deficiency causes a. Glossitis b. Cheilosis c. Angular stomatitis d. Beriberi disease

(d) Beriberi disease

A newborn of HBsAg positive mother should be vaccinated at a. One year b. 6 months c. 1 month d. Birth

(d) Birth

In which of the following diseases Saber shaped tibia is seen a. Renal osteodystrophy b. Gonorrhea c. Rickets d. Congenital syphilis

(d) Congential syphilis

The chest Xray film of a newborn with respiratory distress showing multiple air containing lesions in the left hemithorax and mediastinal shift indicates: a. Congenital lung cysts b. Pneumatoceles c. Neonatal emphysema d. Diaphragmatic hernia

(d) Diaphragmatic hernia

Which of the following is used to access the motor function of a 4 year old child a. Ascends with alternate step from staircase with holding the railing b. Skips smoothly c. Stands on single leg for 15 seconds d. Hops

(d) Hops

False regarding childhood polycystic kidney is a a. It can be diagnosed by USG b. It presents with hypertension and renal insufficiency c. Abdominal mass present d. It is autosomal dominant

(d) It is autosomal dominant

Which of the following is a common cause of bronchiolitis a. Mycoplasma b. Parainfluenza virus c. Adenovirus d. RSV

(d) RSV

Not a feature of wilson's disease in children a. Hemolytic anemia b. Chronic active hepatitis c. Fanconi syndrome d. Sensory changes

(d) Sensory changes

Which of the following is not beneficial in the manangement of acute attack of bronchial asthma in children a. IV hydrocortisone b. Sodium cromoglycate nebulization c. High concentration of oxygen d. Salbutamol nebulization

(d) Sodium cromoglycate nebulization

Cyanosis with plethora is not seen in a. Transposition of great vessels b. Total anomalous pulmonary damage c. Ebsteins anomaly d. TOF

(d) TOF

All are associated with congenital rubella except a. Pulmonary stenosis b. Patent ductus arteriosus c. ASD d. VSD

(d) VSD

A direct inguinal hernia results as a weakness of transversalis fascia in Hesselbach's triangle. What comprises hesselbach's triangle?

(inferior epigastric vessels, inguinal ligament, and the rectus sheath)

Hepatomegaly in Infancy

A normal liver edge is palpated 1 to 2 cm below the right costal margin in a young infant NOTE: hepatomegaly (3cm or greater) is a fairly consistent finding in children with CHF. Decreased renal blood flow, via activation of RAA, leads to fluid retention, systemic venous congestion, and hepatomegaly.

What are the earliest signs of neonatal infection?

Alterations in feeding or decreased levels of activity or alertness. Also neonatal sepsis can present with fever or hypothermia.

Which of the following is the absolute contraindication for DPT vaccine a. Cerebral palsy b. Progressive neurological disease c. ADD d. Febrile convulsion

Answer: (b) Progressive neurological disease

Osteomalacia is due to deficiency of a. Vitamin A b. Vitamin D c. Vitamin B d. Vitamin C

Answer: (b) vitamin D

Tonic neck reflex disppears at a. 36 weeks b. 32 weeks c. 24 weeks d. 18 weeks

Answer: (c) 24 weeks

Which of the following is true regarding AML a. Prognosis is good b. Peak incidence in childhood c. Auer rods present d. Philadelphia chromosome

Answer: (c) Auer rods present.

Which of the following is false regarding neuroblastoma a. Hepatomegaly b. Hypertension c. Cafe au lait spots d. Opsomyoclonus

Answer: (c) Cafe au lait spots

Drug of choice in temporal lobe epilepsy is a. Ethosuximide b. Phenytoin c. Carbamazepine d. Phenobarbitone

Answer: (c) Carbamazepine

Organism which commonly causes meningitis in 6 months to 2 years of age is a. Staphylococcus b. Neisseria c. Hemophilus infleunzae d Streptococcus pneumoniae

Answer: (c) Hemophilus influenzae

Best prognosis in neonatal seizures is due to which of the following causes a. Hypomagnesemia b. Hyponatremia c. Hypocalcemia d. Hypoglycemia

Answer: (c) Hypocalcemia

All of the following causes of neonatal convulsion have bad prognosis except a. Brain tumor b. Intraventricular hemorrhage c. Late hypocalcemia d. Meningitis

Answer: (c) Late hypocalcemia

Which of the following is false regarding neuroblastoma? a. Hepatomegaly b. Abdominal swelling c. Massive splenomegaly d. Proptosis

Answer: (c) Massive splenomegaly

Shakir's tape is used in measurement of a. Height b. Length of infant c. Mid arm circumference d. Head circumference

Answer: (c) Mid arm circumference

In which of the following conditions, iceberg phenomenon is seen? a. Filaria b. Malaria c. PEM d. None of the above

Answer: (c) PEM

A 3-yr-old child presented with fever and a history of recurrent seizures in the past . Which of the following steps should be taken to prevent recurrence of seizueres? a. Paracetamol 6-hourly b. Phenobarbitone c. Paracetamol 6 hourly and diazepam 12 hourly d. Continue diazepam infusion

Answer: (c) Paracetamol 6 hourly and diazepan 12 hourly

Drug not used in absence seizure is : a. Ethosuximide b. Valproate c. Phenytoin d. Clonazepam

Answer: (c) Phenytoin

Most common cause of neonatal death in India is a. Birth injury b. Malabsorption c. Prematurity d. Congenital malformation

Answer: (c) Prematurity

Danger in epiglottitis is a. Defect in drinking b. Defect in speech c. Respiratory obstruction d. Aspiration

Answer: (c) Respiratory obstruction

A half-year-old infant develops high grade fever which subsided after 3 days following which there was apperance of a generalized rash which also subsided after 48 hours without any residual pigmentation. Most probable diagnosis is a. Rubella b. Measles c. Roseola infantum d. Fifth disease

Answer: (c) Roseola infantum

Which of the following is not a major criterion of rheumatic fever a. Chorea b. Subcutaneous nodules c. Streptococcal antibodies d. Polyarthritis

Answer: (c) Streptococcal antibodies

Which of the following is the primary aim of neonatal resuscitation? a. To induce spontaneous cry b. To retain color c. To maintain breathing d. To restore heart rate

Answer: (c) To maintain breathing

In which of the following condition left axis deviation with left ventricular hypertrophy is seen a. VSD b. Coarctation of aorta c. Tricuspid atresia d. TOF

Answer: (c) Tricuspid atresia

Which of the following causes furuncle? a. Pseudomonas b. E. coli c. Streptococcus d. Staphylococcus

Answer: (d)

Which of the following is an essential criteria of rheumatic fever a. Chorea b. Arthralgia c. Erythema marginatum d. Acute streptococcal infection

Answer: (d) Acute streptococcal infection

Rectal polyps usually present with which of the following a. Bleeding b. Foreign body c. Inutussusception d. Rectal polyp

Answer: (d) Bleeding

Which of the following is not a clinical feature of simple febrile seizure? a. Convulsions are always generalized. b. In treatment aspirin should be avoided. c. Seizures does not last> 15 min . d. Convulsions are related with degree of elevation of temperature.

Answer: (d) Convulsions are related with degree of elevation of temperature.

Which of the following is the characteristic feature of acute rheumatic fever a. Hemoptysis b. Tender nodule c. Pansystolic murmur d. Erythema marginatum

Answer: (d) Erythema marginatum

In peti mal (absence) epilepsy, the drug of choice is a. Phenobarbitone b. Phenytoin c. Valproic acid d. Ethosuximide

Answer: (d) Ethosuximide

Which of the following is false regarding thalassemia major? a. Target cells present b. Increased fetal hemoglobin c. Microcytosis d. Iron deficiency

Answer: (d) Iron deficiency

Which of the following is false regarding Gilbert's syndrome a. Unconjugated hyperbilirubinemia b. Normal liver biopsy c. Normal LFT d. Mild conjugated hyperbilirubinemia

Answer: (d) Mild conjugated hyperbilirubinemia

Non a feature of Friedrich's ataxia a. Posterior column involvement b. Cortisospinal tract involvement c. Spinocerbellar tracts involvement d. None of the above

Answer: (d) None of the above

A 2-year old child whose weight is 6.7kg was brought to the hospital with a history of diarrhea and vomitting for 2 days. On examination the skin when pinched up took almost 3 seconds to return to normal. Most likely diagnosis is a. No dehydration b. Some dehydration c. Severe dehydration d. Only skin pinching is not reliable in making a diagnosis

Answer: (d) Only skin pinching is not reliable in making diagnosis.

In children MCC of bleeding per rectum a. Hirchsprung's disease b. Foreign body c. Intussusception d. Rectal polyp

Answer: (d) Rectal polyp

Which of the following is false regarding tricuspid valve atresia a. Severe cyanosis b. Left ventricular hypertrophy c. Left axis deviation d. Right axis deviation

Answer: (d) Right axis deviation

An eight-year-old girl presented with low grade fever, vomitting, headache and convulsions. Her CSF finding revealed protein 105 mg%, sugar 40mg%, chloride 52 mg% and predominant lymphocytes. The diagnosis i s a. Traumatic meningitis b. Viral meningitis c. Acute bacterial meningitis d. Tubercular meningitis

Answer: (d) Tubercular meningitis

Rib notching is seen in a. Tricuspid regurgitation b. ASD c. VSD d. Coarctation of aorta

Answer: (d) coarctation of aorta

At shakir's tape the red zone indicates ___cm a. 14.5 b. 13.5 c. 11.5 d. less than 12.5

Answer: (d) less than 12.5

This class of drugs in GERD acts as a buffer for HCl and increases LES pressure.

Antacids (Mylanta)

Immune Thrombocytopenic Purpura

Antecedent viral infection, petechiae, ecchymosis, mucocutaneous bleeding, thrombocytomenia, smear shows megakaryocytes, IVIG or Glucocorticoids if bleeding

In peds populations this infectious disease presents with non-tender unilateral lymphadenopathy in a systemically well child. Long term presentations: Weight loss, failure to gain weight, and wasting syndrome, leukopenia, hepatosplenomegaly, and persistent generalized lymphadenopathies

Atypical mycobacterial dz

Myotonic Dystrophy

Autosomal dominant, facial weakness, foot drop, cataracts, testicular atrophy

Friedreich ataxia, presentation?

Autosomal recessive disorder that begins before 22 years of age. Symptoms included gait ataxia, frequent falling, dysarthria, and conditions such as concentric hypertrophic cardiomyopathy, diabetes and skeletal deformities such as scoliosis and "hammer toes".

What strain of flu has caused epidemic infection in birds and has been transmitted from birds to humans?

Avian influenza A subtype (H5N1)

What is the treatment for perioral dermatitis, a common skin disorder in young women that presents with micropapulovesicles that spare the vermillion border?

Avoid topical steroids. Topical tx: Metronidazole .75% gel BID or Azelaic acid BID, erythromycin 2% gel BID. Systemic tx: Minocycline/Doxycycline 100mg daily until clear, then 50mg daily for another 2 months

What is the treatment for Urticaria, a wheal and flare with edema that extends to subcutaneous tissue?

Avoid triggers, Epi, Antihistamines, steroids should not be routinely used

In Von Willebrand's disease bleeding occurs in nasal, sinus, vaginal and GI mucous membranes. Common cause of menorrhagia. Spontaneous hemarthrosis and soft-tissue bleeds are rare except type II and hemophilia A. What will exacerbate the bleeding? What will decrease it?

Bleeding is exacerbated by aspirin or NSAIDs and decreases with use of estrogen or pregnancy.

Mumps

Parotitis, testicular inflammation *most common*, meningitis, pancreatitis, encephalitis

Pt has a seizure that was not observed. History includes LOC, enuresis and bite marks on tongue. Is this more of a partial seizure with generalization or complex partial seizure?

Partial seizure with generalization are more associated with biting of the tongue and enuresis.

Cat bite

Pasteurella multocida, S. aureus, Tx with Amox/Clav

Describe the presentation of tinea capitis

Patchy, fine, white, adherent scales with accompanying adenopathy in an infant or toddler.

Fragile X syndrome. Symptoms and pathogenesis?

Path: Increased CGG repeats Appearance: Large head, long face, prominent forehead and chin with protruding ears), joint laxity and large testes. Behavior: hyperactivity, short attention span and autism.

Patient presents with persistent, progressive, projectile (non-bilious) vomiting that may be blood tinged, poor weight gain, visible peristaltic waves, palpable pyloric "olive" located in mid epigastrium after vomiting. What is the likely diagnosis?

Pyloric Stenosis

IF you give amoxicillin for Strep Pharyngitis to someone who has EBV, what happens?

RASH! is not penicillin allergic

You can diagnose allergic rhinitis in vitro using what test?

RAST

Acute bronchiolitis is caused by what virus?

RSV Also due to parainfluenzae, influenza, adenovirus and mycoplasma

NOTE:

C3 is the major opsonin of the complement system; its deficiency results in infections with encapsulated organisms

Di George syndrome

CATCH-22, Congential heart (Tetralogy of Fallot), Abnormal facies, Thymic dsyplasia, Cleft Palate, Hypocalcemia, (Chromosome 22)

Di George Syndrome

CATCH-22, Congential heart (Tetralogy of Fallot, Truncus Arteriosus), Abnormal facies, Thymic dsyplasia, Cleft Palate, Hypocalcemia, (Chromosome 22)

Child with breath holding spells. What labs should you order and why?

CBC and ferritin as iron-deficiency anemia is often associated with this.

Patient presents with Weight loss, wheezing and salty taste on skin. Patients typically have a hx of recurrent pneumonia, sinusitis or asthma. Clubbing of fingers, increased AP chest diameter, apical crackles. Likely diagnosis?

CF

Your infant keeps presenting to the clinic with frequent, bulky, foul-smelling stools that may be oily, and failure to thrive or poor weight gain due to malabsorption of fat and protein. Also can see pancreatitis, and related diabetes...what is the likely dx?

CF

Cystic Fibrosis

CFTR defect, respiratory and pancreatic infections, meconeum ileus is almost pathognomonic, male infertility (congenital bilateral absence of vas deferens)

Pertussis

Catarrhal stage (first 1-2 weeks) - common cold Paroxysmal (2-6 weeks) - coughing paroxysms with inspiratory "whoops", posttussive emesis, risk of apnea and death in infants < 6m onths Lymphocytosis Macrolides reduce severity and contagiousness, first-line

Lesch-Nyhan Syndrome

Choreoathetosis, self mutilative behavior, renal failure, hyperuricemia, gouty arthritis, mental retardation, hypoxanthine-guanine phosphoribosyl transferase

Brodie's abscess

Chronic bone abscess, usually metaphyseal, distal tibia, irregular radiolucent area with no adjacent osteoblastic reaction

Recurrent or unusual lymphadenitis, hepatic abscesses, osteomyelitis at multiple sites and unusual infections wit catalase positive organisms in a 2 year old. Diagnosis? Test? Treatment?

Chronic granulomatous disease. Nitroblue tetrazolium slide test (NBT) TMP-SMX and gamma-interferon TIW

Seborrheic Dermatitic

Chronic, common inflammatory papulosquamous dx OFTEN begins on the scalp as craddle cap other areas include: eyebrows, nasolabial folds, eyelashes, paranasal skin described as transparent to yellow papules with occasional scaling plaques TX: moisturizers, anti-fungal, and topical steroids

Regarding vaccinations of neonates how are they given to preterm babies?

Chronologically (when they are born you start the timer then and there, not their gestational age).

Deficiency of Vitamin C can cause Scurvy, what are good sources of Vitamin C?

Citrus fruits, Tomato, strawberries, broccoli, greens

Patau Syndrome Trismomy 13

Cleft lip, flexed fingers, polydactyly, bulbous nose, hypoplastic ribs, cardiac malformations, genital abnormalities

Best medication to give a child with unilateral cervical adenitis of acute onset or associated cellulitis or suppuration?

Clindamycin Note: this is to treat staph aureus and strep pyogenes.

Unilateral cervical adenitis in an older child with dental caries. Treatment?

Clindamycin and amoxicillin/clavulanate.

Floppy baby.

Clostridium botulinum.

Chediak-Higashi

Coagulopathy, pancytopenia, neutropenia, giant lysosomes, frequent s. aureus infections

Hurler Syndrome

Coarse facies, large tongue, flat nasal bridge, short neck, mental retardation, HSmegaly, umbilical hernia, corneal clouding

Why is whole-bowel irrigation sometimes instituted for acute iron poisoning via ingestion?

Complications of pyloric stenosis and risk of gastric scarring that occurs within 2-8 weeks after ingestion.

What are complications of DMI?

Complications: DKA (tx with IV fluids, regular insulin, correct electrolyte and treat underlying cause)

Anemia is reduction in RBC mass or blood hemoglobin conc. Reduction in Hct < __ in F and < ___ in males, or Hgb < ___ in females and < ___ in males defines anemia.

Reduction in either Hematocrit (<36% in females, 42% in males) or Hgb (<12 mg/dl in females, < 14 mg/dl in males)

How is subluxation of radial head tx'ed?

Reduction is done by placing thumb over the radial head and supinating the forearm. If successful, child will begin using arm immediately. Results in "pop"

Large binucleated cells with prominent nuclei are called ___________ cells in Hodgkins Lymphoma.

Reed-Sterberg cells

These cells noted in the in lymph node tissue makes the diagnosis of Hodgkins Lymphoma.

Reed-Sternberg cells

What is the treatment for colic in infants?

Relief typically with passage of feces or flatus. Holding infant upright or prone across the lap occasionally helps. Attacks may be prevented with change in diet and improved feeding techniques. Support of parents needed.

Congenital Syphilis

Transplacental, initially asymptomatic, maculopapular rash involving palms & soles, rhinitis, lymphadenopathy, anemia, thrombocytopenia, meningitis, chorioretinitis, Huthinson teeth, mulberry molars, perforated hard palate, saber shins, hearing loss, saddle nose, interstitial keratitis, diagnosed by dark field, RPR, VDRL,

Murmur of Tetralogy of Fallot

Right ventricular outflow tract murmur = harsh, systolic ejection murmur over the left upper sternal border. Poor blood flow through a stenotic pulmonary tract results in a single S2 (no separation of S2)

T or F: In appendicitis, A free perforation leads to suppurative peritonitis with toxicity. Septic thrombophlebitis (pylephlebitis) of the portal venous system is rare and suggested by high fever, chills, bacteremia, and jaundice.

T

The persistence of lactase production past the age of 12 years is common only in northern Europeans; for most of the world, lactase does not persist and lactose-containing products are not well digested. T or F.

T

Ventricular Septal Defect: Symptoms vary with size of defect and range from being asymptomatic to presenting signs of CHF. T or F?

T

Congenital HSV

Transplacental, perinatal, vesicular lesions, keratoconjunctivitis, cataracts, chorioretinitis, seizure, tremor, full fontanelle, pneumonia, necrotizing enterocolitis, DIC, diagnosed by PCR of CSF, IgM titre, culture

Egg on a string on x-ray

Transposition of the great vessels.

What should be done before the treatment of congenital hypothyroid? What is the treatment for congenital hypothyroid?

Treat sooner rather than later. Levothyroxine 10 ug/kg. Bone scan to obtain baseline bone age and thyroid scan should be obtained before starting tx.

What is the treatment for pinworms?

Treat with mebendazole, pyrantel pamoate, or albendazole as a single dose repeated in 2 weeks. Encourage personal and family hygiene. Family member treatment should be done concurrently. Hand washing after defecation and before meals is imperative!

Burn patient who presents with ssx of cyanide poisoning should be treated with..

Treat with nitrite-thiosulfate antidote.

How can you prevent erythroblastotis fetalis? How can you treat it?

Treat: exchange transfusion Prevention: RhoGam (Human anti-D globulin) to mother.

Management for a child < 2 years of age with febrile UTI?

Treatment with 2 weeks of antibiotics and renal and bladder ultrasound to evaluate for anatomic abnormalities.

1 week cyanotic infant with left axis deviation and small or absent R waves in precordial leads. Decreased pulmonary marking on CXR. Holosystolic murmur present at left lower sternal border. Diagnosis and why?

Tricuspid valve atresia. Decreased pulmonary circulation and left axis deviation would support this diagnosis over tetralogy of Fallot and truncus arteriosus.

Sturge-Weber Syndrome

Trigeminal cavernous hemagioma, seuzires, retardation, intracranial calcificaitons

What is the etiology of DIC?

Triggered by endothelial cell injury or release of tissue factors that activate the coagulation cascade. Can also be caused by: Obstetric complications (amniotic fluid embolism, retained dead fetus, abruption placentae), Transfusion reactions, Malignancy (pancreatic carcinoma, adenocarcinoma, acute promyelocytic leukemia), Trauma (brain injury, crush injury, burns), Infection or sepsis (gram negative bacteria), Acute pancreatitis, ARDS

Is it true? All of those exposed to pertussis need to prophylactically take a macrolide.

True

T or F. Direct hernias are rare in children and usu. follow an indirect inguinal hernia repair.

True

T or F. Pityriasis Rosea is usu. self-limited and resolves on its own in 6-12 weeks.

True

In glioblastomas, these tumor markers will be elevated in urine.

Tumor markers, homovanillic acid and vanillylmandelic acid are elevated in the urine.

Primary amenorrhea, delayed puberty, short stature and bicuspid aortic valve. Diagnosis?

Turner syndrome.

Coarctation of the aorta has a male to female ratio of 2:1. When it occurs in a female, you must consider this syndrome.

Turner's syndrome

What is the treatment for paraphimosis?

Tx: Manual reduction follow up elective circumcision Emergent circumcision

There are three types of Von Willebrand's Disease. What type accounts for 75-90% of cases?

Type 1

Mild Metatarsal Adductus (a congenital foot deformity)

Type 1 = overcorrects into abduction with passive and active movement => reassurance, fixes itself Type 2 = into neutral position with movement => orthosis or corrective shoes first, then can try casting if that doesn't work Type 3 = rigid => serial casts

follicular conjunctivitis and pannus (neovascularization) formation in the cornea with a concurrent infection in the nasopharynx, leading to nasal discharge

Trachoma (Chlamydia trachomatis A-C); diagnosis made by Giemsa stain exam of conjunctival scrapings TX: topical tetracycline or oral azithromycin

5 yo boy with left hip pain. Recent history of runny nose and congestion. VS WNL. Lymphocyte elevated with normal WBC. CRP and ESR mildly elevated. Bilateral hip X-rays are normal. Diagnosis? Treatment?

Transient synovitis. Ibuprofen, rest and follow up in 1-week.

X-ray finding for transient tachypnea of the newborn? How about respiratory distress syndrome? How about persistent pulmonary HTN?

Transient: bilateral perihilar linear streaking. RDS: diffuse, reticulogranular appearance, air bronchograms, and low lung volumes. P. HTN: clear lungs with decreased pulmonary vascularity.

Wheal and flare. Edema extends to subcutaneous tissue. Etiology: cold, solar, cholinergic (exercise, heat), physical (dermatographism), allergy, infection, mycoplasma, strep, hepatitis, thyroid disorders, neoplasm What is the likely diagnosis?

Urticaria

What can cause Reye Syndrome, a complication of influenza

Use of aspirin

This type of GN occurs after respiratory tract symptoms and is characterized by fever, malaise and weight loss. ANCA will be positive and the treatment is corticosteroids and cyclophosamide. What kind?

Wegner's granulomatosis

From birth to 1 year what is expected in terms of growth?

Weight should triple and heigh should increase by 50%

This type of hearing loss is caused by a lesion in the auricle. This type of hearing loss is caused by a lesion in the inner ear.

Conductive, SN (or 8th CN nerve)

For the 1st week of an infants life, how many wet diapers a day are they suppose to have?

Depending on the day take the day of life and that's how many diapers per day these children are suppose to have wet diapers.

Thymic hypoplasia is causes what syndrome? What are these patients at risk for?

DiGeorgie syndrome. Viral and fungal infections.

Epiglottitis

Distress, dysphagia, drooling - intubate

The cremasteric reflex is absent in testicular torsion. What is the cremasteric reflex?

Examiner strokes or pinches medial thigh Stimulus usually causes cremasteric muscle contraction Observe for rise of the Testicle on same side (normal)

Nasolcrimal duct obstruction

Excessive tears, non-purulent

Child with non-tb mycobacteria as the etiology of a unilateral cervical adenitis. Treatment?

Excision and macrolide with or without rifampin.

Hypoplastic maxilla, long philtrum and microcephaly.

Fetal alcohol syndrome.

Septic Arthritis

Fever >38.5, unable to weight bear, WBC>12 000, ESR>40, CRP>2. Do Arthrocenteisis

Kawasaki disease

Fever >5 days, conjunctivitis, rash, edema, erythema, adenopathy, mucosal involvement (strawberry tongue)

How does one clinically diagnose Kawasaki disease?

Fever for 5 consecutive days followed y 4/5 of the following: 1. Conjunctivitis. 2. Oral mucosal changes. 3. rash. 4. extremity changes. 5. cervical lymphadenopathy. Note: peak age is 18-24 months.

Serum Sickness-like reaction

Fever, urticatia, polyarthralgia, 1-2 weeks post penicillin or TMP/SMX, Type III hypersensitivity, no mucosal involvement

Tularemia, presentation?

Fevers, chills headache and malaise with acute unilateral cervical lymphadenopathy.

What mutation occurs with Marfan's?

Fibrillin-1 (pay attention to the number)

This treatment for androgenetic alopecia should not be handled in women because it is teratogenic.

Finasterid

What labs are indicated in DM I&II?

Glycosuria, elevated glycosylated Hgb, Ketonuria (may also be noted in starvation, high-fat diets, alcoholic ketoacidiosis and fever), proteinuria and microalbuminuria (early predictor of diabetic neuropathy).

What is the most common cause of hyperthyroidism?

Grave's disease

Fetal Alcohol Syndrome

Growth retardation, small palpebral fissures, smooth philtrum, thin upper lip, short nose

What is seen on skull x-ray with patients with Sturge-Weber?

Gyriform intracranial calcification on x-ray! also port-wine stains of the face, glaucoma, seizures, mental retardation, and ipsilateral leptomeningeal angioma (cerebral malformations and tumors)

Which "bugs" can cause epilgottitis?

H. Influenzae B, s. pneumo, s.pyogenes, s. aureus

This class of medication in GERD decreases HCl secretion. Examples are cimetidine and famotidine.

H2 receptor antagonists

You suspect your patient has been infected with HepB. You know that post exposure prophylaxis with _______ can help prevent hep D infection.

HBIG

Which vaccine is administered to a newborn only if they are > 2 kg in weight?

HBV vaccine.

HIV

HIV DNA PCR: primary assay to diagnose HIV infection in children under 18 months of age =>detects HIV DNA in white blood cells =>definite exclusion of HIV with two negative assays after 1 month of age HIV Antibody Elisa: screening for HIV IgG =>initially detectable 2 weeks to 6 months after exposure Western Blot: confirms screening antibody assay by directly visualizing antibodies to virion proteins

DMI is associated with ___ genes.

HLA

This form of hepatitis remains most commonly reported vaccine-preventable disease in US among American travelers, incidence in US has declined substantially since vaccine

Hepatitis A

6 year old pt with persistent jaundice, hemolytic anemia, splenomegaly and spherocytes on peripheral blood smear.

Hereditary spherocytosis.

A herpes simplex infection involving the finger or nail area is called..

Herpes Whitlow

What is commonly associated with intestinal malrotation with midgut volvulus?

Heterotaxy, in which the intestines began their rotation from an already abnormal position and this results in final fixation in a variable position => twisting of the intestines and volvulus

What are the 2 distinct presentations of vesicourethral reflux and how are they diagnosed?

Hydronephrosis: often prenatally identified using US. UTI: kids difficult to diagnose due to nonspecific presentations in kids.

What is the treatment for atopic dermatitis?

Hydroxyzine and other antihistamines. Hydration of skin w/ petrolatum, eucerin cream or Lac-Hydrin. Topical steroids used to dec. inflammation, side effects include skin atrophy and suppression of the HPA axis in children. Skin infections with staph aureus are common and may require tx. with oral antibx such as erythromycin, dicloxacillin or 1st gen cephalosporin.

Elevated serum IgE levels, eosinophilia and coarse facial features. Disease? What is the presentation of this disease?

Hyper-IgE syndrome. Chronic pruritic dermatitis, recurrent staph infections (skin & respiratory and joints), and dental/bone/facial abnormalities.

abrupt onset of petechiae, purpura, and epistaxis. Often follows viral infection. No splenomegaly. Superficial bleeding of the skin, mucous membranes and GU tract. What is the likely diagnosis?

ITP

ALLERGIC contact dermatitis elicits a type ___ cell mediated or delayed hypersensitivity reaction. Must be sensitized to offending agent. Can spread beyond contact area or generalize. Erythema-papules-vesicles-erosions-crusts-scaling.

IV

What is the treatment for HSV encephalitis?

IV acyclovir

Management of Septic Arthritis:

IV antibiotics (vancomycin) after collecting culture, though most common bugs in children are Staph and Strep Synovial fluid aspiration with >100,000 leukocytes, with >90% neutrophils, and purulent fluid => emergency surgical drainage => complication with delay of even 4-6 hours => femoral head necrosis

o Primarily affects children, is a common, contagious, superficial skin infection produced by s.aureus. o Transmission is via skin/skin contact. Predisposing factors: warm temp, high humidity, poor hygiene.

Impetigo

Fragile X Syndrome

Low IQ, short attention span, prominent jaw, low set ears, macroorchidism

What will you see on labs of a pt with hemophilia A?

Low factor VIII activity level.. Prolonged PTT but PT, platelets and BT are normal.

Bruton (X-linked) Agammaglobulinemia

Low levels of all Ig (B-cells), normal T-cells, recurrent infections, tx with IVIG

How does rubella present in children?

Low-grade fever, conjunctivitis, coryza, cervical lymphadenopathy, Forschheimer spots (red macular papular rash roof of mouth and throat and trunk).

Where is the rash with Henoch-Schönlein purpura/HSP?

Lower extremity joints mainly from hips to ankles.

mixed aneraboic infection of the submandibular space

Ludwig's Angina

Patient presents w/ "Stage A" symptoms of painless, firm, freely mobile, localized lymphadenopathy in 1-2 nodes. Often cervical or supraclavicular, sometimes mediastinal mass. Fatigue, anorexia, weight loss, fever, night sweats, cough, chronic pruritis are common. What is the likely dx and what does the stage mean?

Lymphoma (hodgkins)

Signs of Diamond Blackfan Anemia

Macrocytic anemia with no hypersegmentation of neutrophil nucleus low reticulocyte count congenital anomalies (50%) with normal chromosomal analysis e.g. cleft palate, upper limb defects, cranio-facial abnormalities PATH: intrinsic defect in erythroid progenitor cells leading to apoptosis TX: corticosteroids, then try transfusion

Beckwith-Weidemann Syndrome

Macroglossia, macrosomia, omphalocele, ear creases, neonatal hypoglycemia

What antibiotic do you use for Bordetella pertussis?

Macrolide.

Nevus simplex. Presentation?

Macular stain, salmon patch looking skin finding. Blanchable, pink-red patches most commonly seen on eyelid, glabella, and midline of the nape of the neck.

What is the treatment for epiglottitis?

Maintain a patent airway, Humidified O2, heliox

Rheumatic Fever

Major Criteria: Subcutaneous nodules, pancarditits, migratory arthritis, Sydenham's chorea, erythema marginatum Minor Critera: arthralgia, fever, elevated ESR, prolonged PR interval

What are the signs and symptoms of hypercalcemia?

Major sx = constipation, polyuria. In severe hypercalcemia, may note stupor, coma and azotemia (elevated BUN and SCr)

Child with mono-symptomatic (isolated) enuresis after the age of 5 with no findings of pathology and is undergone multiple behavior modifications and an enuresis alarm. Next best step?

Medicines: 1st line: desmopressin. 2nd line: TCAs

ABG and CXR is useful to diagnose hyaline membrane disease. What abnormalities will you see on ABG?

Metabolic and Respiratory Acidosis

What embryonic tissue do Wilms' tumor originate from?

Metanephros

Congenital Varicella Syndrome

Microcephaly, limb hypoplasia, IUGR, and cataracts with a maternal hx of a pruritic, generalized vesicular rash

Congenital rubella syndrome. Define it.

Microcephaly, microphthalmia and meningoencephalitis. with sensorineural deafness, cardiac abnormalities (PDA & ASD), congenital glaucoma and cataracts.

Lead intoxication

Microcytic hypochromic anemia, peripheral neuropathies

How does pertussis present chronologically?

Mild cough and rhinitis for 1 to 2 weeks. (Catarrhal) Coughing paroxysms with inspiratory "whoop", posttussive emesis, apnea and cyanosis (Paroxysmal) Resolution of symptoms sometime weeks after or months. (Convalescent)

Deficiency of Vitamin D can cause rickets and osteomalacia. What are good sources of Vitamin D?

Milk, liver, eggs, salmon, tuna

Nephrotic Syndrome

Minimal Change dx - maltese crosses (oval fat bodies) Focal Segmental Glomerulosclerosis: MC 2-7 HTN and nephrotic syndrome. Doesn't respond to steroids.

This drug rxn is most frequent and very similar to viral xanthems. Due to ampicillin, isoniazid, phenytoin, quinidine, sulfonamides and thiazides. Occur 7-10d after starting the drug. Macpap eruption, red macules and papules become confluent and often spare the face. Itching is common. (Urticarial, Mobilliform, Fixed Drug)?

Mobilliform

NOTE:

Mom's with lupus are associated with congenital third-degree AV block in infants

Blue-grey sacral patches. Concerning?

Mongolian spots are benign.

Enterobious

Most common helminthic infection, nocturnal perianal pruritis, treat with Albendazole, treat household contacts

Gilbert Syndrome

Most common hereditary hyperbilirubinemia (unconjugated)

What is the most common location of obstruction in foreign body aspiration?

Most common location of obstruction is right main stem bronchus

Sarcoma botyroides

Most common lower genital tract malignancy in young girls

Freidrich Ataxia

Most common spinocerebellar ataxi, gait, hypertrohic cardiomyopathy, cervical spinal cord atrophy, diabetes, scoliosis, hammer toe, most die of cardio/resp, autosomal recessive

Significant asymmetric breath sounds indicates what?

Most likely a foreign body aspiration with unilateral airway obstruction Cough, tachypnea, focal wheeze on one side, retractions, and being afebrile are common

What is the treatment for a VSD?

Most small VSDs close without intervention by age 10. Large VSDs require surgical closure

Congenital Cytomegalovirus

Transplacental, asymptomatic, periventricular calcifications, IUGR, microcephaly, hearing loss, retinitis, Hsmegaly, thombocytopenia, poor suck, diagnosed by culture and PCR,

Congenital Rubella

Transplacental, blueberry muffin rash, triad of [cataracts, PDA, hearing loss], salt and pepper retinopathy, PDA long bone disease, IUGR, glaucoma, jaundice, HSmegaly, diabetes mellitus, maternal rash diagnosed by culture and IgM titre, supportive care,

Congenital Toxoplasmosis

Transplacental, fecal-oral, cat feces, first trimester death, triad of [hydrocephalus, intracanial calcifications, chorioretinitis], microcephaly, hearing loss, Hsmegaly, anemia, diagnosed by placenta, serum, CSF, PCR, IgM

The diagnosis of foreign body aspiration is sometimes delayed up to 1 mo after aspiration. CXR may reveal obstructive asymmetric hyperinflation caused by which effect?

check valve effect

Biliary cyst, presentation? Treatment?

child of < 10 years of age presenting with pain, jaundice and palpable mass. Resection to relieve symptoms and prevent malignant transformation.

Patients with chronic otitis media present with persistent or recurrent purulent otorrhea with TM performation. They may also have a mass of squamous epithelium debris that forms at the site of invasion called a _______

cholesteatoma

What disease causes patients to be mainly at an increased risk of Aspergillus and staph infections?

chronic granulomatous disease/CGD as both of these infections are catalase positive.

A 2-year-old child presented with respiratory distress. On examination there is wheezing. Most likely diagnosis is a. RSV infection b. Severe pneumonia c. Adenovirus d. Rhinovirus

a. RSV infection

Roseola Infantum

a.k.a. sixth disease, exanthem subiticum caused by HHV-6 => HIGH fever for 3-5 days in a well-appearing child; may have rhinorrhea => followed up with abrupt resolution of fever and development of maculopapular rash => bulging fontanelle (often leads to eval for meningitis) => may cause febrile seizures (20-30% of first febrile seizures in children)

Other causes of direct hyperbilly?

always r/o SEPSIS; also, galactosemia, hypothyroid, choledochal cyst, CF

Infants less than 2 months are at high risk of developing what conditions due to bronchiolitis?

apnea and respiratory failure.

Myasthenia Gravis

autoimmune fatigable muscle weakness, improves with rest, ptosis, diplopia, dx with EMG studies, edrophonium test

False regarding rheumatic fever in India is a. Most common MR b. Aortic root is commonly involved c. Progresses rapidly d. Starts in young age

b. Aortic root is commonly involved

Most common complications of mumps in children is: a. Orchitis b. Aseptic meningitis c. Myocarditis d. Pancreatitis

b. Aseptic meningitis

Visual acuity loss is a red flag in conjunctivitis and you should be concerned about..

concerns about *infectious keratits*, iritis, angle closure glaucoma

Low frequency hearing loss is usu. _________, high frequency hearing loss is usu. ___________

conductive SN

Cretinism is also known as...

congenital hypothyroidism

In childhood thyrotoxicosis, treatment of choice is: a. Radioiodine b. Lugol's iodine c. Carbimazole d. Surgery

d. Crown rump length

Sitting height is equal to: a. Head circumference b. Upper segment c. Chest circumference d. Crown rump length

d. Crown rump length

Which of the following is a live attenuated vaccine a. ATS b. Salk c. BCG d. DPT

d. DPT

Williams Syndrome

deletion on chromosome 7 loquacious "cocktail party" personality, elfin facias supravalvular aortic stenosis hypercalcemia

What is the treatent for vWb Dz?

desmopressin/DDAVP for episodes; vWF- containing concentrate (Humate P) • DDAVP is a synthetic analogue of ADH that induces the release of vWF.

Galactosemia

develop vomiting and diarrhea after feeding + profound hypoglycemia and hepatomegaly, seizures + vulnerable to E. coli sepsis

Pinealoma Symptoms

develops in the dorsal aspect of the midbrain + endocrine syndrome + intracranial HTN + Parinaud's syndrome (paralysis of vertical gaze) + Collier's sign (retraction of the eyelid)

Patient presents with vomiting 24-48 hours after birth usually but can be delayed for days or weeks in infants with partial obstruction due to stenosis. Gastric distension and vomiting that may be bilious. Affected infants may pass meconium. What is the likely diagnosis?

duodenal atresia

Divergent, outward misalignment of the eye in strabismus is termed...

exotropia

These two syndromes can cause jaundice with decreased rate of conjugation and normal reticulocyte count.

from Gilbert or Crigler-Najjar syndrome

How do you diagnose GERD?

hx of heartburn + response to acid suppressing drugs

A deficient of surfactant will cause this newborn breathing disorder...

hyaline membrane disease

What is the most common cause of respiratory failure in the 1st few days of life?

hyaline membrane disease

Isolated premature adrenarche

increased adrenal androgen secretion; slightly increased levels of DHEA-S generally benign but a significant risk factor for: + T2DM + PCOS + metabolic syndrome pubarche, axillary hair, acne, oily skin and hair, and odor in girls <8 and boys <9 with obesity

What are common precursors to epistaxis?

infection, trauma, allergic rhinitis, atrophic rhinitis, hypertension, tumors

Ciliary flush, foreign body sensation, photophobia and corneal opacity in conjunctivitis are all red flags and should make you concerned for

infectious keratitis

Acne characterized by papules and pustules is inflamm or non inflamm? Acne characterized by comedones is inflamm or non inflamm?

inflamm, non inflamm

Prenatal exposure to Cocaine looks like:

jitteriness, excessive suckling, and hyperactive Moro reflex

Mucutaneous lymph node syndrome is also called...

kawasaki disease

Patient presents with bilateral conjunctivitis, cervical lymphadenopathy, truncal polymporphous rash, oropharynx mucosal chains injection, dry fissured lips, strawberry tongie, peripheral edema and desquamation...what is the likely diagnosis?

kawasaki disease

This is a disease of inflammation of medium and small size vessels;and is one of the most common vasculitudes in children.

kawasaki disease

Lesser known findings of Kawasaki

low album high platelets sterile pyuria transaminitis (if hyperbili check for hydrops of gallbladder) CSF pleocytosis Dyslipidemia Aseptic meningitis (don't give IVIG)

Thalaseemia lab findings:

low low MCV, high levels of iron and ferritin, low TIBC, and high high transferrin saturation (iron/TIBC) DUE TO high red blood cell turnover

CF causes thick viscous secretions in what areas of the body?

lungs, pancreas, liver, intestine, and reproductive tract, and to increase salt content in sweat gland secretions.

Kawasaki disease most commonly affects what demographic?

male infants

Lymphoma (hodgkins) has a 4:1 (male or female) predominance in the 1st decade of life? Which race is it LEAST common in?

male, blacks

Hodgkin Lymphoma

malignant proliferation of lymphoid cells ,Reed-Sternberg cells, splenomegaly, asymptomatic lymphadenopathy, associated with EBV

Patients with chronic OM may have a mass of squamous epithelium debris that forms at the site of invasion (cholesteatoma). This can lead to what complications?

meningitis, brain abscess

NOTE:

meningococcal conjugate vaccine (MCV-4) is recommended for all children 11-18; and college freshman and military recruits give booster dose if given before 16 years

Signs of severe asthma exacerbation:

minimal air exchange and absence of wheezing due to poor airflow, cyanosis and pulsus paroxdoxus; can have focal wheezing with mucus plugging, though normally diffuse

In conjunctivitis, if the causative organism is ________ it may cause copious purulent discharge. If the causative organism is __________ it may cause mucopurulent discharge.

neisseria chlamydia

Chlamydia Trachomatis pneumonia

neonates 1-3 months of age, staccto cough, 50% have conjunctivitis, afebrile, elevated eosinophils on CBC TX: PO erythromycin or azithromycin

Gaucher's disease

neurodegeneration, splenomegaly, and bony changes (esp Erlenmeyer flash-shaped distal femur)

1 year old girl with feeding problems since birth, freckles in her armpit and groin, and cafe-au-lait spots. Diagnosis?

neurofibromatosis 1.

Are seizures and vomiting associated with congenital hypothyroidism?

no

How do you diagnose and treat PTA?

o Diagnosis: Needle aspiration. o Treatment: Needle aspiration or incision and drainage may be required. Systemic antibx = penicillin. Clindamycin are required.

In what views should the legs be visualized in avascular necrosis of the femoral head?

o Diagnosis: XR (AP and frog leg view) of the hip reveals a flattened femoral head w/ joint space narrowing. MRI and bone scan may reveal early changes.

What is the treatment for an anterior nosebleed?

o Direct pressure = compress elastic area of nose for 10 - 15 minutes o Vasoconstrictive agents= phenylephrine, oxymetazoline or epinephrine o Nasal packing = When other methods fail. Consider prophylactic tx against TSS with cephalexin, clindamycin or augmentin. o Cautery with silver nitrate or electric.

What is the normal distribution of lichen planus/

o Distribution; cutaneous lesions form on flexor surfaces of wrists, forearms, legs. Oral lichen planus, genitalia, scalp lichen planus, nail lichen planus

What is the best way to dx congenital hip dysplasia? What is the best way to treat it?

o Dx: US most accurate. XRays reliable in 4-6 old o Rx: Pavlik harness to 6 month old, older child may need spica cast or open reduction.

Where does scabies most commonly present?

o Fingers, wrists, toes

most common vasculitis syndrome in children. Primarily males age 3-15 years. Hallmark is palpable purpura: maculopapular rash with progresses to diagnostic purpuric rash on ankles, elbows, and buttocks. Micragtory arthralgia, abdominal colic, hematuria

o Henoch-Schonlein Purpura (HSP):

What will lab studies reveal in pyloric stenosis?

o Labs may reveal hypocholoremic alkalosis with hypokalemia.

What is the most common valvular residual of rheumatic fever?

o Mitral insufficiency is the most common valvular residual of acute rheumatic fever. Mitral stenosis is more commonly seen 5-10 years after RF and therefore more commonly seen in adults. Aortic insufficiency is the second most common valve affected.

Phimosis is the inability to retract foreskin. What is the difference b/t physiologic and pathologic phimosis?

o Physiologic phimosis: seen in almost all newborn males, due to development of congenital adhesions between foreskin and glans. Physical exam reveals a pliant unscarred preputial orifice o Pathologic phimosis: foreskin that is truly nonretractable secondary to distal scarring of prepuce. Scarring appears as a contracted, white fibrous ring around the preputial orifice

How should you rspond to an infant with a positive Barlow or Ortolani?

o Positive Barlow and/or Ortolani maneuvers require bilateral US of hips and referral to a pediatric orthopaedic surgeon to prevent a lifelong disability.

What is the treatment for a posterior nose bleed?

o Posterior nasal packing - Carry a significant morbidity, includes difficulty swallowing, otitis media, sinusitis and hypoxia. Embolization, Ligation Sitting up, leaning forward, pressure, topical vasoconstrictors, intranasal tampon-ENT paged for posterior packing if tampon fails. Other- foley, nasal sponge/tampon, inflatable nasal balloon catheter, rhino rocket.

What is the "bug" we call scabies?

o Sarcoptes scabiei

When does osgood-schlatter's usu. resolve?

o Self-limiting dz, usu. heals when epiphysis closes.

What is the ann arbor staging for hodgkins lymphoma?

o Stage I - 1 lymph node region/structure o Stage II - 2 or more lymph node regions on same side of diaphragm o Stage III - Lymph node regions or structure on both side of diaphragm. o Stage IV - Involvement of other organs such as liver, bone marrow and CNS.

What is tx for SCFE? What are the longterm complications of SCFE?

o Treatment: Non wt bearing and on crutches, ultimately -> surgical repair with pinning (ORIF) o long term complications = avascular necrosis

Causes of advanced bone age?

obesity hyperthyroidism precocious puberty (neoplasm) CAH

cognitive

object permanence => 0-4 months, won't look for fallen object but later will look down => 4-8 months, reach for partially hidden objects => 8-12 months, look for completely hidden objects causality - what makes things happen by 12 months (begin to understand how toys are activated), will play with toys appropriately

Many males with CF are infertile due to...

obstructive azoospermia

Osteomalacia is a clinical manifestation of vitamin D deficiency. How does it present?

older adolescents and adults, growth is compete, epiphyseal plates are fused, and there is usually reserve mineral. • Asymptomatic or muscle and bone pain

Pathologic Jaundice =

on 1st DOL, billy > 12, d-billy > 2, rate of rise > 5/day

What part of long bones is osteosarcoma usu. found in? What part of of long bones is ewing's sarcoma usu. found in?

osteosarcoma - metaphyseal areas ewing's sarcoma - diaphysis

11 year old Allie has a history of Cystic Fibrosis and presents to your clinic with symptoms of pneumonia? Which causative agents do you suspect?

p. aeruginosa, p. cepacia

Diarrhea (and general GI complaints), Dementia (range from poor concencreation or irritability, aggressiveness), Dermatitis (symmetric, bilateral, like a sunburn)

pallegra caused by Niacin deficiency, which can occur with bowel disease that interferes with vitamin absorption

What will you find on PE of the pt with encopresis?

palpable stool throughout the distribution of the colon, especially in the left lower quadrant, stool smeared around the anus, lax and patulous anal sphincter, rectum typically enlarged and filled with soft stool that yields negative results on fecal occult blood testing.

What condition of the blood is usu. present in tetralogy of fallot?

polycythemia

What are the clinical manifestations of hyperthyroidism?

tachycardia, appetite changes, diarrhea, DOE, HA, fatigue, heat intolerance, nervousness, palpitations, weight loss. PE = pretibial myxedema, exophthalmos, lid lag and hyperreflexia. Goiter with bruit is noted. May present with AF.

In intussuception, US will show single or hypoechoic ring with hyperechoic center...this is called the...

target/donut sign

Language: Waves bye-bye, Uses four or five words

15-17 months

Personal/Social: Attempts use of spoon

15-17 months

Moro's reflex disappears by __ month a. 7 b. 3 c. 5 d. 8

Answer: (b) 3 months

T or F. Phototherapy is beneficial in all cases of jaundice.

T

T or F. Posterior cervical lymphadenopathy may be noted in lice.

T

What type of regurgitation is physiologic and warrants reassurance and positioning therapy? What symptoms from the history would warrant treatment and an alternative dx?

"Happy spitter" and an otherwise asymptomatic child who regurgitates a mild amount of food. Vomiting, eczema, bloody stools (milk protein allergy) Failure to thrive, Sandifer syndrome and significant irritability (GERD) Projectile nonbilious vomiting (pyloric stenosis) Note: Sandifer syndrome is intermittent opisthotonic posturing.

What is the major "bug" that causes viral conjunctivitis?

*Adenovirus 3,8,19* Herpes Simplex 1 and 2 Picornavirus

What are medications that can cause constipation?

*Anticholinergics*, antidepressants, antihistamines, antiparkinsonian drugs, *antihypertensives*, CCBs, clonidine, cation-containing agents, Fe, Ca, Antacids, Opiates, Morphine, Codeine

What is the treatment for phimosis?

*Circumcision*. stretching exercises, topical corticosteroids-betamethasone cream. Triamcinalone or fluticasone sometimes used.

How do you manage hypospodias?

*Do not circumcise!!!!*-prepuce used for surgical repair at 18 months of age. Refer to pediatric urologist.

What is the treatment for orbital cellulitis?

*HOSPITALIZE WITH IV ANTIBX*, and possible surgery if recalcitrant ((nafcillin + metrondizaole OR clindamycin OR ceftriaxone) Vanco if MRSA suspected

What is the treatment for lice? o Body lice (Pediculus humanus) o Head lice (Pediculosis capitus) o Pubic lice (Pthirus pubis)

*Nix cream rinse (1% permethrin)* Kwell shampoo (1% lindane) Resistance to both developing and Occlusion questionable

What are the most common causes of orbital cellulitis?

*S. Pneumoniae, Haemophilus influencz*. Staph aureus, Strep pyogenes,

Prader-Willi Syndrome

+ hypogonadism, almond-shaped eyes, short stature + hyperphagia with obesity during childhood

In uncongujugated hyperbilirubinemia antibody mediated hemolysis will be coombs + or -? Non-immune hemolysis will be coombs + or -?

+, -

What are the two possible etiologies of congenital hypothryroidism?

- 1. Due to hypoplasia or aplasia of the thyroid gland. 2. Failure to secrete hormone secondary to enzyme deficiency.

A 4-day-old healthy newborn has jaundice and serum bilirubin of 22 mg/dl, treatment of choice is: a. Investigation of liver function b. Phototherapy c. Exchange transfusion d. Observation

...

Autohemolysis test is positive in which of the following conditions: a. Sickel cell disease b. Vitamin E deficiency c. Hereditary Spherocytosis d. Beta thalassemia

...

Patients with tinea cruris present with sharply demarcated erythematous lesions in the inguinal folds. What are the likely "bugs"/

. T rubrum, T. menagrophytes, Epidermophyton floccosum.

10-year-old child presented with 10 days continuous fever with soft, enlarged spleen. The diagnosis is: a. Enteric fever b. Hodgkin's disease c. Meningitis d. Malaria

...

8-year-old boy presents with swelling in left eye of 3 months duration. Examination revealed proptosis of left eye with preserved vision. Right eye is normal. CT scan revealed intraorbital extra coneal mass lesion. Biopsy revealed embroynal rhabdomyosarcoma. Metastatic work up was normal. The standard line of treatment is: a. Chemotherapy only b. Wide local lesion c. Enucleation d. Chemotherapy and radiotherapy

...

98% of neonates void within: a. 8 hours b. 48 hours c. 12 hours d. 24 hours

...

A 1 year old child having leukocoria was detected to be having a unilateral, large retinoblastoma filling half the globe. Current therapy would involves: a. Enucleation b. Chemotharapy followed by local dyes c. Direct laser ablation using photodynamic cryotherapy d. Scleral radiotherapy followed by chemotharapy

...

A 10-year-old girl presents with swelling on one knee joint. All of the following conditions can be considered in the differential diagnosis except: a. Tuberculosis b. Juvenile rheumatoid arthritis c. Hemophilia d. Villonodular synovitis

...

A 2-year-old boy has vitamin D 'refractory' rickets. Investigations show serum calcium to 9-mg/dl, phosphate 2.4-mg/dl, alkaline phosphatase 1040IU. Parathyroid hormone level and bicardonate levels are normal. The most probable diagnosis is: a. Distal renal tubular acidosis b. Hypophosphophatemic rickets c. Vitamin D dependent rickets d. Proximal renal tubular acidosis

...

A 2-year-old child has a weight of 6.4 kg and has vitamin A deficiency. The grade of malnutrition in this child is: a. Second degree b. Fourth degree c. Third degree d. First degree

...

A 2-year-old child presented with cough, fever, and dyspnea. His respiratory rate is 55/ minute, chest in-drawing present but he has no cyanosis or convulsions. Most likely diagnosis is: a. Very severe disease b. Severe pneumonia c. Pneumonia d. Cough and cold

...

A 2-year-old presented with leukocoria in the right eye since 2 months. On examination, a total retinal detachment was present in the same eye. Ultrasound B scan revealed a heterogeneous subretinal mass with calsicication, associated with a retinal detachment. The most likely diagnosis is: a. Coats disease b. Retinoblastoma c. Toxocariasis d. Retinal tuberculoma

...

A 3-month-old girl weighs 4 kg and is suffering from loose motions. On examination, she is found to be suffering from moderate dehydration. The amount of ORS to be given to her in first four hours should be: a. 600 ml b. 300 ml c. 100 ml d. 500 ml

...

A 3-year-old can not do: a. Climb upstairs b. Hobble 5 steps c. Draw a circle d. Speak sentences

...

A 4 kg infant with severe dehydration (10%) needs the following amount of IV fluid in the first 24 hours: a. 1200 ml b. 800 ml c. 500 ml d. 100 ml

...

Bacterial endocarditis is most commonly seen in: a. AS b. ASD c. VSD d. PDA

...

A 45-day-old infant developed icterus and 2 days later symptoms and signs of acute liver failure appeared. Child was was found to be positive for HBsAg. The mother was also HBsAg carrier. The mother's hepatitis B serological profile is likely to be: a. HBsAg positive only b. HBsaG and HBeAg positive c.HBsAg and anti-HBe antibody positive d. Mother infected with mutant HBV

...

A 5-year-old child can remember how many digits: a. 8 b. 4 c. 5 d. 10

...

A 5-year-old child has anemia of long duration. The investigation to be done is: a. Peripheral smear b. Estimation of hemoglobin % c. RBC count d. PCV

...

A child can copy a circle first at: a. One and half years b. 4 years c. 2 years of age d. 3 years

...

A child has recurrent hypoglycaemic attacks and hepatomegaly. He is most likely suffering from: a. Neonatal hepatitis b. Neonatal diabetes c. Von Gierke disease d. Galactosemia

...

A child presents with migratory polyarthritis, the investigation of choice to confirm the diagnosis is: a. ESR b. ASO titre c. Pharyngeal swab for culture and sensitivity d. ECG

...

A child with IQ of 50 is graded as: a. Moderately intelligent b. Moderate mental retardation c. Severe mental retardation d. Mild mental retardation

...

A child with VSD developed Eisenmenger syndrome. Correct sequence of events which leads to this change is: a. Pulmonary hyperension, right to left shunt, right ventricular hypertrophy, left to right shunt b. Pulmonary hypertension, right ventricular hypertrophy, left to right stunt, right to left shunt c.Pulmonary hypertension, left ventricular hypertrophy, left to right stunt, right to left shunt d. Left to right shunt, right ventricular hypertrophy, pulmonary hypertension, right to left shunt

...

A comatose child had garlic like odor to the breath. The most probable diagnosis is: a. Lead poisoning b. Flavism c. Atropine poisoning d. Arsenic poisoning

...

A mother while kissing her baby found that the baby's skin is salty. Baby is suffering from a. Niemann- pick disease b. Cystic fibrosis c. Thalassemia d. Fanconi syndrome

...

A neonate less than 7 days old presents with fever,features of meningitis, anorexia. CSF examination revealed protein decreased sugar and increased WBC. Most likely diagnosis is: a. Listeria monocytogenes b. Shigella c. Mycoplasma d. Mycobacterium tuberculosis

...

A patient has diarrhea. Stool examination reveals no RBC or pus cells. Identify the organism: a. Balantidium coli b. Entamoeba histolytica c. Giardia d. E. intestinalis

...

Abnormal finding seen in classical hemorrhagic disease in children is: a. Fibrinogen activated partial thromboplastin time b. Activated partial thromboplastin time c. Thrombin time d. Platelet count

...

According to Indian Academy of pediatrics, PEM gradeIII is: a. 50-60% b. 70-80% c. 60-70% d. 80-90%

...

Acute coryza is most commonly caused by: a. Rhinovirus b. RSV c. Influenza virus d. Arenavirus

...

All are false regarding Typhoid in children except: a. Urine culture positive in 4-6 days b. Mild splenomegaly is usual c. Meningitis is always seen d. Leukocytosis is characteristic

...

All are true of ' cartilage hair hypoplasia syndrome' except: a. Short-limbed dwarf b. Sparse hair c. Neutropenia d. T-cell dysfunction

...

All of the following are associated with low C3levels except: a. Post-streptococcal glomerulonephritis b. Membranoproliferative glomerulonephritis c. Goodpasture's disease d.Systemic lupus erythematous

...

All of the following are features of cerebral palsy except: a. Hypotonia b. Spasticity c. Erb's palsy d. Quadriplegia

...

All of the following are seen in atrial septal defect except: a. There may be no murmurs audible b. Right ventricular hypertrophy c. Wide split 2nd heart sound d. Pulmonary oligemia

...

All of the following cause splenomegaly except: a. Malaria b. Kala azar c. Tay-Sachs disease d. Typhoid

...

All of the following may occur in Noonan's syndrome except: a. Hypertrophic cardiomyopathy b. Cryptoorchidism c. Infertility in females d. Autosomal dominant transmission

...

All of the following milestones are seen in a 16 weeks child except: a. Kicks the ground on placing it on its feet b. Brings object to the mouth c. Polysyllabic vowel sounds d. Laughs loudly

...

An 8- month-old child presented with itchy, exudative lesions on the face, palmes and soles, the siblings also have similar complaints. The treatment of choice in such a patient is: a. Systemic ampicillin b. Topical betamethasone c. Systemic prednisolone d. Topical permethrin

...

An 8-mths old child will not be able to: a. Pincer grasp b. Says mama baba c. Stand without support d. Sitting stably

...

An eight year old boy takes part in an icecream competition comes to hospital with pain in abdominal distention and bloating. Ha also has previous similar episodes on eating ice-cream/ milk products. Which of the following enzymes is deficient: a. Maltase b. Salivary amylase c. Lactase d. Sucrase

...

An infant with the history of diarrhoea 15 days back, presented with fever, purpuric rash, anaemia and tachycardia. The diagnosis is: a. Prerenal azotemia b. Henoch- Schonlein purpura c. Idiopathic thrombocytopenia d. Hemolytic-uremic syndrome

...

Antibodies seen in a neonate after an intrauterine infection are: a. IgD b. IgA c. IgM d. IgG

...

Apt test is done for: a. Neonatal tachypnea b. Fetal haemoglobin c. Neonatal asphyxia d. Fetal maturation

...

Asymmetric Moro's reflex is not seen in: a. Hemiparesis b. Fracture of clavicle c. Sternomastoid tumor d. Brachial plexus injury

...

autosomal dominant with a deficiency of vWF = what type of von willebrands? Autosomal recessive with complete absence of vWF = what type of von willebrands? autosomal dominant with defective vWF. = what type of von willebrands

1, 3, 2

How long does the fever of influenza last?

1-7 days

What 4 defects does tetralogy of fallot refer to?

1-VSD, 2-Pulmonary stenosis, 3-overriding aorta over the VSD, 4-RVH

Evaluation of Foreign Body Aspiration

1. AP and lateral chest films 2. Bilateral decubitus or inspiratory/expiratory chest films for large airway obstruction 3. Chest Fluroscopy - dynamic test over several breaths; req radiologist and uses more radiation

What is the diagnostic criteria for kawasaki disease?

1. Acute onset fever (up to 105) for atleast 4 days. 2. 4/5 of the following= Bilateral conjunctivitis, Cervical lymphadenopathy, Truncal polymorphous rash, Oropharynx mucosal chanes injection, dry fissured lips, injected lips, strawberry tongue, Peripheral extremity changes; edema, erythema, desquamation.

When is hydroxyurea indicated for SCD?

1. Frequent, acute painful episodes. 2. Acute chest syndrome. 3. severe symptomatic anemia.

What is the treatment for acute rheumatic fever?

1. Bed rest. 2. Antibiotics DO NOT modify the course of disease. 3. Anti-inflammatory drugs (NSAIDs) suppress the ssx but are not curative (aspirin, corticosteroids, etc) 4. Decrease risk for dz by prevention and timely strep treatment. 5. Secondary prevention of rheumatic fever, prevention of recurrent attacks = penicillin G or V, or sulfadiazine. Erythomycin in PCN allergy

Management of Gonadotropin-Dependent Precocious Puberty = premature thelarche and adrenarche, advanced bone age, and elevated LH levels at baseline or with stimulation

1. Brain MRI with contrast to check for pituitary and hypothalamic tumors 2. GnRH continuous agonist therapy to prevent epiphyseal plate closure

Besides poor countries where local diet consists of cereal, corn or sorghum..what other 3 settings is niacin deficiency common?

1. Carcinoid syndrome- deficiency of active forms of niacin and development of pellagra 2. Prolonged use of Isoniazid 3. Hartnup disease: autosomal recessive congenital disorder. Dx made by detecting neutral amino acids in urine, something not seen with dietary pellagra.

What are the major criteria for Rheumatic fever?

1. Carditis 2. Erythema marginatum 3. Subcutaneous nodules 4. Chorea 5. Polyarthritis

What is the treatment for DMII?

1. Diet (even modest weight loss in the obese patient will improve glycemic control). Diet high in carbs (>50%) may improve insulin action and glycemia control in mild fasting hyperglycemia. In patients with severe fasting hyperglycemia or elevated triglycerides a diet <45% carbs is indicated. 2. Limit cholesterol < 300 mg/day 3. Daily protein 10-20% of total calories 4. Sodium < 2.4 mg.day • Oral antidiabetics, self monitoring

Differential for HTN in kids

1. Essential 2. Coarctation of the Aorta 3. Pheo 4. Fibromuscular Dysplasia 5. Conn's syndrome

What are the minor criteria for rhemuatic fever?

1. Fever 2. Polyarthalgias 3. Reversible prolongation of PR interval 4. Elevated ESR 5. Elevated CRP

What is the treatment for mild OE? What is the treatment for malignant OE?

1. Gentle cleaning with saline or alcohol and acetic acid mixture. 2. Topical neomycin, polymxin and quinolone are also started. 3. IV anti-pseudomonal antibitoics sich as ceftazidime or piperacillin, plus aminoglycosides or fluroquinolones are required for malignant OE.

Nutritional considerations:

1. Goat's milk - low in Vit D, iron, folate and Vit B12, so risk for megaloblastic anemia; supplement; boil milk to prevent brucellosis 2. Breast milk = awesome => low Vit K - give at birth to prevent hemorrhagic disease of newborn => low Vit D and fluoride => low but highly bioavailable iron - supplement at 4 to 6 months => breast-feeding vegan mothers are given Vit B12 to prevent infant developing methylmalonic acidemia

Strawberry tongue is a characteristic finding of :

1. Grp A Strep Pharyngitis => Scarlet Fever 2. Kawasaki disease 3. Toxic Shock Syndrome

Why do newborns at age 2-4 days of life have an increase in unconjugated hyperbilirubinemia?

1. High hemoglobin turnover from RBCs (fetal?). 2. Not enough uridine diphosphogluconurate glucuronosyltransferase (UGT) 3. Enterohepatic recycling is increased as the sterile newborn gut cannot break down bilirubin for fecal excretion.

What are 5 common etiologies of glomerulonephritis?

1. IgA nephropathy (Berger's dz) 2. Postinfectious GN 3. Lupus nephritis 4. Pauci-immune GN (wegner's), 5. Goodpasture's dyndome

Language: Looks for source of sound, Laugh and squeals, orients to voice, begins to make consonant sounds

4-5 months

What are the congenital anomalies associated with maternal hyperglycemia?

1. heart disease, 2. neural tube defect/spinal bifida's, 3. Small left colon syndrome, 4. hypoglycemia 5. TTN, 6. Macrocephaly more!!

Clinical manifestation of HSP?

1. Palpable purpura. 2. arthritis & arthralgia. 3. abdominal pain & intussusceptions (GI hemorrhage and this needs to be worked up) 4. renal disease similar to IgA nephropathy. 5. scrotal pain and swelling (rare)

Diagnostic features of acute bacterial rhinosinusitis

1. Persistent symptoms for >= 10 days without improvement 2. Severe symptoms: fever (>= 39 C or 102 F), purulent nasal discharge, or face pain >= 3 days 3. worsening symptoms >= 5 days after initially improving viral URI TX: augmentum (oral amoxicillin-clavulanic acid) NOTE: if symptoms persist or worsen after 3 days of antibiotics, cultures should be obtained by sinus aspiration

What factors predispose you to hyaline membrane disease?

1. Premature birth 2. Diabetic mother 3. Positive family history

What does a differential diagnosis of hematochezia (fresh blood from anus) in an early toddler include?

1. hemorrhoids. 2. infections colitis. 3. intussusception. 4. Meckel's diverticulum. 5. inflammatory bowel disease.

What is the treatment for A.L.Leukemia?

1. Remission induction a. Chemotherapy with vincristine, prednisone, anthracycline & cyclophosamide or L-asariginase 2. Post remission Therapy a. Consolidation chemotherapy with Ara-C in combination with an anthracycline. 3. Central Nervous System prophylaxis required a. Intrathecal methotrexate or brain radiation. 4. Bone marrow transplant.

What is the treatment for appendicitis?

1. Surgical removal of the appendix. 2. Broad spectrum antibiotics needed with perforation.

Fines motors: Builds tower of two or three cubes

15-17 months

Gross motors: Stoops and recovers walk well, Walk alone

15-17 months

What is the treatment for acute testicular torsion? What is the treatment for testicular torsion of the appendix?

1. Torsion treated with prompt surgical exploration and detorsion. If completed w/ in 6hrs=excellent survival of tsticles. After detorsion, the testis is fixed to scrotum by orchiopexy (on both testes). 2. Torsion of the appendix resolves in 3-10d. No surgery needed, pain control important.

How does a child present with acute iron poisoning? Treatment?

1. abdominal pain. 2. hematemesis. 3. metabolic acidosis. 4. radiopaque findings on X-ray. 5. possible hepatic necrosis (day 2) Treatment includes chelation with deferoxamine (almost seems like it is latin for de-ironing)

For the following list the most pertinent lab findings. 1. x-linked agammaglobulinemia. 2. common variable immunodeficiency. 3. IgA deficiency. 4. Hyper-IgM syndrome. 5. IgG subclass deficiency.

1. absent or low B-cells and immunoglobulins. 2. normal B-cells and low immunoglobulins. 3. Normal b-cells low IgA. 4. Normal b-cells, low IgG, low IgA and high IgM. 5. Normal B-cells, low- IgG.

What symptoms of Sickle cell disease/SCD are treated with blood transfusions?

1. acute stroke. 2. acute chest syndrome. 3. multiorgan failure. 4. acute symptomatic anemia. 5. aplastic crisis.

Congenital rubella syndrome, define it.

1. cataracts. 2. PDA. 3. Sensorineural hearing loss.

Congenital toxoplasmosis what is the triad?

1. chorioretinitis. 2. hydrocephalus. 3. intracranial calcifications.

Patau's syndrome (trisommy 13), what are the findings?

1. cleft lip. 2. flexed fingers with polydactyly 3. ocular hypotelorism. 4. bulbous nose. 5. low-set malformed ears.

Causes of pediatric stridor include what (name 4)

1. croup. 2. foreign body aspiration. 3. laryngomalacia. 4. vascular rings.

What are the risk factors for otitis mdia?

1. day care attendance 2. sibling with AOM 3. parental smoking 4. drinking from a bottle while laying flat

XYY males

1. explosive (often antisocial) behavior 2. weakness with poor fine motor control 3. accelerated growth in mid-childhood 4. large teeth 5. severe acne at puberty

What is the pathogenesis for the following diseases with regards to carpal tunnel syndrome? 1. pregnancy. 2. hypothyroidism. 3. ESRD, Chronic hemodialysis, amyloidosis. 4. Acromegaly. 5. rheumatoid arthritis.

1. fluid accumulation. 2. matrix substances. 3. amyloid fibral deposition. 4. synovial tendon hyperplasia and edema. 5. tenosynovial inflammation.

What are the clinical features of rabies?

1. hydrophobia. 2. aerophobia. 3. pharyngeal spasm, spastic paralysis. 4. agitation. 5. ascending flaccid paralysis.

What medicines are used in the treatment of gestational diabetes.

1. insulin. 2. glyburide. 3. metformin.

Other than the cat-like cry what else is associated with Cri-du-chat syndrome?

1. microcephaly with protruding metopic suture 2. short stature. 3. moonlike face. 4. hypertelorism. 5. bilateral epicanthal folds. 6. high arched palate. 7. intellectual disability.

Define the findings of Edward's syndrome (trisommy 18).

1. microcephaly. 2. prominent occiput. 3. micrognathia. 4. rocker-bottom feet. 5. closed fist with index finger overlapping the 3rd digit and the 5th digit overlapping the 4th

Why do newborns need vitamin K?

1. poor placental transfer. 2. absent gut flora. 3. immature liver function. 4. inadequate levels in breast milk.

What is the leading risk factor for intraventricular hemorrhage in newborn?

1. premature. 2. low-birth-weight infants.

Most common causes of meningitis in children? (name 2)

1. viral->enterovirus 2. bacterial->Streptococcus pneumoniae or pneumococcus and Neisseria meningitidis or meningococcus.

Criteria for Failure to Thrive

1. weight falls below the 3rd percentile 2. weight for height/length falls below the 3rd percentile 3. rate of weight gain slows compared with previous growth, crossing two or more major percentiles on the growth chart in a downward direction.

What is the normal age of onset of pityriasis rosea?

10 - 43 years

Management of Acute Rheumatic Fever

10 days of penicillin to prevent ARF after pharyngitis NOTE: pts with ARF even without pharyngitis should be treated with long-acting IM benzathine penicillin G until adulthood

Fines motors: Bangs two objects together, Places pellet in bottle

12-14 months

Gross motors: Stands alone for 2 sec,

12-14 months

Language: Uses one or two words with meaning

12-14 months

Personal/Social: Hugs dolls or stuff animal, Routinely gestures to meet needs, exhibits separation anxiety, Follow 1 step commands

12-14 months

A burn that extends through the entire dermis into underlying fat is a burn of what degree?

3

McCune Albright Syndrome

3 P's + Precocious puberty + cafe au lait spots (Pigmentation) - irregular borders + multiple bone defects (Polyostotic fibrous dysplasia)

presentation of cyclical vomiting syndrome?

3 episodes of vomiting in a 6 month period. Easily recognizable to family. Lasts 1-10 days. vomiting > 4 times/hr. No symptoms between vomiting episodes and no underlying condition.

In most cases, cryptochidism usu. descends by what age?

3 mos

How many shots are in the HepB vaccine series? When is the first 1 given.

3, first at birth

How long do the fever, NV, crampy abdominal pain and bloody diarrhea last in gastroenteritis?

3-5days

What is the median survival rate for CF?

31

Fines motors: Copies circle

36 months

Gross motors: Stands on one foot at least 2 sec, rides tricycle

36 months

Language: Uses pronouns (I, me, you), Gives name

36 months

Personal/Social: Takes turns, Toilet trained

36 months

When does a child usu. develop day time bladder control?

4

McCune-Albright Syndrome

4 P's: Precocious puberty, Pigmentation,Polyostosis

double or triple newborn birth weight by when?

4 months/1 year

Laryngomalacia, what is the presentation for this disease. Diagnostic test? Treatment?

4 to 8 month old who has inspiratory stridor that worsens in the supine position. Laryngoscopy, showing collapse of supraglottic structures during inspiration and omega-shaped epiglottis Assurance as this resolves by age 18 months or supraglottoplasty if necessary due to respiratory compromise.

What age is the maxillary sinus aerated? What age is the front sinus aerated? What age is the ethmoid sinus aerated?

4 years old, 7 years old, birth

Age group and sex for Legg-Calve-Perthes disease?

4-10 years in boys Legg-Calvé-Perthes disease (LCPD) is avascular necrosis of the proximal femoral head resulting from compromise of the tenuous blood supply to this area. LCPD usually occurs in children aged 4-10 years. The disease has an insidious onset and may occur after an injury to the hip. In the vast majority of instances, the disorder is unilateral.

What ages typically do absent seizures typically present? Treatment?

4-10 years of age. Ethosuximide.

Fine motor: hands together, grasps objects

4-5 months

Gross Motor: Head steady in supported position, rolls front to back, back to front (4 months)

4-5 months

Short stature is defined as less than __th percentile on growth chart

5th

On CT, appendix diameter > __ mm is diagnostic for appendicitis.

6

At what age after birth do babies with thalassemia present with hemolytic anemia?

6 months (adult hemoglobin has now replaced fetal).

Hepatitis A is a 2 dose series starting 12 months. How many months should you separate the vaccine by?

6-18 months

Fine motors:Reaches for objects, Raking grasp

6-8 months

Gross Motors: rolls over, sits unassisted

6-8 months

Language: Vocal imitation

6-8 months

Personal/Social: Responds to own name, Holds own bottle, Stranger anxiety

6-8 months

What percentage of newborns experience jaundice within the first week of life?

60%

In HCM: Echocardiogram will show septal wall thickness and ejection fraction > __%. EKG reveals____.

60, LVH

What is the frequency criteria for the minimum number of pharyngitis episodes to have an indication for a tonsillecteomy? Past year ___, past 2 years ____, past 3 years ____,

7,5,3

Fanconi anemia presentation?

8 year old with progressive pancytopenia and macrocytosis with cafe-au-lait spots, microcephaly, microphthalmia, short stature, horseshoe kidneys and absent thumbs. Autosomal recessive disorder.

By this age, Barlow and Ortolani are not useful.

8-12 wks

Leukemia: What is the cure rate for ALL?

80%

Fines motors: Passes object from hand to hand

9-11 months

Gross motors: Stands while holding on, crawls,

9-11 months

Language:Understands no, May say mama

9-11 months

Personal/Social: Feeds self, Imitates waving

9-11 months

What are the likely causative agents in bacterial conjunctivitis?

: *Staph aureus*, Strep pneumo, *Haemophilus influenza*, and Moraxella catarrhalis

What is the treatment for hepatitis A?

: 1. Hep A Vaccine, 2. Postexposure prophylaxis with IG and household intimate contacts 3. HAV vaccine and immunoglobulin can be use concurrently

What is the treatment for hyperthyroidism?

: 1. Propanolol until resolved, 2. Thiourea drugs, methimazole and propylthiouracil, 3. Radioactive iodine (avoid in pregnancy), 4. Thyroid surgery for pregnancy

What will abdominal films and ultrasounds show in intussuception/

: Abd. film shows lack of bowel gas, US = single or hypoechoic ring with hyperechoic center (target/donut sign)

What is the treatment for a mobilliform drug rxn that is maculopapular but often spares the face. It is usu. due to ampicillin, isoniazid, phenytoin and thiaizides and occurs 7-10d after starting drug.

: Antihistamine and cooling lotions

What is the study of choice to diagnose oral candidiasis?

: Budding yeast and hyphae can be seen on KOH PREP or gram stain of scrapings

How do pts with SCFE present?

: Cause is unknown (trauma is not a cause). Patients note pain referred to the thigh and medial side of the knee and a limp. Any child with knee pain should have hip pathology ruled out.

How are pinworms diagnosed?

: Cellophane tape test. Diagnosis made by noting ovoid eggs under the microscope.

How do you diagnose tinea?

: Clinical presentation, KOH prep, Fungal culture

This treatment is effective in treating PDA by decreasing prostaglandin E levels

: Indomethacin effective in closing PDA (decreases prostaglandin E levels). Surgical litigation may be needed.

Child doubles the height at the age of __ years a. 5 1/2 b. 4 1/2 c. 3 1/2 d. 2

Answer: (b) 4 1/2

What is the treatment of scoliosis depending on degree of curvature?

<25 degrees = observation, > 45 = surgical intervention. 25-45 degrees = bracing. o MRI for onset before 8y/o

What is a normal QT interval?

<440 ms in males and < 460 in females.

In non-inflam acne with comedones, - use agents that cause mild drying and peeling such as ____ , ___

= benozyl eroxide, retinoids

What is large for gestational age?

> 4 kg

Rinne • With sensorinueral hearing loss AC ___ BC • With conductive hearing loss AC ___ BC great or less?

>, <

Indications for tonsilectomy?

>5 episodes of strep/year for 2 years OR >3 episodes/year for 3 years

Neurofibromatosis

>6 Café au lait, 2+ neurofibromas, axillary freckling, optic glioma, Lisch nodules, osseous lesions

Which strand of influenza causes the epidemic?

A

Acute febrile convulsion in children is treated with a. Ethosuximide b. Diazepam c. Phenobarbitone d. Phenytoin

Answer: (b) Diazepam

Most common histological type of tumor in both the supratentorial and infratentorial portions of the brain in kids?

Astrocytoma.

Presentation of Bowen's disease?

A squamous cell carcinoma in situ that presents as a erythematous plaque with well-defined irregular borders and an overlying scale or crust.

This is the only form of hepatitis under the DNA genome.

Hep B

Which of the following is the DOC in status epilepticus in children? a. Carbamazepine b. Diazepam c. Phenobarbitone d. Phenytoin

Answer: (b) Diazepam

What will you see on ABG, PFT and CXR in CF?

ABG = hypoxemia and compensated RA. PFT = mixed obstruction and restriction. CXR = hyperinflation, peribronchial cuffing, mucous plugging, bronchiectasis, increased interstitial markings, small round peripheral opacities, focal atelectasis or pneumothorax.

A lactose tolerance test tests for absorption or malabsorption? A lactose breath hydrogren test test for absorption or malabsorption?

ABSORPTION, MALABSORPTION

Patient presents with acute testicular pain and swelling of the scrotum with tenderness. *Cremasteric relex is absent*. What is the likely diagnosis?

ACUTE testicular torsion

Osteogenesis Imperfecta

AD connective tissue collagen disorder, varying spectrum of severity + multiple fractures, hearing loss + easily bruised, hypotonia + blue sclerae + dentinogenesis imperfecta

What is the autosomal dominant cause of QT prolongation? How about autosomal recessive?

AD: Romano-ward syndrome. AR: Jervell-Lange-Nielson syndrome.

Immature lymphoblasts are produced with no further differentiation in this form of leukemia.

ALL

What is the most common pediatric malignancy?

ALL Acute Lymphobastic Leukemia

In a patient with chronic nose bleeds, which blood d/os will you consider?

ALL, vWB

If you see auer rods on peripheral blood smear which type of leukemia does the pt likely have?

AML

All are false regarding simple febrile convulsions except: a. No spontaneous remission b. Follow high fever c. Recurrent in nature d. Occurs at 6 years onwards

Answer: (b) Follow high fever

Which of the following is true regarding juvenile CML a. Philadelphia chromosome b. High fetal hemoglobin content c. Low acid phosphatase d. Massive splenomegaly

Answer: (b) High fetal hemoglobin

Bull neck in diphtheria occurs due to a. Cellulitis b. Lymphadenopathy c. Retropharyngeal abscess d. Laryngeal edema

Answer: (b) Lymphadenopathy

Toxic shock syndrome may occur due to a. Hepatitis B vaccine b. Measles vaccine c. DPT vaccine d. BCG vaccine

Answer: (b) Measles vaccine

Deficiency of which of the following leads to pellagra a. Pyridoxine b. Nicotinamaide c. Riboflavin d. Thiamine

Answer: (b) Nicotinamide

POD 1-2, Acute

Atelectasis *most common*, early wound infection, aspiration pneumonitis, Addisonian crisis, thyroid storm, transfusion reaction

A child presents with change in color of hair, blue eyes and mental retardation. He is suffering from a. Cystinuria b. PKU c. Malonyl urea d. Homocystinuria

Answer: (b) PKU

Carbamazepine is the drug of choice in which of the following conditions a. Absence attacks b. Partial complex seizure c. Infantile spasm d. Post-anoxic myoclonus

Answer: (b) Partial complex seizures

Not a feature of juvenile CML a. lymphadenopathy b. Philadelphia chromosome positive c. Fetal hemoglobin is increased d. Thrombocytopenia

Answer: (b) Philadelphia chromosome positive

Which of the following inborn error of metabolism is associated with mental retardation? a. PKU b. Duchenne's syndrome c. Down syndrome d. Mcardle syndrome

Answer: (a) PKU

Kartagener's Syndrome

AR, due to immotile cilia (primary ciliary dyskinesia) Triad: + situs inversus + recurrent sinusitis + bronchietasis

2 month old patient presents with a history of slow weight gain and recurrent LRT . On physical exam right ventricular heave, wide and constantly split S2 and a systolic ejection murmur in the pulmonic area and a mid-diastolic rumble in the lower right sternal border are noted. What is the likely dx?

ASD

widely split, fixed S2 in a child 3-5 years of age; systolic murmur due to increased flow across a normal pulmonic valve

ASD

Cardiac exam for Complete AV septal defect VS. Tetralogy of Fallot

AV Septal Defect (ASD + VSD) + loud S2 due to pulmonary HTN + systolic ejection murmur from increased flow across the pulmonary valve from the L => R flow across the ASD + holysystolic murmur of VSD that can be soft/absent if large @ left lower sternal border Tetralogy of Fallot + harsh cresendo-descrendo murmur due to RVOT obstruction + single S2

Management of suspected biliary atresia/choledochal cyst with hepatomegaly and direct hyperbilly?

Abd U/S, radionucletide imaging, and liver biopsy

Wilms Tumor

Abdominal mass, fever, aniridia, kidney tumor, caused by deletion on chromosome 11

Henoch-Schönlein Purpura

Abdominal pain, blood in the stool, lower extremity rash, hematuria, joint pain, intussusception, preceded by URTI

Why is there growth failure with renal tubular acidosis?

Acidic conditions is a poor growth medium.

What is the difference b/t acute and chronic urticaria/hives?

Acute - lasts < 6wks. More common to atopic individual. Due to histamine and IgE. Chronic - > 6wks. Patients should be evaluation for the 5 I's: Ingestants (foods, additives and antibx), Inhalants (dust, pollen), Injectants (drugs, stings, bites), Infections (bacterial, fungal and parasitic), Internal dz (chronic infections, SLE, thyroid dz)

Patient presents with Diffuse wheezing, variable, fever, cough, tachypnea, rhinorrhea, difficulty feeding and cyanosis. PE=intercostal retractions, hyperinflation, crackles, nasal flaring, prolonged expiration and wheezing. HAPPY WHEEZER. What is the likely dx?

Acute bronchiolitis

"Swimmer's ear" is what category of OE? Acute localized, acute diffuse, chronic or malignant?

Acute diffuse

What are acute treatment options in tetralogy of fallot?

Acute treatment options = vagal maneuvers, BBs, oxygen, vasoconstrictors, morphine and fluids

Herpes conjunctivitis should be treated with ________ to prevent corneal involvement.

Acyclovir

What is the best screening test for scoliosis? What is used to confirm it? How do you measure the degree of curvature?

Adam's forward bend test. AP Spinal XR. Degree of curvature measured by Cobb method

What is the treatment for hyaline membrane disease?

Adequate resuscitation and respiratory support should begin immediately with assisted ventilation or continuous positive expiratory pressure. Restrict fluids to avoid pulmonary edema 80-90% of patients survive w/ normal lungs by 1 mo of age.

1 year old child has meningococcemia. In the ED she becomes hypotensive and dies despite aggressive fluid and antibiotic resuscitation.

Adrenal gland failure due to adrenal hemorrhage. This is known as Waterhouse-Friderichsen syndrome.

How do male pinworms affect the body? How do female pinworms affect the body?

Adult worms take up residence in cecum, Female migrate to perianal region at night where they lay eggs. Eggs are infectious for up to 20 days.

Pts with acute rheumatic fever ought to be treated with benzathine penicillin G until when? How about those with concurrent chorea or pericarditis or arthritis?

Adulthood. Corticosteroids for chorea. NSAIDS for pericarditis and joints

What is the treatment for toxic epidermal necrosis?

Aggressive fluid & electrolyte management, wound care, pain control

Recent introduction of pureed fruits has led the child to vomiting, poor feeding and lethargy and recent episodes of seizure and encephalopathy. Diagnosis?

Aldolase B deficiency causes hereditary fructose intolerance. (fructose-1-phosphate accumulates)

All individuals born between which years should be screened for hepatitis C?

All individuals born b/t 1945-1965 should be screened for HepC

All children should take vitamin D supplements. What is the dosin?

All infants: 400 IU daily beginning a few days after birth Children 1-18 years of age: 600 IU daily.

This medicaiton in GERD acts as a mechanical barrier and buffers HCl. The class is "barrier", what is the drug?

Aluminum Hydroxide

Leukocoria in any child, what do you do?

Always refer to opthalmologist.

How does respiratory distress syndrome present in a newborn?

Always suspect a premature infant who has grunting, flaring and retractions immediately after birth.

Influenza: Tricyclic Amines are Effective in preventing influenza A in institutional settings. CNS SE: insomnia, anxiety, confusion and rarely, seizures. What are examples?

Amantadine, Ramantadine

Emperic therapy for Pseudomonas, esp with CF in older kids and adults?

Amikacin, ceftazidime, and ciprofloxacin

What is the 1st line oral agent for patients > 3 months, < 5 years with suspected bacterial pneumonia?

Amoxicillin (preferred) then augmentin if atypical, azithromycin, clarithromycin, erythromycin If influenza suspected, tamiflu

Children less than 8 years old and have lyme disease need what antibiotic for treatment and why?

Amoxicillin. Doxycycline causes enamel hypoplasia and permanent teeth discoloration.

What is the treatment for typical bacterial pneumonia outpt in pts 5 to 18 years?

Amoxicllin preferred, then augmentin. If atypical try azithromycin, clarithromycin, ertyrhomycin and doxycycline.

All newborns with pneumonia should be admitted and on a regimen of 2-3 antibx. What is the recommended regiment?

Ampicillin, Gentamicin, Cefotaxime

Croup is not associated with a. Polio virus b. Adenovirus c. Influenza virus d. RSV

Answer: (a) Polio virus

In india which of the following is not an important cause of post-neonatal mortality a. Tetanus b. Diarrhea c. Malnutrition d. Respiratory infection

Answer: (a) Tetanus

When should a pediatrician be concerned for biliary atresia?

An initially well-appearing child who develops the following symptoms in sometime within 1-8 weeks after birth: 1. jaundice. 2. hepatomegaly. 3. acholic (pale) stools or dark urine. 4. Conjugated hyperbilirubinemia with mild elevation in transaminases.

Radial head subluxation (Nursemaid's elbow), presentation?

An injury due to the pulling of a child by hand or arm which causes axial traction of the forearm which leads to the name of this condition. Child keeps arm pronated and cannot supinate as there is resistance and pain with the movement.

Regarding DTaP, what are the contraindications?

Anaphylaxis. Unstable neurologic d/o (Pertussis component) Encephalopathy within a week of previous vaccine dose (Pertussis)

Describe the etiology of androgenetic alopecia.

Androgen exposure shortens Hair follicle anagen phase. Men with increased androgen levels, women with increased 5a-reductase, androgen receptors. Affected hair becomes shorter, finer, less pigmented. Progresses with each successive cycle. Related to higher DHT levels.

Patient presents with hair loss onset between 12-40 years. Hair loss evident by age 50 in >50% of patients. What is the likely diagnosis?

Androgenetic alopecia

This is the most common type of Alopecia. Hereditary trait.

Androgenetic alopecia

Pathophysiology of hereditary spherocytosis?

Ankyrin gene mutation with resulting spectrin deficiency.

This ligament slips proximally and becomes trapped b/t the radius and ulna in subluxation of radial head.

Annular

New born with stridor and dysphagia. NGT and OGT can be advanced into respective sites. Diagnosis?

Anomalous vascular branches of aortic arch.

Diphtheric neuropathy occurs in __week of infection a. 4th b. 3rd c. 2nd d. Ist

Answer: (a) 4th

Neonatal hydrocephalus is not seen in a. AIDS b. Brain tumor c. Aqueductal stenosis d. Intraventricular hemorrhage

Answer: (a) AIDS

Which of the following is true regarding Dawn phenomenon? a. Early morning hyperglycemia b. Early morning hypoglycemia c. Hypoglycemia is followed by hyperglycemia d. Insulin dose is decreased in treatment

Answer: (a) Early morning hyperglycemia

In which of the following conditions transient rash is seen a. Chickenpox b. Exanthem subitum c. Scarlet fever d. Eczema herpeticum

Answer: (a) Exanthem subitum

Which of the following is not caused by thiamine deficiency in children a. Infantile tremor syndrome b. Wernicke's encephalopathy c. Korsakoff's psychosis d. Congestive heart failure

Answer: (a) Infantile tremor syndrome

Which of the following is true regarding febrile convulsions? a. It is a benign condition b. If first episode is not treated then recurrence occurs c. Phenobarbitone is given for 2 years d. Focal neurological deficit

Answer: (a) It is a benign condition.

Which of the following is the most common cause of community acquired neonatal meningitis? a. Klebseilla pneumoniae b. Neisseria c. Entamoeba coli d. Pseudomonas

Answer: (a) Klebsiella pneumoniae

Renal biopsy was done of a child less than 6 years of age having nephrotic syndrome, which is responsive to steroids. Under light microscopy what will be seen a. Nothing b. Loss of foot process c. Tubule atrophy d. Crescents

Answer: (a) Nothing

Coarctation of aorta is common in a. Noonan's syndrome b. Down syndrome c. Klinefelter syndrome d. Turner's syndrome

Answer: (a) Turner's syndrome

What are some common allergy causing drugs in peds?

Anticonvulsants, Insulin (esp animal sources of insulin), Iodinated (containing iodine) xray contrast dyes (these can cause allergy-like reactions), Penicillin and related antibiotics, Sulfa drugs

What is the treatment for lichen planus?

Antihistamines for pruritus. Topical steroids are initial tx for localized dz. Intralesional triamcinolone acetonide is used for hypertrophic lesions. Oral steroids for generalized skin involvement. Severe cases may require systemic retinoids or cyclosporine.

What is the treatment for urticaria (hives)?

Antihistamines used initially. Histamine-1 blockers such as hydroxyzine work best. Nonsedating histamine-1 blockers such as loratidine and certrizine don't work as well. Prednisone is used in cases not controlled by antihistamines. May not be helpful in chronic urticaria. Epinephrine is used in EXTENSIVE, SEVERE cases.

Fetal hemoglobin is increased in which of the following a. Hereditary spherocytosis b. Pure red cell aplasia c. Polycythemia vera d. Juvenile CML

Anwer: (d) Juvenile CML

Why is there a single S2 in neonates with transposition of the great vessels?

Aorta is dislocated anteriorly.

systolic ejection murmur followed by an early diastolic murmur

Aortic stenosis and aortic insufficiency

Fanconi Anemia

Aplastic macrocytic anemia, pancytopenia, short stature, strabismus, pigmentation abnormalities

Staphylococcal conjunctivitis

Appears several weeks after birth

Your patient presents with RLQ tenderness, fever, N/V that has now persisted over 36 hours, a palpable abdominal mass and marked leukocytosis. What should you suspect?

Appendicitis with perforation

What arteries supply the femoral head?

Ascending arteries and the foveal artery which lie within the ligamentum teres.

14 year old black child with pain in right hip. What are the serious top two differentials on your mind?

Aseptic necrosis from sickle cell. Slipped capital femoral epiphysis.

For children older than 5 with bedwetting symptoms, why is a U/A done?

Assess for UTI, DM and diabetes insipidus.

What is the most common chronic disease of childhood?

Asthma

Cutaneous and visceral angioma-like blood vessels in an AIDS patient.

Bacillary angiomatosis due to Bartonella.

This test in intussuception is both diagnostic and therapeutic.

Barium Enema (will reveal intussuception as an inverted cap and possible obstruction)

String sign on Barium GI is seen in pyloric stenosis. Shoulder sign is also seen. Describe the signs.

Barium UGI: String sign (elongated pyloric channel), shoulder sign = bulge of pyloric muscles into antrum

Presentation of vascular rings? How does one go about diagnosis?

Biphasic stridor and feeding difficulties. Barium swallow followed by MRI with angiography.

Performed with the infant fully relaxed; it attempts to dislocate the hip via posterior pressure. The examiner adducts the fully flexed hips while pushing the thighs posteriorly. If during this maneuver the femoral head is felt to dislocate or leave the acetabulum, it is considered a positive. Which test is this?

Barlow

What views are essential in dx'ing SCFE?

Based on clinical findings and XR. (views = AP, Launstein [frog] lateral views that show epiphyseal displacement)

Non Hodgkins lymphoma is also called what kind of lymphoma?

Burkitt Lymphoma

Patient with a vitamin deficiency presents with symmetric, hyperpigmented rash, similar to a sunburn. Red tongue. Other nonspecific symptoms-diarrhea, vomiting, insomnia, anxiety, disorientation, delusions, dementia, encephalopathy. What vitamin are they deficient in?

B3 (Niacin) deficiency causes Pellagra

What anti-HTN medicines are used for treating Hypertrophic obstructive cardiomyopathy/ HOCM?

BB and CCBs.

What is the treatment for HCM if symptomatic?

BB's (propranolol) and CCB (verapamil

What lifestyle modifications can children adopt in GERD?

Babies: sit up, eat small amounts, more frequently. Kids- Lose weight, remain upright after meals, raise head of bed 6-8 inches with wood blocks under bedposts, avoiding smoking or being around people who smoke

Fetal macrosomia, rapid growth until late childhood; omphalocele or umbilical hernia macroglossia hemihyperplasia fetal hyperinsulinemia => hypoglycemic @ birth

Beckwith-Wiedemann Syndrome PATH: dereglation of imprinted gene expression in ch 11p15 Complications: Wilms tumor, Hepatoblastoma Surveillance with: serum AFP, ABD/renal US

At what age is bed wetting normal?

Before 5.

When is bedwetting normal?

Before the age of 5

What can you use to as prophylaxis for rheumatic fever?

Benzathine Pencillin q 2 weeks

TX for symptomatic patients with prolongation of QT interval? (these patients are at risk for synpcope, ventricular arrhythmias, and SCD)

Beta-blockers such as propranolol Pacemaker if congenital QT syndrome and symptomatic Maintain normal levels of Ca, K, and Mg and avoid meds that block potassium channels (Class IA and III) NOTE: AR, sensorineural deafness, QT prolongation = Jervell-Lange-Neilson syndrome

Tay-Sach's Didsease

Beta-hexosaminidase A deficiency, loss of motor milestones, hypotonia, hyperreflexia, red macula

How do migraines present differently in children vs adults?

Bifrontal and shorter duration.

How do you dagnose hypospodias (abnormal placement of the urethra where the meatus is proxmal and ventral to its normal anterior location)?

Bilateral renal US is warranted to rule out ascending pathology.

What is the 5th most common cause of accidental death in children?

Burns

Dilated loops of bowel in an infant, how do you work this up?

Bowel = large intestine issue. Do a contrast enema to see for micro colon (meconium ileus) or rectosigmoid transition zone (Hirschsprung).

What are the clinical manifestations of congenital hypothyroidism?

Bradycardia, dry coarse hair, poor muscle tone, constipation, pale cool skin and intellectual retardation (permanently lowers IQ by 10-15pts)

How is breast milk jaundice distinguished from breast feeding failure jaundice? When do both of these pathologies occur?

Breast feeding failure is associated with weight loss, brick-red urate crystals in diaper and decreased urine output. Breastfeeding failure: 1st week of life. Breast milk: Day 3 or 4 to 2 weeks. Note: make sure to compare changes in weight on the history if values are given.

What is the timeline of nutrition for children?

Breastfeeding till 6 months, transition to pureed foods, and introduction to cow's milk at age 1.

What is the treatment for acute bronchiolitis? What is the treatment for high risk patients with acute bronchiolitis?

Bronchodilator for wheezing. Antiviral, ribavirin given by aerosolization. Some patients may develop wheezing later in life. High risk patients can be prophylactically treated with palivizumab (a monoclonal antibody and RSV F-protein blocker)on a monthly basis.

Neonatal Listeriosis

Brown amniotic fluid, granulomatosis infantiseptica, exposure to unpasteurized dairy products

Persistent Pulm HTN of newborn

By definition, excludes presence of congenital heart disease. Perinatal asphyxia and meconium aspiration syndrome are common causes in term infants, resulting in increased pulm vascular resistance and significant R => L shunting and significant resp distress TX: O2 is the most potent vasodilator

Ascorbic Acid is also known as Vitamin __

C

Scratching in pinworms can cause excoriations and secondary infections such as...?

Impetigo

T or F. Pyloric stenosis (caused by hyertrophic pyloric spinchter) is more common in 1st born males, with a 4:1 M/F predominance.

T

What is the treatment for osgood-schlatter's?

Casting may be required in severe cases. Avoid steroid injections (weakens patellar ligament, thins and depigments skin) Analgesics as needed.

This is the test of choice in orbital cellulitis to determine the extent of the disease. It will show broad infiltration of the orbital soft tissue.

CT scan you can also use MRI but CT is better

Management of Mastoiditis

CT scan to confirm TX includes myringotomy, fluid culture, and IV antibiotics

How is the diagnosis of HSV made?

Can be made by cell culture, tzanck smear, antigen detection and PCR.

How long do the lesions of a herpes simplex outbreak last?

Can involve any mucosal surface. Lesions are very painful and last 5-10d. May become latent w/ in sensory nerve root ganglion. Recurrences are typically unilateral and last 7d.

What is the common causative "bug" in diaper dermatitis?

Candidiasis

What is the treatment for tinea capitis? What is the treatment for tinea corporis, cruris and pedis?

Capitis: Griseofulvin. Corporis, Cruris, Pedis: topical Lotrimin (clotimazole), Spectrazole (econazole), Lamisil (terbinafine)

Caput succedaneum or cephalhematoma is associated with vacuum assisted delivery?

Caput succedaneum.

Which cause of swelling in the scalp of a newborn is limited to the surface of one cranial bone and is not associated with discoloration and presents several hours after birth.

Cephalohematoma Note: subperiosteal bleeding is a slow process hence the presentation.

What is considered a low-phenyalanin diet?

Cereal, starches, fruits, vegetables, and phenylalanine free milk formulas. Avoid high protein foods (phenylalanine is a protein building block)

Mild coagulopathy, peripheral and cranial neuropathy, pancytopenia and partial oculocutaneous albinism, frequent bacterial infections and progressive lymphoproliferative syndrome. Diagnosis?

Chediak-Higashi Syndrome

Which hemangiomas are found in adults?

Cherry.

If a foreign body is suspected in a 1-3 year old what do you do?

Chest x-ray or Immediate bronchoscopy.

Rickets is a clinical manifestation of Vitamin D deficiency. How does it present?

Child can by asymptomatic or have varying degrees of pain and irritability, motor delays, poor growth, and increased susceptibility to infections. Younger children may manifest with delayed closure of fontanelles, craniotabes, frontal bossing, prominence of costochondral junctions, widening of wrists and ankles, and bow legs or knock knees.

Diamond-Blackfan syndrome (congenital hypoplastic anemia), what is the presentation of this disease?

Child with macrocytic anemia, low reticulocyte count and congenital anomalies such as short stature, webbed neck, cleft lip, shielded chest and triphalangeal thumbs.

What is the WHO respiratory rate thresholds for identifying children with pneumonia?

Children < 2months: Greater than or equal to 60 breaths/min Children 2-11 months: Greater than or equal to 50 breaths/min Children 12-59 months: greater than or equal to 40 breaths/min

CHARGE syndrome:

Coloboma Heart defects Atresia of choanae Renal anomalies Growth impairment Ear abnormality/deafness

CHARGE Syndrome

Colobomas, heart deffects, choanal atresia, retardation, genitourinary defects, ear abnormalities

This type of hydrocele may develop into an inguinal hernia?

Communicating

Newborn presents with bloated, puffy eyelids, dry skin, little perspiration, muscular weakness, constipation, somnolence, sluggish mental and physical processes, slow reflexes, decreased heart rate, edema, depression. What is the likely diagnose?

Congenital Hypothyroidism

Hirchsprung disease is due to a congenital absence of what two plexuses that innervate the bowel wall?

Congenital absence of Meissner and Aurebach autonomic plexuses enervating the bowel wall.

Equinovarus

Congenital club foot, stretching, traction, and serial casting

Large tongue, sluggish movements, abdominal bloating, umbilical hernia, apathy and weakness. Diagnosis?

Congenital hypothyroidism.

Hirschprung Disease

Congential aganglionic bowel, bilious or feculent vomitus, failure to pass meconium, megacolon, absence of submucosal and mytenteric ganglia

Neonatal Chlamydia

Conjunctivitis and subsequent pneumonitis

CATCH-22 (DiGeorge Syndrome)

Conotruncal abnormalities (truncus arteriosus MC) Abnormal Facies Thymic aplasia Cleft palate Hypocalcemia

Blunt abdominal trauma in a young child with an isolated duodenal hematoma obstructing the bowel. Treatment?

Conservatively with NGT and parenteral nutrition.

Selective Mutism

Consistent failure to speak in specific situations for at least one month

Patient presents with sharply marginated defined to area of contact with erythema-vesicle-erosion-crust-scale and evolution rapid within few hours or chronic form-months. What is the likely diagnosis?

Contact dermatitis

Why is bicarbonate elevated with patients who vomit?

Contraction alkalosis. Aldosterone responds to dehydration and hypovolemia

Barium enema is both diagnostic and therapeutic in intussuception. It will reveal it as an inverted cap with possible obstruction. When is it contraindicated?

Contraindicated with free air in abdomen.

Dilated bowel loops are confirmed on KUB of a child with delayed passage of meconium. Next best step?

Contrast enema: will show microcolon to diagnose meconium ileus or rectosigmoid transition zone for Hirschsprung.

What is the treatment for glomerulonephritis..a condition that presents with edema in periorbital and scrotal regions along with HTN?

Correction of HTN and fluid overload. Steroids and immunosuppressive drugs may be used to control the inflammatory response, which is responsible for the damage. Diet mgmt.: Salt and fluid intake should be decreased. Dialysis should be performed if symptomatic azotemia is present. Medical Thereapy: Consider ACEI

_________ are contraindicated in the treatment of kawasaki disease.

Corticosteroids

What is the MC symptom in the infant presenting with pneumonia?

Cough

What needs to be done if a parent decides not to have his or her child treatment for a fatal condition in a non-emergent setting?

Court order.

This special test in strabismus determines the angle and reveals latent strabismus and heterophoria.

Cover/uncover test

5 year old boy with gray vesicles and ulcers at the posterior oropharynx. Etiology?

Coxsackie.

Describe seborrheic dermatitis in terms of presentation.

Cradle cap, adherent greasy scales with a mild erythematous base on the scalp.

How would you describe the murmur in tetralogy of fallot?

Crescendo-decrescendo holosystolic at LSB, radiating to back

Crohns vs. UC?

Crohns - diarrhea, abd craming, loss of appetite; perianal skin disease, such as skin tags, fissures, and fistulas may precede intestinal symptoms; poor growth and delayed puberty UC - rectal bleeding, diarrhea, abd pain, ANCA

Barky cough. Diagnosis? Best means of identifying this disease?

Croup or laryngotracheitis. Lateral neck x-ray to find subglottic narrowing.

Presentation of choanal atresia?

Cyanosis that is aggravated be feeding and relieved by crying.

What are the 2 types of breath holding spells?

Cyanotic - crying and breath-holding in forced expiration. Pallid - minor trauma causes child to lose consciousness followed by breath-holding (lasts < 1 minute).

Recurrent predictable patter of acute and frequent vomiting that resolves spontaneously with no symptoms between episodes. What disease is associated wit this?

Cyclic vomiting syndrome. Migraines

Newborn infant presents with persistent pulmonary infections, meconium ileus and failure to thrive. What is the likely dx?

Cystic Fibrosis

o Autosomal recessive d/o, most common inherited disease in American whites. 1/2000 births

Cystic Fibrosis

If a child presents with recurrent UTIs and you suspect a vesicoureteral reflux issue. What test should you order?

Cystourethrogram.

Bleeding is the most common clinical finding (from skin and mucous membranes). May develop small purpuric to large ecchymoses. Thrombotic events include gangrenous digitis and nose and hemorrhagic necrosis of skin. This sounds like the signs of symptoms of what bleeding d/o?

DIC

One of the most common and potentially life threatening causes of non-immune peripheral platelet destruction Condition in which coagulation factors are activated and degraded simultaneously. *Patient will present w/ both bleeding and clotting.*

DIC

How is DIC different from microangiopathic hemolytic anemia?

DIC involves changes in PT and aPTT.

Chemical conjunctivitis

DOL 1, caused by silver nitrate

Gonococcal conjunctivitis

DOL 2-5, bilateral, purulent, lid swelling

Chlamydial conjunctivitis

DOL 5-14, watery mucoid discharge

What is the treatment for SJS including supportive treatment?

D/C offending drug immediately. Supportive tx: fluid and electrolyte balance, fluids, high caloric supplementation, prevention of sepsis. Steroids- controversial

what you do to a newborn right after birth

DWPSTS Dried Warmed Positioned Suctioned Tactile Stimulation give newborn 10ml/kg IV push over 5 minutes if blood pressure is really low at birth, and then check response. Continue giving fluids if not better up to 90 ml/kg. 2ml/kg D10W if newborn is hypoglycemic NOTE: IgG antibodies have a half life of 30days (1 month) if there is a hemolytic process

What is the treatment for verrucae plana/vulgaris?

Debriding or paring down callus, warm water soaks, occlusion with duct tape Salicylic acid OTC, Keratolytic agents- Cantharidin, Imiquimod Cream (Aldara), Retin A (Tretinoin) .05% cream Procedures: 1st line- cryotherapy, 2nd line- wart immune therapy

What is the treatment for intussuception?

Decompression of intestine, correct dehydration, surgery indicated if reduction not possible/successful . (50-90% success)

Common variable immunodeficiency syndrome

Decreased IgM, recurrent resp and GI infections, failure of lymphocyte differentiation

What symptoms are associated with congenital hypothyroidism?

Decreased activity, hoarse cry and jaundice.

Pathophysiology of breastfeeding failure?

Decreased bilirubin elimination and increased enterohepatic circulation all due to lactation failure resulting decreased stooling.

Pathogenesis of Chediak-Higashi syndrome? Lab findings associated with this?

Decreased degranulation, chemotaxis and granulopoiesis. Neutropenia and giant lysosomes in neutrophils

Legg-Calvé-Perthes Disease

Decreased vascular supply to femoral head causing avascular necrosis, pain on internal rotation, knee pain

Hydroxyurea for Sickle Cell

Decreases pain crises, need for transfusions, and episodes of acute chest syndrome by increasing fetal hemoglobin. SE: temporary and reversible bone marrow suppression NOTE: do exchange transfusion (and have hydroxyurea on board) as a primary treatment for stroke in a sickle cell patient (sickled RBCs cause sludging and occlusion of cerebral vessels)...it helps to decrease the percentage of sickle cells in the bloodstream, making additional strokes less likely.

Ataxia Telangectasia

Defficiency in IgA and IgE, choreoathetoid movements, slurred speech, opthalmoplegia, mental retardation

Leukocyte Adhesion Deficiency Type 1

Deficient CD18, delayed umbilical cord separation, skin and mucosal infections, leukocytosis with neutrophil predominance

Diagnostic workup of FTT

Definitely CBC, UA, BUN/Cr; sometimes also... 1. BUN/creat - poor growth can reflect renal failure 2. Chest X-ray - look for cardiomegaly and pulm infiltrates 3. CBC/smear - anemia/infection 4. Electrolytes - hydration status, renal function, acid/base status 5. Fecal fat analysis (72-hour stool collection) - identify fat malabsorption 6. LFTs - might suggest congenital/acquired infections or conditions that cause biliary congestion, such as cystic fibrosis 7. Neonatal screen 8. Occult blood, stool/stool reducing substances - milk protein intolerance 9. Ova and parasities, stool 10. Stool culture 11. Sweat Chloride- best screening and diagnostic test for CF 12. Thyroid function tests 13. UA - assess renal tubular acidosis, hematuria, UTI 14. Urine culture - if UA is anormal or hx of fever

Why is an Upper gastrointestinal endoscopy tube done within 24 hours of caustic ingestion and not latter?

Delayed endoscopy is associated with increased perforation risk.

Cri-du-chat syndrome, what is the genetic abnormality?

Deletion of 5p

Wiskott-Aldrich syndrome, presentation? Inheritance?

Eczema, thrombocytopenia and hypogammaglobulinemia. X-linked.

What diagnostic studies are indicated in neoplasia of the MSK system? What is essential?

Diagnostic studies: Alkaline phosphatase and lactate dehydrogenase are elevated when the bone is broken down and remodeled. Serum and urine protein electrophoresis studies can detect the specific abnormal globulin of multiple myeloma. Biopsy is essential to diagnose whether benign or malignant, the cell type and grade of lesion. (open incisional bx is best and should be done by specialist)

Macrocytic pure red cell aplasia associated with short stature, webbed neck, cleft lip, shielded chest and triphalangeal thumbs. Diagnosis?

Diamond-blackflan syndrome (congenital hypoplastic anemia)

What other disease can present like croup?

Diphtheria.

This abdominal abnormality is acquired as a result of weakness of transversalis fascia in Hesselbach's triangle.

Direct inguinal hernia

How do you manage acne vulgaris/

Do not squeeze lesions: forces pus into dermis, causes inflammation and scarring. Limit washing face to 2-3 times per day. Avoid abrasive soaps, gear soap to skin condition. Skin dry: Purpose soap. Skin oily: Dial soap. Change cosmetics to water based products. Topical meds: Retin A 0.025% cream every other day, Benzoyl peroxide 2.5%

How do you diagnose testicular torsion?

Doppler or US

Duodenal atresia

Double bubble sign, associated with Down Syndrome

Duodenal atresia is a complete congenital obstruction of the lumen of a hollow viscus. 30% of patients with it have a chromosomal abnormality, primarily ____________

Down Syndrome

Hirschsprung disease is associated with what congenital disorder?

Down syndrome.

Simian creas and Brush field's spots

Down syndrome.

What are the complications of maternal hyperglycemia?

Due to fetal hyperinsulinemia... 1. Erythropoiesis (due to increased metabolic demand, see note). 2. Organomegaly. 3. macrosomia & shoulder dystocia. 4. Neontal hypoglycemia. Note: baby is in constant anabolism, sometimes mother will not be able to provide enough nutrients from the placenta to the child.

Complete congenital obstruction of the lumen of a hollow viscus. One of the most frequent causes of bowel obstruction in the newborn and can occur at any point in GI tract. Ileum is most commonly affected.

Duodenal Atresia

Double bubble sign.

Duodenal atresia.

Refeeding syndrome with anorexia? when to hospitalize? TX?

During the onset of anabolism, patients require close monitoring for electrolyte depletion, arrhythmias, and heart failure, which can result from fluid and electrolyte shifts. NOTE: hospitalization and acute stabilization of anorexia are high recommended for: - dehydration - electrolyte disturbances (low potassium and phosphate) - bradycardia - severe weight loss TX: olanzapine for patients who fail to gain weight despite initial nutritional rehabilitation and CBT

What virus might be linked to Lymphoma (hodgkins)?

EBV

Adam-Stokes Attack

ECG shows P waves without QRS complexes (asystole), AV block (syncope), v fib during attacks. Can present with seizures.

Toddler who fainted while running in the park. He had a 7 yo brother who died drowning in the pool. What is the best study to do?

EKG looking for QT prolongation.

Due to degrantulation of mast cells inflammatory response release of histamine, leukotrienes, cytokines and prostaglandins. Occurs w/ in 10 - 15 minutes of exposure. Symptoms = sneezing, rhinorrhea, itching and increased vascular permeability. Early or Late Allergic Rhinitis?

Early

How does biliary atresia present?

Early infancy with obstructive jaundice and acholic stools. Note: no one out of infancy has this disease without either receiving treatment or dying of this disease.

What form of herpes is associated with atopic dermatitis? What findings define this disease? Any key notes to keep in mind of with this disease?

Eczema herpeticum. Vesicles over the area of atopic dermatitis. With infants acyclovir needs to be started.

Causes for pathological gynecomastia?

Either: 1) increased estrogen production or peripheral conversion a. tumor b. cirrhosis or malnutrition c. thyrotoxicosis d. congenital excessive aromatase activity e. androgen use e. drugs (spirnolactone, cimetidine), herbal products (tea tree oil, lavender oil) 2) Androgen deficiency a. Klinefelter syndrome or testicular damage b. hyperprolactinemia c. renal failure

Nezelof Syndrome

Elevated immunoglobulins with poor function

What abnormal labs are associated with hereditary spherocytosis?

Elevated mean corpuscular hemoglobin concentration. Spherocytes. Osmotic fragility on acidified glycerol lysis test. Abnormal eosin-5-maleimide binding test.

What is the treatment for scabies?

Elimite (5% permethrin) Eurax (10% crotamiton) Kwell (1% lindane- avoid in infants)

Hx of constipation or painful defecation, inability to differentiate passing gas and passing feces in underwear, soiling episodes usually occuring during daytime are all symptoms of...

Encopresis

This is an involuntary discharge of feces in the newborn. In most cases, it is the consequence of chronic constipation and resulting overflow incontinence.

Encopresis

IgA Nephropathy

Episodic hematuria with interepisodic microhematuria

The clinical manifestations of this viral xanthem are asymptomatic or subclinical. Develops most often in children < 10. Starts w/ nonspecific prodrome, then a nonspecific febrile illness with HA, coryza and diarrhea. This is followed by a bright red "slapped-cheek" facial rash. Later a maculopapular rash appears on the trunk and extremities. Adults infected w/t he virus are more likely to present w/ arthritis.

Erythema Infectiosum (5th disease) which is caused by Human Parvovirus B19

Patient presents with targetoid "iris" lesions that are dusky red and round, maculopapules, typically on extremities, may have oral mucosal involvement. What is the likely diagnosis?

Erythema Multiforme

This skin disorder occurs in younger males, freq recurrences, less fever, milder mucosal lesions, no association with collagen vascular dz, HIV, or cancer (SJS, Erythema Multiforme, TEN)??

Erythema Multiforme

Measels

Erythematous, non-pruritic, maculopapular rash, includes palms and soles, Koplik's spots, respiratory prodrome, coryza, conjunctivitis, cephalocaudad and centrifugal rash, Forschheimer spots on soft palate, Vitamin A prevents mortality

Hyperbilirubinemia w/in 1st 24 hrs of life must be evaluated immediately. This condition results from Rh [+] father (+D antigen) and previously sensitized Rh [-] mother (-D antigen).

Erythroblastosis fetalis

Diamond-Blackfan Anemia

Erythroid aplasia, platelets and leukocytes normal

______________ this antibiotic is associated with increased risk of hypertrophic pyloric stenosis in infants < 6 wks of age.

Erythromycin

What is the treatment for bacterial conjunctivitis?

Erythromycin (Ilotycin, Romycin) 5mg/g: Instill 1 cm ribbon into each conjunctival sac once. Trimethoprim and Polymyxin B (Polytrim): 1 drop in eye every 3 hours for 7-10 days. Ofloxacin (Ocuflox) .3% 5ml: 1-2 drops in affected eye every 2-4 hrs while awake for the first 2 days then 4x/d for an additional 5 days

full thickness circumferential burn of an extremity may result in vascular compromise. May note loss of pulses and increase in tissue compartment pressure. What is the treatment?

Escharotomy prevents ischemic injury.

Polyhydramnios on prenatal ultrasound and history of drooling, cough and regurgitation with initial feedings immediately on birth.

Esophageal atresia and tracheoesophageal fistula

This is the gold standard for identifying reflux in GERD.

Esophageal pH monitoring

How long does it take for allergic contact dermatitis to evolve?

Evolution 12-72 hours, or months

Seperation Anxiety Disorder

Extreme anxiety in children when they are separated from home or loved ones. 3 of the following is req: 1. excessive distress when separated from home 2. excessive worry about losing major attachment figures 3. refusal to go to school or other locations because of fear of separation 4. nightmares about separation 5. physical complaints when separated (headaches, nausea, abd pain)

T or F. A patient with hypospodias should be circumcised to prevent further complicaitons.

F!!! *Do not circumcise!!!!*-prepuce used for surgical repair at 18 months of age. Refer to pediatric urologist.

T or F: Flat warts are most evident under lateral illumination.

FALSE oblique

T or F. Intransal Cromolyn must be used after the onset of sx.

FALSE, prior to onset

T or F. Since verrucae are caused by various strains of HPV, HPV typing is standard in their treatment.

FALSE, unless in the case of genital dysplasia

T or F. Erythema infectiosum is only contagious when rash erupts.

FALSE. Non contagious when rash erupts.

Galactosemia

FTT, cataracts, jaundice, hypoglycemia

Pityriasis roasea is most common in what seasons?

Fall/spring

T or F, in Tetralogy of Fallot, cyanosis is due to L to R shunting and decreased pulmonary flow.

False. R to L shunting

Alport Syndrome

Familial disorder (X-linked dominant MC) presenting in childhood as recurrent hematuria and proteinuria + sensorineural deafness EM: alternating areas of thinned and thickened capillary loops with splitting of the glomerular basement membrane

____________ characterized by asymptomatic hypercalcemia and very low levels of urinary calcium. This is rare.

Familial hypocalciuric hypercalcemia (FHH)

What are the goals of blood sugars for gestational diabetes?

Fasting < 95. 1 hour < 140. 2 hour less than 120.

When does endocardial fibroelastosis occur? What is the consequence of this disease?

First 2 years of life. Diffuse fibroelastic thickening of the left ventricular endocardium, causing restrictive heart disease.

Evaluation/Diagnosis and TX of Biliary Atresia

First Step - Abdominal US: may show absent or abnormal gallbladder Scinitigraphy - failure of the liver to excrete tracer into the colon is highly suggestive Cholangiogram - diagnostic gold standard in OR TX: Kasai procedure (hepatoportoenterostomy) allows time for growth and liver transplant later

What is the flow diagram for screening children for developmental dysplasia of the hip?

First do a Barlow and Ortolani maneuver. If positive refer to orthopedics. If negative, depending on the age do hip ultrasound if < 6 months or X-ray if > 6months.

What is the typical presentation of infantile hypertrophic pyloric stenosis?

First-born boys ages 3-5 weeks.

Deficiency of Niacin will cause Pellagra. What are good sources of Vitamin B3?

Fish, liver, meat, poultry, grains, eggs, milk

Teracycline and cotrimoxazole frequently cause this kind of drug eruption.

Fixed drug eruption

Neonatal Botulism

Floppy baby, poor suck, lethargy, constipation, weak cry, 6th CN palsy, ptosis, sluggish pupils, deceased gag reflex, supportive care

How to remember NF2 vs NF1? Both clearly cause fibromas, neurofibromas and other tumors.

Focus on the 2 in "NF2" -bilateral acoustic neuromas and cataracts (we have 2 eyes). NF1: cafe-au-lait spots, macrocephaly, feeding issues, short stature & learning disability.

Goat milk is deficient in what vitamin?

Folate (associated goat milk with Mexican cooking and how tortillas are not fortified with folate among other vitamins).

What vaccine needs to be given to a child with certain parameters because of weight?

For HBV vaccine children need to be at least 2 kg.

NOTE:

For kids in early shock you will find the following: elevated heart and respiratory rates, peripheral blood vessel constriction (causing cool, clammy extremities and delayed capillary refill time), and decreased peripheral pulses (due to vasoconstriction and decreased stroke volume). The vasoconstriction can lead to difficulty obtaining a pulse oximeter measurement. Blood Pressure can be completely normal until very late

Most common INHERITED form of mental retardation?

Fragile X syndrome 1. macrocephaly 2. long face, high-arched palate, large ears 3. macroorchidism after puberty

_____ happens due to increased frequency of transient LES relaxations.

GERD

Children with laryngomalacia are at increased risk of what?

GERD Note: this can aggravate and cause airway collapse.

Of the all the diseases which cause galactosemia, which of them is asymptomatic and causes only cataracts?

Galactokinase deficiency.

Newborn with failure to thrive, bilateral cataracts, jaundice and hypoglycemia. Diagnosis and enzyme?

Galactosemia due to galactose-1-phosphate uridyl transferase deficiency.

Newborn or young infant with failure to thrive, bilateral cataracts, jaundice and hypoglycemia. Diagnosis to consider? What are other complications of this disease?

Galactosemia due to galactose-1-phosphate uridyl transferase deficiency. Aminoaciduria, hepatic cirrhosis, and mental retardation.

Hyperbilirubinemia and feeding intolerance of an infant. Diagnosis?

Galactosemia.

Generally infants who present with bacterial meningitis, what is the best thing to do? When would CT be warranted in these infants?

Get an lumbar puncture before antibiotics. CT: for infants who are comatose, have focal neurologic findings, or a history of neurosurgical procedures.

What defines precocious puberty?

Girls < 8 or boys < 9 years of age developing secondary sex characteristics.

Patient presents with edema noted in periorbital and scrotal regions. HTN may be present. Urine is tea or cola colored. Edema of the face and eyes is present in the morning and edema of the feet and ankles occurs in the afternoon and evening. Likely diagnosis?

Glomerulonephritis

Patient presents with: hematuria, edema, and often hypertension. What is the likely dn?

Glomerulonephritis

Verruca plana "the flat wart" is the most common wart in children and adults. What HPV strains cause it?

HPV 3,10,28,41

What strains of HPV cause Verruca Vulgaris (common wart)?

HPV types 1,2,4,7

What is the principle risk factor for erythema multiforme?

HSV

On gross exam what would make it more likely that the patient has HSV gingovostomatitis than herpangina (coxsackie).

HSV: anterior oropharynx and lip involvement. Cox: posterior oropharynx and tonsillar involvement

Hemolytic Uremic Syndrome

HUTS - hemolysis uremic thrombocytopenia syndrome tx. supportive with fluids

Which cause of meningitis is associated with the greatest risk of sensorineural hearing loss in infants?

Haemophilus influenzae type B.

Hereditary Spherocytosis

Hemolytic anemia, jaundice, splenomegaly, spherocytes on peripheral filmm

: severe disease presents with infant males with spontaneous bleeding into joints and soft tissues called hemarthrosis and excess bleeding @ circumcision. Hemarthroses may be spontaneous of follow trauma and retroperitoneal hematomas and CNS bleeds may occur w/o trauma. What is the likely bleeding d/o?

Hemophilia A (factor VIII deficiency)

This factor deficiency presents similarly to hemophilia A and will have the same lab results and treatment.Inheritance is x-linked (only males affected) 1/30,000 births and it is more rare than hemophilia A.

Hemophilia B

NOTE:

Hemophilia and other coagulation disorders commonly present with hemarthrosis (joint effusion with minor trauma) Platelet aggregation disorders such as Von Willebrand dx present with easy mucosal bleeding, ecchymoses, and petechiae

Pathophysiology of breast milk jaundice?

High levels of beta glucuronidase in breast milk, deconjugate intestinal bilirubin and increase enterohepatic circulation.

LGA sequelae

Hip subluxation, talipes calcaenouvalgus

What are risk factors for avascular necrosis of the proximal femur?

Hip trauma/fracture, steroid use, alcoholism, sickle cell disease, rheumatoid arthritis, gaucher dz and SLE

This special test in strabismus determines eye position and reveals misalignment and heterotropia in strabismus.

Hirchberg corneal light reflex

This condition presents in 1st 24-48 hours with failure to pass meconium, abdominal distention, and bilious vomiting. It causes inability of bowel complex to relax resulting in obstipation, vomiting and failure to thrive.

Hirchsprung disease (congenital megacolon)

upper eyelid swelling with EBV infections?

Hoagland Sign, very specific for EBV

Infant at day 3 has gonococcal conjunctivitis. Treatment?

Hospitalization and ceftriaxone or cefotaxime IV.

What causes erythema infectiosum or "5th" disease?

Human Parvovirus B19

Hunter + Hurler disease

Hurler's: corneal clouding, organomegaly, progressively coarsened facies, including frontal bossing, widened nasal bridge, and thickening of the nasopharnygeal tissues BOTH: dysostosis multiplex (bony abnormalities that include a thickened skull, malformed vertebrae, abnormal ribs and clavible)

Newborn infant presents with grunting respirations and diminished, harsh tubular lung sounds. CXR reveals diffuse reticulogranular pattern of uniform distribution causing a GROUND GLASS APPERANCE and doming of the diaphragm. What is the likely diagnosis?

Hyaline Membrane Disease

Newborn infant presents with increased inspiratory effort (accessory muscle use and chest wall retractions) and hypoxemia. PE = tachypnea, grunting respirations, diminished harsh tubular lung sounds. What is the likely diagnosis?

Hyaline Membrane Disease

Treatment of cyclic vomiting syndrome?

Hydration, antiemetics (odansetron), possible sumatriptan for those with migraines and reassurance.

Child who develops isolated thrombocytopenia and petechiae after a viral infection. Diagnosis?

Immune thrombocytopenia.

What are cause of hypercalcemia?

Hyperparathyroidism, neoplastic disease, thiazide diuretics, sarcoidosis Vitamin D intoxication Williams Syndrome - elfin facial appearance, low nasal bridge, cheerful demeanor, ease w/ strangers, developmental delay but strong language skills, aortic stenois & hypercalcemia. Hypophosphatemia Bartter syndrome - hypokalemia, acidosis, hypercalcuria, Gitelman syndrome

After lesion resolution occurs in lichen planus, post inflammatory ________________________ may cocur.

Hyperpigmentation

SGA sequelae

Hypoglycemia, hypocalcemia, hypothermia, polycythemia, asphyxia, meconium aspiration

Infant of diabetic mother

Hypoglycemia, polycythemia, hypocalcemia, jaundice, RDS, hypoplastic left colon, sacral agenesis, thromboses

How does systemic carnitine deficiency present?

Hypoglycemia. Hyperammonemia. Hypoprothrombinemia. Encephalopathy. (very similar to Reye syndrome.)

Ductal-Dependent Systemic Blood Flow Syndromes (esp as an etiology of shock in the first week of life)

Hypoplastic Left Heart syndrome Critical/severe AS Coarctation Interrupted aortic arch Complex CHD with above

This condition is characterized by abnormal placement of the urethra where the meatus is proximal and ventral to its normal or anterior location. Epispadias, dorsal displacement is less common.

Hypospodias

Fatigue, constipation, menorrhagia, dry skin and recent carpal tunnel. What am I?

Hypothyroidism.

Signs of Congenital Hypothyroidism

Hypotonia Large tongue umbilical hernia abdominal distension (constipation) pathological jaundice poor feeding, sluggish hypothermia, noisy breathing refractory macrocytic anemia NOTE: newborns are OK at birth due to maternal hormones in the bloodstream

Down Syndrome Trisomy 21

Hypotonia, upslanting palpebral fissures, epicanthal folds, excess nuchal skin, relative macroglossia, single transverse palmar crease, AVSD>VSD>ASD,

When is CT scan of the sinuses indicated?

IF patient develops complications along with sinusitis including: periorbital edema, vision abnormalities, or altered mental status

NOTE:

IF you start penicillin/amoxicillin within 9 days of an acute Grp A Strep pharyngitis you can prevent rheumatic fever

These diabetics have an elevated C-peptide and strong family history. I or II?

II

Your patient has conjunctivitis caused by n. gonorrhoae...Throw down your eye drops, you know it's time to start...

IM ceftriaxone

After incubation period of 18-72H, patients exhibit an abrupt fever, chills, malaise, muscle aches, substernal chest pain, headache, nasal stuffiness and occasionally nausea. The fever lasts 1 - 7days and is accompanied by coryza, nonproductive cough, photophobia, eye pain, sore throat, pharyngeal injection and flushed facies. Wheezes and rhonchi may be heard and children often develop diarrhea. Likely dx?

INfluenza

Common childhood autoimmune process. Autoimmune bleeding d/o in which pts develop antibodies against their own platelets. Childhood form is usu. acute and follows a preceding illness and is chronic in course.

ITP

Management of HUS

IV fluids and electolyte management transfusions for anemia and thrombocytopenia 50% require dialysis for acute renal failure

What is the treatment for burns?

IV fluids for burns, wound cleaning, supportive measures, transfer to burn center, surgery or physical therapy Fluids w/ burns >10% TBSA. 14-16 gauge venous cannula is placed in 1 or 2 peripheral veins through unburned skin if possible. Given as lactated ringer solution (large amts of normal saline could result in hyperchloremic acidosis)

What is the treatment for severe dehyrdation in the child?

IV fluids until the pt is stabilized. 20 mL per kg of isotonic crystalloid (normal saline or lactated ringer solution) over 10 to 15 minutes. Stabilization often requires up to 60 mL per kg of fluid within an hour

________ and __________ in kawasaki disease reduces the initial inflammation and reduces incidence of complication

IV immunoglobulin and aspirin

Child with ITP and is bleeding, treatment?

IVIg or glucocorticoids.

6 month old presents with pneumonia and has a history of previous infections from Giardia and recurrent ear infections. Likely diagnosis and pathogenesis of that disease?

IgA deficiency due to an abnormal B-lymphocyte maturation.

What is the pathogenesis of Henoch Schonlien purpura/HSP? Treatment?

IgA-mediated leukoclastic vasculitis. Supportive hydration and NSAIDs. Hospitalization and steroids for severe symptoms.

Jessie is a 4y2mo old female presenting to your clinic with a history of what appears to be a 1st exposure drug reaction. You know that since it is the first time, she likely hasn't had any problems and her immune system may have produce an antibody called ___ against the drug.

IgE

3 year old swallows a battery. X-ray shows it is in the esophagus. Best next step? How about if it is in the stomach?

Immediate endoscopic removal. Leave it alone, so it gets expelled on its own.

How does vitamin A improve morbidity and mortality in those with measles?

Immune enhancement and assists GI and respiratory epithelium to regenersate.

Management of Varicella Exposure

In nonimmune neonates and immunocompromised, give VZIG within 10 days of exposure. In nonimmune, asym, and immunocompetent give varicella vaccine within 3-5 days of exposure.

What causes transient tachypnea of the newborn? Treatment

Inadequate alveolar fluid clearance at birth resulting in mild pulmonary edema. Supportive/reassurance as this resolves by day 2 of life.

Sickle Cell Trait

Increased risk for renal issues: 1. hematuria from sickling in the renal medulla 2. Isosthenuria = impairment in concentrating ability, which can present as nocturia and polyuria NOTE: less commonly, increased risk of UTIs, esp during pregnancy

What are the indications for a VCUG?

Indications for imaging: imaging after the first UTI in all children < 5 years, children of any age with febrile UTI, and boys with UTI. Children with prenatally identified hydronephrosis should be evaluated postnatally, however US performed during the first 3 days of life may have high false negative result because of relative dehydration during neonatal period.

Descending flaccid paralysis in an infant. Findings are bulbar palsies, constipation and hypotonia. No history of honey consumption. Diagnosis and treatment?

Infant botulism. Human-derived botulism immune globulin. Note: even if there is no history of honey consumption this is the diagnosis.

How do you treat infant botulism? how about food-borne botulism?

Infant: human-derived botulism globulin. Food: equine-derived botulism antitoxin.

A child has a febrile UTI at the age of 15 months. What are we recommended to do? How about recurrent?

Infants and toddlers with a first febrile UTI at ages 2-24 months must undergo a renal ultrasound to evaluate for anatomic abnormalities. Recurrent UTIs in this age group warrants further evaluation with a voiding cystourethrogram.

Newborns born to mothers with Graves' disease. Presentation? Pathogenesis? Treatment?

Infants with fetal tachycardia, hydrops and growth restriction. Anti-TSH receptor antibodies via transplacental passage. Treatment: methimazole & beta blocker.

How are pinworms transmitted?

Infection transmitted by pts *hands*, food, drink and fomites. Eggs are swallowed and hatch in duodenum; larvae pass to cecum and mature in 3 to 4 weeks. May affect multiple family members.

Immediate, hyperacute, hours after surgery

Inflammation from surgery, reaction to blood products, malignant hyperthermia

An orhthomyxovirus causes what infectious disease?

Influenza

In terms of developmental dysplasia of the hip, what other exam findings can suggest this disorder other than Barlow and Ortolani maneuvers?

Inguinal folds that extend beyond the anal aperature

What is the most commonly performed surgical procedure in children?

Inguinal hernia repair

What is the treatment of duodenal atresia?

Initially, feedings withheld, nasogastric or orogastric tube for suction. IV access for parenteral fluids. Broad spectrum antibiotics given to prevent postoperative infection (ampicillin and gentamicin) to prevent postoperative infection Decompression of GI tract and surgery to open the bowel obstruction. Duodenoduodenostomy indicated.

1 day old boy had a forceps-assisted vaginal delivery done. Complicated by shoulder dystocia. Exam shows a supinated left forearm with extension of the wrist, hyperextension of the metacarpophalangeal joints and flexion of the interphalangeal joint. Grasp reflex is absent in the left and findings of Horner's on the left. Diagnosis?

Injury to C8 and T1 causing Klumpke palsy and Horner syndrome.

Laryngomalacia

Inspiratory stridor that is sensitive to air flow changes

Congenital syphilis how does the disease present?

Intermittent fever, osteiti and osteochondritis, mucocutaneous lesions, lymphadenopathy, rhinitis (snuffles) and hepatomegaly.

Trauma to roof of mouth with a sharp object and hemiplegia, hemianesthesia and motor aphasia. Diagnosis?

Internal carotid artery dissection

POD 8+, Delayed

Intra-abdominal abscess, DVT/PE, drug fever, cholesystitis, URTI, C. difficile, endocarditis

NOTE:

Intraventricular hemoorhage (IVH) is a common complication in neonates born at < 30 weeks gestation or < 1500 g (3.3 pounds) SCREEN with serial head ultrasounds can PRESENT without symptoms; lethargy, hypotonia, high-pitched cry, bulging fontanels with severe hemorrhage NOTE: communicating hydrocephalus is a complication in 33% of cases NOTE: prevention of preterm labor and antenatal corticosteroids can reduce incidence and improve mortality

Patient presents with paroxysmal colicky abdominal pain --> vomiting --> diarrhea --> becomes bloody w/ mucus = currant jelly stool. That is the likely diagnosis?

Intussuception

The most frequent cause of intestinal obstruction during first 2 years of life. 3:1 males. Idiopathic, MC location = ileocolic.

Intussuception

This is described as Intestinal invagination at ileocecal valve Telescoping of the bowel onto itself. May lead to impaired blood supply and necrosis.

Intussuception

What is the relationship b/t calcium and phosphorous?

Inverse. As calcium goes up, phosphorous goes down.

What disease is associated with breath holding spells?

Iron deficiency anemia.

What is the most common nutritional deficiency in infants and children?

Iron deficiency.

What are some symptoms of polycythemia in a neonate? Treatment?

Irritation, respiratory distress, tachypnea, cyanosis, and poor feeding. If symptoms present, exchange transfusion.

4 year old boy presents with thick golden-yellow crusts on his chin and cheek. How do you go about treating the child?

Is it non-bullous or bullous? (pustules vs flaccid bullae... bullae look like transparent ticks) If non-bullous - a topical antibiotic such as mupirocin is useful. If bullous - oral antibiotics like cephalexin, dicloxacillin or clindamycin is suggested.

In extremely nodular or cystic acne, what medicaiton can be effective?

Isotretinoin (Accutane) is very effective. Triamcinolone can be injected into cysts.

Is congenital myasthenia a thing? How is it different from botulin toxin?

It is real. Congenital MG lacks autonomic symptoms like constipation and drooling in botulism.

How does Wiskott-Aldrich syndrome present? (Low IGM)

It presents with a young boy with eczema, thrombocytopenia and recurrent infections with encapsulated organisms. Serum studies will show high IgA and IgE levels and low IgM This disease is literally the opposite of Hyper IgM Syndrome.

Characterized by chronic synovitis. Also note villous hypertrophy, hyperplasia of synovial lining, edema, hyperemia and increased lymphocytes and plasma cells. MC in girls with age of onset < 16.

Juvenile Rheumatoid Arthritis

Adolescent teen with nasal obstruction, visible nasal mass with bony erosion and frequent nosebleeds. Diagnosis?

Juvenile angiofibroma.

What is the MC cause of arthritis under the age of 16?

Juvenile rheumatoid arthritis

What is the best way to dx any kind of tinea?

KOH prep

Fever >5 days plus: bilateral nonexudative conjunctivitis, mucositis (injected lips, pharynx, or strawberry tongue), cervical lymphadenopathy with at least one LN >1.5cm, erythematous rash, extremity changes (edema & erythema) in a child < 5

Kawasaki disease aka mucocutaneous lymph node syndrome TX: aspirin plus IVIG within 10 days of fever onset to prevent complications: 1. coronary artery aneurysms 2. myocardial infarction and ischemia Management: =>baseline echocardiography and repeated in 6-8 weeks

Acute management of Epiglottis?

Keep child calm and comfortable as possible Emergent intubation in the OR (controlled enviornment). Tracheostomy as a seocnd option. Do not perform Cricothyroidtomy in children <12 due to risk of subglottic stenosis

90% of nose bleeds are anterior. What is the source of the bleed?

Keisselbach Plexus

Patients with tinea capitus present with erythema, scaling, and broken hairs on the scalp. What are the features of the likely "bugs"?

Kerion: boggy mass- M. canis (fluoresces yellow-green), or T. tonsurans (fluorescence) Trichotillomania

________ results from toxic bili levels > 20-25 mg/dl and associated w/ encephalopathy

Kernicterus

UTI in a child <2 years and general recommendations

Kids <2 should be treated with antibiotics for 1-2wks Renal and bladder U/S to evaluate for anatomic abnormalities such as Vesicoureteral Reflux IF: 1) first febrile UTI <2 years 2) recurrent febrile UTIs any age 3) UTI with family hx of renal, urologic dx, HTN, or poor growth 4) IF no response to antibiotics Get repeat blood work and urine culture in kids who fail to improve after 2-3 days on antibiotics Voiding cystourethrogram (gold standard for diagnosing VUR) IF hydronephrosis or scarring on U/S or pt has recurrent UTIs. Also in newborns <1 month and kids <2 with recurrent UTIs or first UTI by organism other than E.coli NOTE: VUR is a risk factor for recurrent UTIs, pyelonephritis => focal parenchymal scarring and blunting of the calices => renal scinitigraphy with DMSA is preferred for long-term evaluation of renal scarring => monitor renal function with creatinine

Patients with lichen planus may form in lines related to scratching. This is called the _______ phenomenon.

Koebner

What is the best imaging study to diagnose appendicitis? What else can be noticed on labs?

Labs reveal a leukocytosis with a left shift. Urinalysis is normal, a few white and RBCs may be noted. US or CT scan of the appendix = dilated and thickened wall.

What procedure is done to fix a volvulus in a child?

Ladd procedure (fixes bowel in a non-rotated position by surgical division of Ladd's bands, widening of the small intestine's mesentery, performing an appendectomy and correctional placement of the cecum and colon)

Laryngomalacia VS. Vascular Ring

Laryngomalacia => Stridor is most severe at 4-8 months => Persistent stridor that worsens with supine position and IMPROVES in prone position Vascular Ring => presents before age 1 year => persistent stridor that improves with neck extension => associated with cardiac abnormalities (50%)

What can be used to predict the risk for developing hyaline membrane disease? What therapeutic option can be used *prophylactically*?

Lecithin-to-sphingomyelin ratio = used to predict risk for development of HMDz. Surfactant replacement

Anaerobic pharyngitis (or due to fusobacterium necrophorium) that spreads to...jugular venous thrombus and septic emboli in the lung

Lemierre Syndrome

Patient presents with malaise, fatigue, fever, pallor, lymphadenopathy, weight loss, bone pain, intermittent fevers, hepatosplenomegaly PE: pallor, petichiae, easy bruising. What is the likely diagnosis?

Leukemia

Frequent infections and a history of delayed separation of umbilical cord. Diagnosis?

Leukocyte adhesion defect.

What will lab values show in orbital cellulitis?

Leukocytosis

What will you see on diagnostic studies of influenza? What is the best way to diagnose?

Leukopenia and proteinuria may be found. Virus can be isolated from the throat or nasal mucosa. Viral cultures take 3-7d to return. Direct immunofluorescent tests are labor intensive and less sensitive, but recently developed rapid serology tests are proving to be helpful. Results are most accurate during 1st few days of illness.

These class of asthma medications work by decreasing mucus production and airway edema?

Leukotriene Modifiers o Montelukast: Singulair o Zafirlukast: Accolate o Zileuton: Zyflo

Uncommon, inflammatory paqulosquamous d/o of unknown cause. Rare in children < 5 and more common in women. This type of reaction reaction is noted w/ certain drugs such as gold salts, beta blockers, antimilarials, thiazide and furosemide.

Lichen planus

Recurrent cystitis and constipation in a toddler. what is the cause of the cystitis?

Urinary stasis due to the compressive effect of a dilated rectum.

Lice and scabies are typically tx with premethrin, lindane or oral ivermectin. Avoid which one in infants?

Lindane

Deficiency of Vitamin A will cause night blindness and dry skin. What are good sources of Vitamin A?

Liver, dairy, yellow and dark green leafy vegetables

Marfan Syndrome

Long fingers, hypermobile joints, pectus carinatum, aortic aneurysms, Autosomal dominant, fibrillin-1 gene

What symptoms are associated with complex partial seizures?

Loss of consciousness with automatisms such as chewing, picking movements of the hands or lip smacking.

Patients with tinea corprois present with Scaly, annular, red papules, with raised border and a central clearing. What are the likely "bugs"?

M. canis, T. mentagrophytes

Presentation of lymphadenopathy due to non-tuberculous mycobacteria?

MAC causes unilateral subacute-chronic lymphadenopathy associated with a violaceous color and less than 4 cm.

Acute Lymphoblastic Leukemia

MC in children 2-10 >25% lymphoblasts is diagnostic +TdT immunostaining lymphoblasts contain cytoplasmic aggregates of periodic acid Shiff (PAS+) but lack PAS+ granules

Impetigo

MCC Staph/Grp A Strep Presents as multiple painful pustules that over the course of a week rupture and harden into golden-yellow crust w/ predisposing factors including eczema or skin trauma (e.g. insect bites) TX: topical antibiotics (mupirocin) USE oral antibiotics for bullous impetigo (flaccid bullae) - oral cephalexin, dixcloxacillin, or clindamycin OR if non-bullous impetigo is widespread

Myocarditis

MCC by coxsackievirus B and adenovirus Symptoms: + Viral prodrome => worsening respiratory distress due to acute left heart failure and pulmonary edema => holysystolic murmur due to cardiomyopathy and functional mitral regurgitation + Hepatomegaly, cardiomegaly (>60% < 1; >50% >1) TX: ICU, diuretics and inotropes

Laryngomalacia

MCC of chronic stridor in infants and is characterized by inspiratory stridor that is worse in the supine position and exacerbated by crying or feeding. => loudest at 4-8 months DX: confirmed with direct visualization of the larynx by laryngoscopy, showing omega-shaped epiglottis and collapse of the supraglottic structures during inspiration OFTEN there is a spontaneous resolution by 18 months, though increased risk for GERD. TX: supraglottoplasty for severe symptoms (poor weight gain, apnea, cyanosis, rapidly worsening stridor)

Ewing sarcoma

Malignant medullary tumor in long bones, pain fever, leukocytosis, onion skin appearance

Bilious vomiting and abdominal distension in a 15 day old.

Malrotation with midgut volvulus.

Homocystinuria

Marfanoid habitus, [fair skin, developmental delay, cerebrovascular accident], megaloblastic anemia, autosomal recessive

Marfans vs. Ehler Danlos Syndrome vs. Homocystinuria

Marfans - long extremities and fingers, upward lens dislocation, and pectus carinatum; joint hypermobility, skin elasticity, scoliosis/kyphosis => AD, defect in fibrillin-1, leading to systemic weakening of connective tissue => aortic root dilation (=> regurgitation) and mitral valve prolapse are complications Homocystinuria => AR, cystathionine synthase deficiency resulting in errors in methionine metabolism FEATURES: + Marfanoid body habitus, downward lens dislocation*** + mitral/aortic regurgitation; no aortic root dilation*** + large, stiff joints*** + fair complexion + megaloblastic anemia + developmental delay/intellectual disability **** + thromboembolic events Diagnosis - elevated homocysteine and methionine levels TX - Vit B6, B12, folate to lower levels; antiplatelet and anticoagulation Ehler-Danlos - defect in type V collagen; NOT TALL + hyperextensible joints + fragile blood vessels that cause easy brusing + tissue paper-thin scars + cardiovascular complications => mitral valve prolapse => aortic root dilation

These category of asthma medications inhibit histamine release. Useful in allergen and exercise bronchospasm.

Mast cell stabilizers o Cromolyn: Intal o Malizumab: Xolair

What is the most common location for sinusitis in the child?

Maxillary sinus

Precocious puberty, cafe au lait spots and multiple bone defects (polyostotic fibrous dysplasia). Diagnosis?

McCune-Albright syndrome.

Normal RBC values in infants

Mean hemoglobin at birth is 16.5, and the mean for a healthy full-term 2-month-old infant is 11.2, and this decrease in hemoglobin occurs because fetal RBCs have a short half-life that leads to a physiological nadir for hemoglobin, called "physiological anemia" NOTE: in full term newborns the marrow is stimulated to produce new RBCs only when hemoglobin reaches its nadir at 7-9 weeks at 11, after which hemoglobin rises

Koplik spots. Diagnosis?

Measles (rubeola)

Is Hirschsprung or meconium ileus associated with inspissated meconium (thickened).

Meconium ileus.

Where is the level of obstruction with meconium ileus? How about Hirschsprung?

Meconium ileus: Ileum (literally in the name). Hirschsprung disease: Rectosigmoid.

How do you treat croup?

Nebulized racemic epinephrine.

What is the treatment for TM perforation?

Most will heal spontaneously. A follow-up hearing test is needed to confirm that hearing has returned to baseline. Patient should be instructed not to get any water in ear. Management: keep ear dry, refer to audiologist, drop use is controversial (cortisporin otic suspension, ciprofloxacin ophthalmic- Ciloxan), surgery for large perf

What is the management for an umbilical hernia?

Most will spontaneously resolve with eventual closure of umbilical ring by 12 months. Surgical intervention only in a minority of patients to avoid incarceration or strangulation.

Treatment of CF is focused on maintenance = slowing the progression of lung damage by improving mucus clearance, controlling infection and inflammation.. How can you imrpove mucus clearance? Pancreatic insufficiency? What should antibx slection cover?

Mucus clearance is improved with percussion and postural drainage. Pancreatic insufficiency treated with pancreatic enzyme supplements. Antibiotic selection should cover for pseudomonas with anti-pseudomonas beta-lactam agents and an aminoglycoside or ciprofloxacin.

What is the treatment for impetigo?

Mupirocin ointment; Anti-staphloccal antibiotic.

What organism causes atypical mycobacterial disease?

Mycobacterium avium-intracellulare

What will you find on XR in subluxation of radial head?

NORMAL

What are the recommended treatments for influenza?

Neuraminidase inhibitors (Zanamivir inhalation [Relenza] or oral Oseltamivir [Tamiflu]) will significantly reduce severity if given w/ in 48 hours of the onset of symptoms. They are effective against both influenza A and influenza B and have fewer side effects. Recommended for pts with influenza requiring hospitalization or in those with high risk of morbidity and mortality.

Abdominal Masses:

Neuroblastoma (~ 2 years) - MC extracranial solid tumor of childhood; arises from neural crest cells => Calcifications/hemorrhages with imaging of firm and nodular mass => 70% have metastatic dx on presentation => HVA and VMA are elevated (similar to pheochromocytoma) => Genetics: amplication of N-myc proto-oncogene and hyperdiploidy Wilms tumor - arises from the metanephros (precursor to renal parenchyma)

Weight loss/gain in the Newborn

Newborns almost always lose some weight in the first few days after birth. Most will lose between 5-10% of their birth weight, and will be back to birth weight by 10-14 days of age. NOTE: weight gain averages 20-30 grams/day in the first 4 months; healthy full-term infants need to consume 100-110 kcal/kg/day in the first 4 months

Pellagra is caused by deficiency of _______ or Vitamin B3

Niacin

Sheet like peeling of the epidermis with slight pressure is called a positive ____________ sign and is often seen in toxic epidermal necrosis

Nikoloksy

What is the criteria for d/cin a croup pt?

No stridor, 3 hrs. since last epi, well appearing, O2 sat >90% on RA, reliable parents.

Do you need to diagnose a child with a renal biopsy for minimal change disease instead of initiating prednisone?

No. Initiate empiric steroid therapy on suspicion of the diagnosis.

This type of hydrocele is caused by the processus vaginalis being obliterated during development. Will usu. disappear by 1 year of age. communicating or non communicating?

Non communicating

If a patient with impetigo presents with nonbullous honey colored crust, what is the likely causative organism? If they present with a bullous honey colored crust, what is the most likely causative organism?

Nonbullous - S. Aureus or GAS Bullous - S. Aureus

Child with unilateral cervical adenitis that developed slowly and is tender with minimal cellulitis. Cat exposure present. Treatment?

None.

What are hormonal tx for acne?

OCPs, spironolactone

Duodenal atresia may be an isolated finding but it is often associate with other malformations such as...

Often associated with other malformations- biliary atresia, agenesis of the gall bladder, cardiac, renal, and vertebral anomalies.

How does epiglottitis present?

Older child with drooling and stridor and SIRS. Lateral neck x-ray will show swollen epiglottis (thumb sign), thickened aryepiglottic folds and obliteration of the vallecula.

What diagnostic test is abnormal in regards to minimal change disease? Is it light microscopy of a renal biopsy, immunofluorescence staining of the glomeruli or electron microscopy?

Only electron microscopy will confirm MCD as it will show "effacement of foot processes of podocytes"

What is the most common tumor of in toddlers with neurofibromatosis?

Optic pathway gliomas.

Patient presents with multiple white patches on the tongue, palate and other areas of the oral mucosa. The patches are easily removed by scraping with a tongue blade to reveal a red, irritated mucosa. What is the likely diagnosis?

Oral candidiasis

These category of asthma medication is effective for acute exacerbations and for asthma not well controlled. Chronic use associated with adrenal suppression, osteoporosis, cataracts, HPA growth suppression, ect.

Oral steroids o Methylprednisolone: Solumedrol o Prednisone/Prednisolone

Patient presents with swelling of upper and lower eyelid, swelling and bulging of the eyeball, decrease in the childs ability to move the eyeball, decrease in vision, fever, general discomfort. Patients may present with proptosis. What is the likely dx?

Orbital cellulitis

This is a serious complication of sinusitis but other causes include dental infections, facial infections, infection of the glove or eyelids and infections of the lacrimal system. Less often, it results from trauma.

Orbital cellulitis

5 year old with pain with eye movements, proptosis, ophthalmoplegia and diplopia. Diagnosis? Most common predisposing factor for this condition.

Orbital cellulitis. Bacterial sinusitis.

2 week old infant presents with green vomit. Initial KUB is gas-less next best step and why?

Order an upper GI series (barium swallow to see anatomy from esophagus to duodenum) to evaluate for malrotation.

Differential Diagnosis for Failure to Thrive

Organic Causes: 1. CHF - difficulty feeding and respiratory distress, crackles, hepatomegaly 2. Formula allergy - intestinal blood loss (gross or occult) and reducing substances; fussiness after feeds and vomiting 3. Gastroenteritis - vomiting and diarrhea; fever and/or bloody stools; giardia has a more indolent, chronic course 4. hypothyroidism - poor feeding (not eager) and constipation 5. Malabsorption - good caloric intake and loose stools in the setting of good caloric intake

Attempts to identify the hip that is dislocated or subluxed. Grasp the medial aspect of the flexed knee with the thumb and fully abduct the hips. Feel for spasm or a clunk (not a click sound) as the hips are brought to full abduction. Which test is this?

Ortolani

Tibial tubercle apophysitis manifests as...

Osgood-Schlatter's

What is the #1 bone tumor in children?

Osteochondroma

Panner Disease

Osteochondrosis of the capitellum of the humerus. Typical patient is an adolescent who is actively engaged in sports that involve throwing. Common complaints: 1. chronic dull pain 2. crepitation 3. loss of pronation and supination

One of the MC primary malignant tumors of bone. Typically noted in ages 10-30 yrs and more common in males.

Osteosarcoma

What is the most common form of ASD? Ostium Primium, Ostium Secundum or Sinus venosus?

Ostium Secundum

In a patient > 3 weeks, < 3 mos with pneumonia what is the recommended outpatient treatment if afebrile and w/o respiratory distress? What is the recommended inpatient treatment if febrile or hypoxic?

Out pt - Azithromycin or Erythromycin Inpt - Azithromycin + Cefotaxime + Ampicillin

What pathogens are most likely to cause OE?

P. aeruginosa, group A strep, aspergillus (fungi) and S. aureus.

Acute onset of OCD in a child, what should you be cautious for?

PANDAS. Pediatric autoimmune neuropsychiatric d/o associated with streptococcal infections.

______________ a small blood vessel that connects the aorta and the main pulmonary artery. During pregnancy, it allows blood to bypass the lungs because the baby gets oxygen from the mother. Usu. closes soon after birth.

PDA

continuous, machine-like murmur; a bit louder in systole

PDA

Classic continuous machinery systolic murmur. Mid diastolic murmur at the apex. This is the murmur of which congenital heart defect?

PDA o Starts after S1, peaks at S2 and softens during diastole

Infants whose mothers contracted rubella in the 1st trimester are at increased risk for...

PDA • Typically closes spontaneously by 4 days of age.

What will you find on physical exam in a patient with HCM?

PE: mitral regurgitation, S4 and prominent left ventricular impulse.

Though the PE is usu. normal in colic, it is done to rule out...

PE: normal and done to rule out intussusception, strangulated hernia or other causes

Musty odor in urine in a 5 month old with projectile vomiting, diarrhea and generalized tonic-clonic seizure. Diagnosis and test? Pathogenesis?

PKU. Quantitative amino acid analysis Hyperphenylalaninemia causing neurologic injury due to autosomal recessive mutation in phenylalanine hydroxylase.

This class of medication in GERD decreases HCl secretion and gastric volume.

PPI (omeprazole, lansoprazole)

Patient presents with hot potato voice, severe odynogphagia; and on PE, Infected tonsil is displaced to the midline or beyond. Asymmetric rise of uvula. What is the likely dx?

PTA

Medial epicondylitis, pain with "passive" wrist extension or flexion?

Pain with passive wrist extension Note: if you don't remember if the wrist flexors or extensors are attached to the medial or lateral epicondyle, flex and extend your wrist to know which muscle is in action at which origin.

Osteoid osteoma

Pain worse at night, relieved with ASA, sclerotic lesion with central lytic nidus, often in lower extremities or spine, unilateral, 2nd decade of life

How does paraphimosis (inability of the retracted foreskin to reduce) present? How does it occur?

Painful swelling of the foreskin and penis distal to the phimotic ring. Occurs if the foreskin remains retracted for a prolonged period of time.

Milk protein allergy, presentation?

Painless recta bleeding that resolves by age 1.

How does symptomatic Meckel diverticulum usually present?

Painless rectal bleeding. Rare: intussusception.

What antibody is used for the treatment of RSV?

Palivizumab

How far do you need to go when working up a neonatal infection?

Pan culture including a LP

How is pancreatic insufficiency diagnosed in CF?

Pancreatic insufficiency is diagnosed by demonstration fat malabsorption with a qualitative or quantitative fecal fat test.

NOTE:

Paradoxical breathing is almost always a sign of very severe respiratory distress due to respiratory muscle fatigue

Croup is a subglottic inflammation. What are usually the causative agents?

Parainfluenza viruses, RSV, adenovirus, influenza

Inability of the retracted foreskin to reduce is called _________ while - inability to retract foreskin is called _______

Paraphimosis, phimosis

Cyclic vomiting syndrome is most associated with what?

Parents who have a history of migraine headaches.

characterized by involvement of < 4 medium to large joints. Patients are also at risk for development of asymptomatic uveitis that may lead to blindness if they have +ANA. What kind of juvenile R. Arthritis is this?

Pauciarticular

What is the treatment for HepC?

Pegylated interferon-alpha and ribavirin beneficial

__________ (photosensitive pigmented dermatitis) is a rare entity in US, but common manifestation of niacin difficiency in poorer countries where local diet consists of cereal, corn, or sorghum.

Pellagra

What is the best first step in evaluating primary amenorrhea? Why?

Pelvic ultrasound. Doing so assumes the disease process is due to a hormonal issue or a genetic issue (abnormal mullerian development or androgen insensitivity syndrome)

Patient presents with discrete erythematous micropapules and micropapulovesicles that often become confluent on the face with "sparing of the vermilion border". What is the likely diagnosis?

Perioral dermatitis

What should a child be worked up for if cyanosis is present in a neonate?

Persistent pulmonary hypertension.

What diagnostic test would suggest or confirm pertussis? Treatment?

Pertussis culture or PCR. Lymphocyte-predominant leukocytosis. Macrolides: azithromycin, erythromycin and clarithromycin.

Gastrointestinal tract polyposis and mucocutaneous pigmentation. Diagnosis?

Peutz-Jeghers syndrome.

Cause of Fetal Hydrantoin Syndrome?

Phenytoin; nail and digit hypoplasia, cardiac defects, mental retardation, and dysmorphic facies

___________ reduces the risk that total bili conc will reach level at which transfusion is recommended. Jaundice therapeutics.

Phototherapy

2 day old asian girl with jaundice at the nursery. Uncomplicated delivery with appropriate prenatal care. Labs show total bilirubin of 7 and conjugated bilirubin of 0.4. Diagnosis and pathogenesis?

Physiologic jaundice of newborn due to decreased hepatic uridine diphosphogluconurate glucuronosyltransferase activity.

What tumor tends to cause Parinaud's sign and collier's sign in kids?

Pinealoma Parinaud's: paralysis of vertical gaze Collier's: retraction of eyelid.

Most prevalent helminthic infection in the US. Typically affects children in day care centers, institutionalized individuals and people living in crowded places.

Pinworms

Patient presents w/ nocturnal anal pruritis MC, insomnia, abdominal pain, worms (white threads) or eggs on perineum or in stool. Scratching causes excoriations and secondary infections (impetigo). Migration can cause vulvovaginitis, diverticulitis, appendicitis, cystitis and granulomatous reactions. Likely diagnosis?

Pinworms

Lesions are salmon-colored oval plaque, 1 to 2 cm in diameter, with a fine scale at periphery typically located first on the trunk. The first lesions is called the herald patch (often confused w/ tinea). Children may show lesions on the groin, elbows and knees. Multiple smaller lesions appear later on the trunk and give a Christmas tree appearance. Dx?

Pitryriasis Rosea

Presents with single Herald Patch- oval slightly scaly plaque 2 cm, salmon colored with fine callorette of scale. 2 weeks later-fine scaling papules and plaques with collarette of scale in a Christmas tree distribution of trunk. What is the likely diagnosis?

Pityriasis Rosea

What are the 6P's of Lichen Planus?

Planar, purple, polygonal, pruritic, papules, plaques

Thin curvilinear areas of lucency in an infant KUB

Pneumatosis intestinalis due to necrotizing enterocolitis.

____ is a bacterial infection that may develop secondary to aspiration.

Pneumonia

- resembles adult RA with its symmetric involvement and involves > 5 of the small and large joints. Symptoms = low grade fever, fatigue, rheumatoid nodules and anemia. What kind of juvenile R. Arthritis is this?

Polyarticular

Peutz-Jeghers Syndrome

Polyposis, estrogen secreting tumor causing precocious puberty,

Reye Syndome

Post viral encephalopathy, hyperammonemia, elevated PT/PTT/INR, rash, vomiting, microvesicular fatty liver, cerebral edema, exacerbated by Aspirin

Rabies

Post-exposure prophylaxis: Rabies immune globulin and active immunization

Acquired Aplastic Anemia

Post-infectious, iatrogenic, radiation or toxin mediated weakness, fatigue, petechiae,

Nephritic Syndrome

Post-streptococcal glomerulo-nephritis (PSGN): coca cola colored urine and HTN Dnase B: best senisitivty and specificity (better than ASO) low C3 for 6-12 weeks but rebounds C4 is normal UA: hematuria, proteinuria, RBC casts IgA nephropathy - MCC of gross hematuria (recurrent and painless), can be triggered by infection C3 is NORMAL Bx: mesangial deposits of IgA in glomeruli TX: benign, spontaneously resolve, might recur Membranoproliferative GN SX: acute nephritic syndrome Low C3, and stays low (unlike PSGN) TX: prolonged steroids, poor prognosis Lupus nephritis Sx: microscopic hematuria to rapidly progressive GN Low C3 and C4 Alport Syndrome X-linked syndrome hx of hearing loss in male family membranes Dx: biopsy with splitting of basement membrane

In testicular torsion, testes is usu. high or low rding? Is epididymis anterior or posterior?

Posterior

Respiratory distress. Flattened facies, abdominal distension and clubfeet. Diagnosis?

Posterior urethral valves causing potter sequence. Path: congenital thick folds of mucous membrane located in the posterior urethra.

Guillain-Barré Syndrome

Postinfectious ascending polyneuropathy, demyelenation of motor and sensory nerves, autonomic dysfunction, dx with elevated CSF protein, normal glucose, absent pleocytosis, associated with campylobacter jejuni

Bell's Palsy

Postinfectious or autoimmune facial neuritis, unilateral facial weakness, loss of taste anterior 2/3 tongue, preceded by EBV in %20 of cases, also preceded by HSV

Anemia of prematurity presentation? Why do preterm infants tend to present this way?

Pre-term infant with low H&H and low reticulocyte count with normocytic, normochromic RBCs. Frequent phlebotomy, low EPO and short RBC life span.

What is the sanfransico syncope rule?

Preceding or provocative event: prolonged standing (vasovagal), immediately on standing (orthostatic), with exertion (aortic stenosis, CAD, cardiomyopathy, Arrhythmia, Misc causes), after exertion in athlete (vasovagal syncope), valsalva (reflex mediated syncope), neck rotation or pressure (carotid sinus hypersensitivity), use of arms (subclavian steal syndrome), stressful event (vasovagal)

What is the treatment for erythema multiforme, a skin d/o that occurs in younger males that presents with targetoid "iris" lesions that are dusky, red and round, maculopapules that are typically on extremities and may have mucosal involvement?

Prednisone

What is the presentation of neonatal displaced clavicular fracture? Treatment?

Pregnancy associated with fetal macrosomia, instrumental delivery and shoulder dystocia in a baby who is in pain with passive ROM of extremity, crepitus over the clavicle and asymmetric moro reflex. Reassurance., use of a long sleeved garment, and gentle handling.

Intraventricular hemorrhage, greatest risk factor for this in a newborn?

Prematurity.

What is characteristic of a cranial meningocele

Presence of pulsations. Increased pressure with crying. X-ray evidence of bony deficits.

Vascular Ring

Present with biphasic stridor and feeding difficulties DX: screening with barium swallow followed by confirmation with MRI with angiography

Necrotizing Enterocolitis

Presents as abd distension, bloody stools, and vital sign instability in premature infants PATHO: gut immaturity and exposure to bacteria from enteric feeds, leading to a cascade of inflammation and damage to the bowel wall X-RAY: pneumatosis intestinalis (extravasation of gas INTO the damaged bowel wall) & linear branching areas of lucency over the liver representing air in the portal veins COMPLICATIONS: severe intestinal necrosis => perforation and pneumoperitoneum NOTE: decreased rates of NEC with breastfed premature infants

Lateral neck x-rays of a retropharyngeal abscess would show what?

Prevertebral soft-tissue thickening.

What groups does thalassemia primarily effect?

Primarily affects people of Mediterranean, African or Asian ancestry. (Esp. where Malaria is endemic)

What are primary dietary sources for vitamin D?

Primary dietary sources are oily fish (salmon, mackerel, sardines), cod liver oil, liver and organ meats, and egg yolk. Vitamin D is fortified in many foods particularly milk and milk products, orange juice, bread, cereals, infant formula.

Most common form of nonnutritional rickets?

Primary hypophosphatemia (X-linked dominant) => proximal kidney tubule defects in phosphate reabsorption and conversion of 25(OH)D to 1,25(OH)2D are seen Findings: 1. low-normal serum calcium 2. moderate low serum phosphate 3. low serum 1,25 Vit D 4. hyperphosphaturia 5. no evidence of hyperparathyrodism

What may be noted before the appearance of lesions in HSV2 infection?

Prodromal parasthesias may be noted 12 - 24H prior to appearance of the lesions.

Pyloric Stenois

Projectile vomiting, palpable olive shaped mass in epigastrum

This class of medication in GERD increases LES pressure and gastric emptying. Examples are bethanechol and metoclopramide.

Prokinetics

What will you see on lab in pts with vWb Dz?

Prolonged Bleeding Time, Prolonged PTT (due to factor VIII deficiency)

QT prolongation on EKG in a toddler who fainted while running. Treatment?

Propranolol and pacemaker.

What is the treatment for avascular necrosis of the femoral head?

Protective weight bearing should be started until definitive tx. Surgery indicated in most cases, if collapse has occurred hip arthroplasty.

Minimal change disease

Proteinuria, hyperlipidemia, edema, hypoproteinuria, histologically normal

How does atopic dermatitis present in infancy?

Pruritic scaly erythematous lesions on the face, chest and extensor surfaces of the extremities.

prominent systolic ejection click just after S1; harsh murmur

Pulmonic stenosis

Crescendo-decrescendo systolic ejection murmur at the left upper sternal border and single S2 in a 2 month old boy who is cyanotic.

Pulmonic stenosis finding and likely tetralogy fallot diagnosis.

What is the flow diagram of precocious puberty?

Purpose is to make a workup which separates central from peripheral causes What is the bone age? Advanced bone, check LH and follow up with GNRH stimulation test if low. Normal determine if the issue is isolated at breast development or pubic hair (thelarche vs adrenarche)

How do you diagnose acute bronchiolitis?

RSV nasal swab test & PCR rapid antigen

What are the 3 types of renal tubular acidosis and their pathogenesis?

RTA 1: defect in hydrogen excretion. RTA 2: defect in bicarbonate reabsorption. RTA 4: defect in sodium/potassium exchange (aldosterone resistance).

What will physical exam reveal in tetralogy of fallot?

RV heave noted with loud systolic ejection murmur at LSB. Clubbing of fingers may be present with older children. EKG reveals RV enlargement and right axis deviation. CXR shows normal heart size and decreased pulmonary vascularity

5 year old boy presents with dysphagia. Past week of chills, fever, malaise, muscle aches and sore throat. Currently refuses to drink fluids. Sustained a bite to her right forearm (there is a scar at the site). Exam shows a disoriented girl with copious drool and facial grimacing. Full range of motion of neck. Diagnosis?

Rabies.

A 2 year old is presenting with severe respiratory decline. Treatment option?

Racemic epinephrine prior to intubation.

What imaging is indicated in the evaluation of short stature?

Radiography of the distal radius to determine bone age

The optimum treatment for appendicitis is surgery, with broad spectrum antibiotics. What is the recommended antibiotic regimen?

Recommended preoperative intravenous regimens include cefoxitin or cefotetan 1-2 g every 8 hours; ampicillin-sulfabactam 3 g every 6 hours; or ertapenem 1 g as a single dose.

Osteogenesis Imperfecta

Recurrent fractures, blue sclera, thin skin, hyperextensible ligaments, opalescent teeth, Type 1 collagen

Most foreign bodies in the GI tract will pass through. Large or irregular shaped objects may become lodged; sharp objects such as pins may cause mucosal tearing. What is recommended with caustic objects such as batteries?

Removal by esophagogastroscopy is recommended for caustic (ex. Batteries), sharp or lodged objects.

Infant presents with normal anion gap acidosis and failure to thrive. What 2 etiologies could this be?

Renal or GI causes of acidosis (most likely).

Diaphragmatic hernia

Resp distress, cyanosis, scaphoid abdomen, decreased breath sounds, often on left side

A ________ is a cell that remains after the nucleus is lost from an orthochromic erythroblast. Contains RNA and other cellular remnants that stain blue with methylene blue stain. Normal range = 1% - 2% or an absolute count of 50K to 60K.

Reticulocyte

Rogaine (minoxidil) is a treatment for androgenetic alopecia in women. Though effective, it is associated with a risk of hypertrichosis (abnormal amount of hair growth) of the face. What can you use to augment this effect?

Retin A

This is a complication of influenza characterized by fatty liver with encephalopathy. Rapidly progressive w/ a 30% fatality rate and may develop 2 or 3 weeks after onsent of influenza A or varicella infection, esp. if aspirin is ingested. Peak age is 5-14 years; rarely occurs in pts > 18.

Reye Syndrome

vomiting, lethargy, jaundice, seizures, hypoglycemia, increased liver enzymes and ammonia levels, prolonged prothrombin itme and changes in mental status are the clinical manifestations of this syndrome that occurs with Influenza with use of aspirin.

Reye Syndrome

Describe how the crystals would look like in someone with hyperparathyroidism and joint pain.

Rhomboid crystals with positive birefringence.

Infleunza: Nucleoside analogs are Active against influenza A & B. What is an exampple?

Ribavirin

What are the clinical manifestations of vitamin D deficiency?

Rickets, Osteomalacia, Biochemical changes

What are the risk factors for chronic thrush in infants?

Risk factors for chronic thrush: hypoparathyroidism, Addisons disease, hypothyroidism, DM

Rubella VS. Measles

Rubella in children (mild): 1. low-grade fever 2. conjunctivits, coryza, cervical lymphadenopathy, Forscheimer spots (patchy erythema on soft palate) 3. Cephalocaudal spread of blanching, erythematous maculopapular rash VS. in measles (more severe): 1. high fevers up to 104F 2. rash spreads more gradually and appears darker 3. Koplik spots in oral mucosa

Encapsulated organisms

S. pneumoniae *most common*, HIB, E. coli, S. aureus, Salmonella, Klebsiella, Pseudomonas,

Severe blistering mucocutaneous syndrome, involving at least 2 mucous membranes. More common in young adults and children. Associated w/ mycoplasma pneumonia infection and durgs such as phenytoin, phenobarbital, sulfonamides and aminopenicillins.

SJS

acutely ill have erythematous papules, dusky-appearing vesicles and target lesion typically noted on trunk anf face. Oral, genital and perianal mucosa develop bullae and erosions. Lesions may become eroded and secondary infections occur. Pts often note skin to be tender and burning. Diagnosis?

SJS

T or F. Exposure to sunlight is essential for cutaneous vitamin D synthesis. During spring, summer, fall, 10-15 min of sun exposure between 10am-3pm is sufficient in light skinned, but most Asian Indians require 3x as much, and African Americans may require up to 6-10x as light skinned individual.

T

In constipation, do not use this medication in patients with renal failure due to hypermagnesemia.

Saline laxatives magnesium hydroxide - watch use in patients with renal failure due to hypermagnesemia.

Rose spots. Associated with what infection?

Salmonella poisoning

Slipped Capital Femoral Epiphysis

Salter-Harris Type 1, often in obese, black, males

Deficiency of Vitamin C (Ascorbic Acid) results in...

Scurvy

How does Lennox-Gastaut syndrome present?

Seizures in someone < 7 years old and has coexisting intellectual disability.

Define bullous myringitis.

Serous liquid filled blisters on the tympanic membrane.

Adenosine deaminase deficiency

Severe Combined Immunodeficiency, disruption in DNA synthesis, both B-cells and T-cells are decreased, bony abnormalities

What disease is defined by an adenosine deaminase deficiency?

Severe combined immunodeficiency.

Hemolytic Uremic Syndrome

Severe thrombocytopenia, (purpura, GI bleed, epistaxis, hematuria, hemoptysis), microangiopathic hemolytic anema, renal failure,

Sally Mae's mother is in your clinic telling you that her 2 year old daughter had been pulling at her ear for the last week and hadn't been hearing as well. Since yesterday, her pain has abruptly *decreased*. What most likely happened?

She had acute OM, and her TM ruptured which lead to otorhea and abruptly decreased pain

Wolfe Parkinson White Syndrome

Short PR, slurred Delta wave, widened QRS

What is the treatment for uncomplicated cystitis?

Short course of antibx such as Bactrim, Macrobid of Ciprofloxacin. Hot sitz baths or phenazopyridine (pyridum) provide relief for burning...pyridium will turn urine orange.

What lab tests should be done in jaundice?

Should include prenatal and maternal blood type, Rh and antibody testing. Baby's blood type should be done if mother is type O or Rh negative with testing. ---Direct and indirect bili levels should be obtained. CBC, retic count, and blood smear should be considered.

Stroke like symptoms in a black 3 year old. Diagnosis?

Sickle Cell Anemia.

Lead toxicity causes this kind of microcytic anemia.

Sideroblastic

For infants and toddlers what is the approach to obtaining a urine culture?

Since they are using a diaper to urinate, do a straight catheterization.

Parvovirus B19

Slapped-cheek appearance with lacy reticular rash

Displacement of the femoral head through the physis (growth plate). Femoral head is typically displaced medially and posteriorly relative to femoral neck. More common in blacks than whites. What is this called?

Slipped Capital femoral epiphysis

Congenital Hypothyroidism

Sluggishness, constipation, macroglossia, umbilical hernia, hypothermia, enlarged fontanelles, jaundice, coarse brittle hair, most common thyroid dysgenesis

Edwards Syndrome Trisomy 18

Small palpebral fissures, low set ears, low birth weight, rocker-bottom feet, cleft lip, hypotonia, clenched hands, VSD

Direct inguinal hernias are medial and inferior to the deep epigastric vessels and do not go through the inguinal canal. T or F?

T

Lactase increases late in gestation so premature infants born at 28-32 weeks have reduced lactase activity. T or F.

T

Presentation of croup?

Someone less than 3 years of age with hoarseness, barking cough and varying degrees of respiratory distress.

Scarlet Fever

Sore throat, swollen tonsils, strawberry tongue, sandpaper rash, perioral sparing, non-pruritic, non-painful, peeling, skin folds

Gastroenteritis is treated symptomatically. What can be used for severely malnourished patients?

Specific treatment with TMP-SMX, ampicillin or ciprofloxacin is required for severely ill or malnourished patients,

Niemann-Pick Disease

Sphingomyelinase deficiency, loss of motor milestones 2-6 months, red macula, hepatomegaly, areflexia,

Chronic ITP that is refractory to IVIG and glucocorticoid. Treatment?

Splenectomy.

What is the treatment for ASD?

Spontaneous closure likely in 1st year of life. If symptomatic, close defect ASAP. If asymptomatic, most patients undergo closure b/t ages 2 and 4.

How do you manage a vesicourethral reflux?

Spontaneous resolution is common in young children but less common as puberty approaches Severe reflux is unlikely to resolve. Sterile reflux, in general does not result in reflux nephropathy Long term antibiotic prophylaxis in children is safe *Surgery to correct VUR is highly successful. *

Staging of disease in hodgkins lymphoma is very important for treatment and prognosis. What shoul staging include?

Staging should include lymph node bx, CXR, CT of chest and abdomen, CBC, ESR and chem profile including LFTs.

What is the most common cause of pneumonia in a pediatric population of CF patients?

Staph Aureus?? I think pseudomonas

Most common cause of osteomyelitis in children is what?

Staph aureus (via blood stream transmission).

Most common cause of an acute unilateral lymphadenitis?

Staph aureus.

prodrome of fever, irritability and skin tenderness followed by superficial flaccid blisters and a positive Nikolsky sign in a 7 year old. Diagnosis?

Staphylococcal scalded skin syndrome.

How does jaundice progress?

Starts at head and moves down. Sclera-1st to get jaundice, this along with jaundice oral mucosa helps distinguished in darkly pigmented infants. Splenomegaly may be present in hereditary spherocytosis.

Oral Candidiasis, T or F: Candida Albicans is a normal flora in many areas of the body and only causes disease when immunocompromised.

T

T or F. A normal RBC is a biconcave disk with a lifespan of 120 days.

T

JJ is a 7yr3mo female who presents with a fever, chills and body aches. Integumentary exam reveals a painful purplish red rash that spreads and blisters. What syndrome is she suffering from?

Steven Johnson Syndrome

This is a vesiculobullous disease of skin, mouth, eyes, genitalia. Ulcerative stomatitis leads to hemorrhagic crusting.

Steven Johnson Syndrome

These laxatives are useful in acute constipation. They work within 12 hours with oral administration and 60 minutes with rectal administration.

Stimulant laxatives: senna or bisacodyl

Misalignment of the visual axis due to ocular muscle weakness or imbalance.

Strabisumus, present in 4-5% of children

NOTE:

Stranger anxiety is normal in children until age 3

What is the expected language development of a 24 month old child? How about a 36 month old?

Stranger should be expected to understand half the words of what a child says. Child should be able to speak 150-300 words (2 years = 2 zeros, in the hundreds) 3 year old - 1000 words with strangers understanding 75% (3 for 3 zeros)

MC cause of pneumonia beyond the newborn period and up to age 5 to 6?

Strep Pneumo

What is a serious complication for impetigo, a skin disorder presenting with either a nonbullous or bullous honey colored crust?

Streptococcal glomerulonephritis

Lungs sounds

Stridor - squeaky, whistlelike sound that results from turbulence between the supraglottis and trachae => differential: croup, foreign body aspiration, anatomic abnormalities such as laryngomalacia or vascular rings Stertor - snoring type sound that originates form turbulence in the naso or oropharynx Wheezing, rhonchi, rales are banormal sounds due to narrowing or congestion of the bronchioles

Croup

Stridor, subglottic swelling, seal-like cough, classic features of laryngotracheomalacia

Mental retardation, seizures, visual impairment and port-wine stain over territory of trigeminal nerve. Diagnosis?

Sturge-Weber

Osteosarcoma

Sunburst appearance, periosteal elevation (Codman's triangle), palpable tender mass, negative B-symptoms, metaphysis of long bones, elevated ALP and LDH from bone turnover

What is the most common benign vascular tumor in children?

Superficial infantile hemangioma, aka strawberry hemangiomas. Demarcated plaques that blanch with pressure Note: associate strawberry short-cake character with children. Do not pick cherry hemangioma for lesions on kids.

Treatment for Hemolytic Uremic Syndrome?

Supportive with plasmapheresis, dialysis if necessary, and steroids

Treatment of HUS?

Supportive, steroids and plasmapheresis or dialysis. Note: no need for antibiotics.

Treatment for HUS?

Supportive. Plasmapheresis, dialysis if necessary and steroids. Do not bother with antibiotics.

What are the surgical and nonsurgical treatment options for strabismus?

Surgical - resection and recession are used to strengthen and weaken the appropriate muscles. Nonsurgical - Occlusion therapy to treat amblyopia (the good eye is covered to stimulate the amblyopic eye)

What is the treatment for pyloric stenosis?

Surgical pyloromyotomy is curative.

What are the tests for CF in an undiagnosed pt?

Sweat chloride. Genotyping. fecal elastase.

How do you diagnose hypercalcemia?

Sx occur with levels > 12 mg/dl. EKG = shortened QT interval

spiking fevers 102.2 - 104, myalgias, polyarthalgias and a typical salmon-pink maculopapular rash appearing in the evening and w/ fever. The rash may be elicited by scratching the skin in susceptible areas (Koebner phenomenon). There are minimal articular findings but h/s-megaly, lymphadenopathy, leukocytosis, pericarditis or myocarditis may occur. What type of Juvenule r. Arthritis is this?

Systemic (Still Disease: JIA)

What is the murmur of ASD?

Systolic ejection murmur at 2nd LICS; early to middle systolic rumble. Patients also present w/Failure to thrive, fatigability, RV heave and wide fixed split S2.

Because of high mortality and morbidity with neonatal HSV infection, C-section should be used to prevent transmission to infant from actively infected mother. T or F.

T

What is the treatment for mastoiditis? What are the complications of mastoiditis?

TX: Same as OM but must treat for 3-4 weeks. Admit for IV antibiotics, antibiotic course for 21 days Acute: Cefotaxime or Ceftriaxone vs Chronic: Ticarcillin-Clavulanate Mastoidectomy may be needed in severe diseases. Complications = brain abscess, septic lateral sinus thrombosis.

Club foot

Talipes equinovarus.

Ebstein's anomaly is associated with what ECG findings?

Tall P waves and right axis deviation.

Best test for Meckel's diverticulum? Treatment?

Technetium-99m pertechnetate scan. Surgery to resolve symptoms and remove chances of intussusception.

What do you treat tinea corporis (ring worm) with?

Terbinafine

The MC cause of testicular pain in pts > 12 is...

Testicular torsion

How do you diagnose cryptorchidism?

Testosterone level after HCF stimulation to confirm presence or absence of abdominal testes. US or CT can be utilized.

_________ - is a blood disorder in which the body makes an abnormal form of hemoglobin that results in excessive destruction of red blood cells, which leads to anemia. It is a microcytic anemia.

Thalassemia

What are the Microcytic anemias (MCVs < 80), most common anemia type? Acronym is TICS.

Thalassemia Iron Deficiency Anemia Chronic Disease Anemia Sideroblastic anemia (lead poisoning)

The parents of a 6-year-old girl inquire as to her risk of developing type 1 diabetes mellitus. Which of the following factors, if present, places her at greatest risk of developing the disease? A. a sibling with type 1 diabetes mellitus B. body mass index>90th percentile C. early introduction of cereals to diet D. formula feeding

The answer is A A. A sibling with type 1 diabetes places a child at 15 times the normal risk for development of type 1 diabetes. B. A high body mass index may place a child at higher risk for type 2 diabetes. C. Introduction of cereals before 4 months has been suggested to increase the risk of autoimmunity. D. Milk-based protein has been shown to increase susceptibility for type 1 diabetes in animal models, but has yet to be shown in human studies.

A young child presents with right ear pain and pressure, decreased hearing acuity, and a mild fever for 2 days. Physical exam reveals a reddened tympanic membrane with decreased mobility. What is the most likely diagnosis? A. acute otitis media B. barotrauma C. mastoiditis D. osteoma

The answer is A A. Acute otitis media is most common in infants and children and presents with the symptoms described. B. Patients with poor eustachian tube function may be unable to equalize barometric stress on the middle ear (i.e., flying, diving, rapid altitude changes), resulting in barotrauma. They present with severe ear pain and hearing loss. C. Acute mastoiditis may appear acutely or after several weeks of inadequately treated acute otitis media. Presenting symptoms include postauricular pain and redness with a spiking fever. D. Bony overgrowths in the ear canal can be an incidental finding. Solitary osteomas have no clinical significance unless they cause obstruction or infection.

A 15-year-old female complains of abdominal pain that started 12 hours ago after her birthday party where she ate pizza, soda, and birthday cake. The patient has no appetite this morning and she feels warm. LMP was 3 weeks ago. Abdominal exam reveals right lower quadrant tenderness without rebound. In addition to laboratory studies, what is the imaging study of choice? A. abdominal computed tomography B. endoscopic retrograde cholangiopancreatography C. flat plat of abdomen, kidneys, ureter, bladder D. pelvic ultrasonography

The answer is A A. An abdominal CT is the imaging modality of choice for evaluation of suspected appendicitis. CT is more sensitive than ultrasonography. B. ERCP is an imaging modality for evaluation of the pancreas and related structures. C. A flat plate is not specific to appendicitis. It may be beneficial if ileus or small bowel obstruction is in the differential. Gas-filled loops of small and large intestines may be seen with ileus. Air fluid levels may also be detected. D. Ectopic pregnancy is in the differential for this patient. If the patient has a positive bHCG, then pelvic ultrasonography would be appropriate. However, the clinical picture indicates appendicitis.

A mother brings her 5-year-old daughter to the office because of a fever and a fiery red rash on both cheeks for 2 days. The child complains of sore throat and appears tired. What can you tell her about the most likely course of this illness? A. The rash will spread distally and include central clearing areas resembling a reticulated or lacy appearance. B. The cheek erythema will progress to vesicles before crusting over and healing. C. She will likely develop red macules on the buccal mucosa and tongue. D. A symmetric polyarthritis will likely develop and last 2 to 3 weeks.

The answer is A A. Erythema infectiosum (fifth disease) is characterized by a "slapped cheeks" appearance followed by a lacy rash on the trunk, neck, and extremities 1 to 4 days later. Mild symptoms of fever, headache, coryza, and malaise often accompany the facial rash. B. Vesicles are very rare and atypical in erythema infectiosum. C. Mucosal lesions are uncommon in erythema infectiosum. D. Arthralgias are uncommon in children with erythema infectiosum; however, they are quite common in adults (especially women) who contract this viral infection caused by parvovirus B19.

A mother brings in her 2-year-old son stating he has been crying for 2 hours and refuses to move his right arm. He sits with his elbow fully pronated and has tenderness over the radial head. X-rays appear to be negative. What is the recommended treatment for the suspected diagnosis? A. immediate reduction B. long-arm cast C. referral to an orthopaedic surgeon D. sling

The answer is A A. Nursemaid's elbow (subluxation of the radial head) is treated with immediate reduction by supinating the elbow and moving it from full flexion to full extension. B. A long-arm cast is appropriate treatment for forearm fracture or fracture around the elbow. C. Nursemaid's elbow is best taken care of immediately; it should not be left dislocated or subluxed for a long period of time. D. A sling can be used for a brief time after the reduction for comfort.

A 2-year-old is brought to the office for follow-up after a battery of tests including a hearing test, EEG, and CT scan, all of which are normal. The child's parents describe delayed speech development. The child has age-appropriate motor skills; however, his language and social development are severely delayed. The parents also note that the child has limited play skills and seems to be particularly sensitive to the environment (noise and motion). Which of the following diagnostic statements is characteristic of the most likely diagnosis? A. age of onset is generally before age 3 B. more than half of the patients also have a seizure disorder C. most patients will display hallucinations and delusions D. level of intelligence is normal in most patients

The answer is A A. The onset of autism is before 36 months. B. Autism is associated with seizures in less than one-third of patients. C. Autism is not associated with hallucinations or delusions. It is associated with absence of speech, stereotyped phrases, and failure to develop relatedness. D. Autism is almost always associated with subnormal intelligence; intelligence is severely impaired in 70% of patients.

An 8-month-old male infant has had rhinorrhea, sneezing, cough, and low-grade fever for 2 days. On exam, there is nasal flaring, tachypnea, retractions, and wheezes. Chest radiography reveals air trapping and peribronchial thickening. What is the most likely diagnosis? A. acute bronchiolitis B. asthma exacerbation C. cystic fibrosis D. viral pneumonia

The answer is A A. These are classic clinical symptoms and signs of acute bronchiolitis. Diagnosis is typically based upon clinical findings. B. Asthma typically has a recurrent pattern. C. Cystic fibrosis may be associated with poor growth, recurrent pulmonary infections, chronic diarrhea, or a family history of the disease. D. Pneumonia is usually associated with an infiltrate demonstrated on chest radiography.

A 12-year-old girl is found to have elevated blood pressure readings on three separate occasions. Her blood pressure is normal in her legs but femoral pulsations are weak. ECG demonstrates left ventricular hypertrophy. What is the most likely cause of her hypertension? A. coarctation of the aorta B. pheochromocytoma C. tetralogy of Fallot D. ventricular septal defect

The answer is A A. This is the classic description of coarctation of the aorta as a cause of secondary hypertension. B. Pheochromocytoma is a cause of secondary hypertension, but presents with flushing, headaches, and fluctuating blood pressures. C. Tetralogy of Fallot includes over-riding aorta, pulmonic stenosis, right ventricular hypertrophy, and ventricular septal defect. It is not associated with secondary hypertension. Untreated older children risk pulmonary valve problems and congestive heart failure. After surgical correction, older children are at risk for endocarditis, arrhythmias, and coronary artery disease. D. Large ventricular septal defects may cause arrhythmias but are not associated with hypertension.

A 6-year-old presents with intermittent episodes of wheezing. Exam reveals expiratory wheezing but no rales or rhonchi. She is developing well and is cooperative with the exam. She responds well to an albuterol treatment. What is the most effective method to monitor symptoms at home? A. peak expiratory flow rate B. respiratory rate C. pulse oximetry D. blood pressure

The answer is A. A. Peak expiratory flow rate can be monitored at home using an inexpensive handheld device. B. Respiratory rate is unreliable and too subjective for monitoring asthma. C. Pulse oximetry may be normal with mild asthma. D. The only blood pressure abnormality related to asthma is pulsus paradoxus, which may occur with severe asthma.

A 5-year-old male presents with crusting facial lesions present for 3 days. The mother reports that prior to the development of the facial lesions, her son was scratching at insect bites. Exam reveals confluent erosions with honey-colored crusts below the left nares and cheek. Temperature is 99.8F. Which of the following is the most appropriate treatment? A. acyclovir B. cephalexin C. doxycycline D. hydrocortisone

The answer is B A. Acyclovir is an antiviral beneficial in the treatment of herpes infections. Herpes infections usually present as grouped vesicles that eventually erode and crust. B. Cephalexin provides appropriate coverage for impetigo caused by Staphylococcus and Streptococcus species. Systemic antibiotics are recommended with widespread or complicated infections or if systemic symptoms, such as fever and malaise, are present. C. Doxycycline is a reasonable alternative for the treatment of impetigo but is contraindicated for use in children. D. Hydrocortisone is contraindicated in infectious lesions.

Which of the following is used for treating wilsons disease a. Penicillamine b. EDTA c. BAL d. All of the above

a. Penicillamine

A 12-year-old presents with acute onset of dyspnea and wheezing. He reports similar episodes over the past few months. He is not on any medications. Physical exam reveals a respiratory rate of 30, normal blood pressure, no rash, clear throat, and diffuse expiratory wheezing. What is the most likely diagnosis? A. anaphylaxis B. asthma C. cystic fibrosis D. toxoplasmosis

The answer is B A. Anaphylaxis will manifest with urticaria, hypotension, and nausea as well as dyspnea. B. This is a typical presentation of asthma. C. Patients with cystic fibrosis have chronic productive cough and clubbing. D. Acute toxoplasmosis presents with fever, dyspnea, and nonproductive cough. It is more common in immuno-compromised patients.

A 15-year-old male with a history of sickle cell anemia presents with increasing lethargy and generalized pain. Vitals: T 98.2; P 98; R 20; BP 116/74. His lips are slightly dry; he has mildly decreased turgor and 3-second capillary refill. No murmurs, abnormal lung sounds, or abdominal pain are noted on physical exam. Reticulocyte count is increased by twofold; hematocrit and hemoglobin are slightly below his baseline. What is the next best step in management? A. blood transfusion B. hydration and analgesics C. hydroxyurea D. IV broad-spectrum antibiotics

The answer is B A. Blood transfusions are necessary when the hemoglobin or hematocrit has dropped significantly or the patient is short of breath. It is not necessary in all cases of acute pain in patients with sickle cell disease. B. The patient is mildly dehydrated, which can occur during a period of acute sickling or during other complications associated with sickle cell disease. Many times, intravenous or oral fluids along with appropriate pain control are sufficient to correct the problem and bring relief. C. Hydroxyurea is used to prevent or reduce the number of episodes of sickling that can lead to more serious complications of sickle cell disease. D. In this case, CBC, CXR, and urinalysis may be required to look for an underlying infectious cause but, until additional tests are run, broad-spectrum antibiotics would be out of place.

A child exhibits short staring spells at school which are sometimes accompanied by loss of postural tone. Episodes last less than a minute. Electroencephalography reveals bilateral 3-Hz spike-and-wave pattern. What is the medication of choice? A. carbamazepine B. ethosuximide C. phenobarbital D. phenytoin

The answer is B A. Carbamazepine, phenobarbital, and phenytoin are indi- cated for tonic-clonic (grand mal) or partial focal seizures. B. Absence, or petit mal, seizures are best treated with either ethosuximide or valproic acid. Clonazepam is also effective but can be habit forming. C. See A. D. See A.

A 12-year-old male woke suddenly this morning with severe scrotal pain and edema of the scrotal sac that has continued for 2 hours. He has had similar pain before, but it subsided without intervention. Physical examination reveals a tender, swollen, retracted testis. A routine urine exam is normal. Which of the following is the most appropriate next step? A. scrotal support and ice packs B. immediate surgical intervention C. initiation of antibiotic therapy D. Tc99 pertechnetate scan

The answer is B A. Ice packs cause further vasoconstriction, thereby increasing the risk of infarction. B. Testicular torsion is most common in adolescent boys. It is a surgical emergency. If torsion is complete, a testis can be infarcted in 4 to 6 hours. C. Antibiotic therapy has no place in the treatment of torsion. D. Delaying surgical intervention beyond 4 to 6 hours can lead to testicular infarction and subsequent infertility. Tc99 pertechnetate scans will confirm the presence of torsion; however, Doppler studies are quicker, less invasive, and just as effective to confirm the diagnosis.

A 2-year-old was recently taken to a nursing home to visit a relative. A few days after the visit, the relative was diagnosed with influenza. In the last 24 hours, the child has been experiencing high fever, cough, sore throat, and myalgias. The child did not receive a flu vaccine. What treatment is warranted at this time for the child? A. amantadine (Symmetrel) B. oseltamivir (Tamiflu) C. rimantadine (Flumadine) D. zanamivir (Relenza)

The answer is B A. Amantadine and ramantidine are no longer recommended for the early treatment or prophylaxis of influenza because of the resistance that has developed to these two antivirals. B. Oseltamivir is recommended for both the treatment and prophylaxis of influenza A or B in children greater than 1 year of age and should be initiated within 48 hours of symptoms suggestive of the flu. C. See A. D. Zanamivir is not recommended in children under the age of 7 years.

A 15-year-old girl reports that she has never had menses. She is short in stature and has a webbed neck and wide-spaced nipples. Initial lab work shows a high FSH. Which of the following is best to confirm diagnosis? A. estrogen levels B. karyotype C. LH levels D. ultrasonography

The answer is B. A. Estrogen levels will be abnormally low but are not diagnostic. B. This is a classic picture of Turner's syndrome (gonadal dysgenesis), which is a frequent cause of primary amenorrhea. Karyotyping will show a 45XO pattern. C. LH levels are useful in diagnosing pseudo-hermaphroditism (XY genotype, XX phenotype) and pituitary tumors but not Turner's syndrome. D. Ultrasonography will show a gonadal streak, misshapen functionless ovarian tissue. While present in Turner's syndrome, gonadal streaks are also seen with other genetic mutations and, therefore, are not diagnostic.

A 7-year-old presents with his parents who are complaining that for the last 9 months the child has been having increasing difficulty in school. He has consistently been inattentive in school, has difficulty following directions, cannot stay on task, is easily distracted, and is often forgetful about what he is assigned to do. At home he does not listen to his parents. What is the most likely diagnosis? A. Asperger's syndrome B. attention-deficit disorder C. disruptive behavior disorder D. hyperkinetic conduct disorder

The answer is B. A. Asperger's syndrome is a pervasive developmental disorder characterized by impairment of social functioning and interactions. Patients display restricted repetitive and stereotyped patterns of behavior. B. Attention-deficit disorder is characterized by inattentive behavior in school and at home. C. Disruptive behavior disorder includes oppositional and defiant behaviors rather than inattentive behaviors. D. Hyperkinetic conduct disorder includes symptoms of hyperactivity, impulsivity, and aggression.

While working in a rural clinic, you are asked to assist with the delivery of a 29-year-old multipara. She successfully delivers a baby boy; a continuous, rough, machinery-like murmur is heard on cardiac auscultation of this newborn. What is the drug of choice to administer to the newborn? A. ampicillin B. Beta-blocker C. heparin D. indomethacin

The answer is D A. Antibiotics are not indicated in the treatment of patent ductus arteriosus. B. Beta-Blockers will slow the heart rate and, therefore, may be harmful. C. Anticoagulants are not indicated and may be harmful in the neonate. D. Indomethacin, a prostaglandin inhibitor, is routinely administered to help close a patent ductus arteriosus.

A 2-year-old boy presents after two hours of paroxysmal coughing. His mother states he had a gagging episode at lunch, just prior to the cough. Stridor is noted on exam, as well as unilateral wheezing. He has no significant past medical history. Which of the following is highest on the differential diagnosis? A. angioedema B. epiglottitis C. foreign body D. laryngotracheobronchitis

The answer is C A. Angioedema can present at any age. Symptoms have a sudden onset. The patient may exhibit stridor and a hoarse voice and may develop facial edema. It may be associated with ingestion of food causing anaphylaxis. B. Epiglottitis is most commonly seen in children 2 to 6 years old. It has a rapid onset, stridor may be present, and the patient may drool and appear toxic. C. Airway foreign bodies most commonly occur in children 1 to 3 years old. The classic triad of physical findings is sudden onset of paroxysmal coughing, wheezing, and diminished breath sounds. Stridor may also be present. The history of having choked or gagged raises this diagnosis to the top of the differential. D. Croup is most commonly seen in children 6 months to 3 years old. It has an insidious onset and is associated with upper respiratory symptoms.

A 16-year-old Hispanic female presents to the clinic complaining of severe pain in her right ear. She is on the local high school swim team and has been participating in extra practice sessions. Her ear pain is so severe that she cannot lie on her right side to sleep. On physical exam, the right tragus is tender to palpation. The right ear canal is swollen and has scant white, clumpy discharge. Culture of this discharge would most likely reveal what organism? A. Haemophilus influenzae B. Moraxella catarrhalis C. Pseudomonas aeruginosa D. Streptococcus pneumoniae

The answer is C A. The incidence of otitis media due to Haemophilus influenzae has been significantly decreased with vaccination programs. It rarely causes otitis externa. B. Moraxella catarrhalis is a common pathogen in otitis media. C. The majority of cases of otitis externa are caused by Pseudomonas aeruginosa or Proteus spp. Less common causes include Staphylococcus aureus, Staphylococcus epidermidis, Aspergillus, and Actinomyces. Topical antibiotic coverage should include antipseudomonal activity. D. Acute otitis media typically follows a viral URI. Viruses can cause the infection or predispose the patient to a secondary bacterial infection. Streptococcus pneumoniae is the most common bacterial pathogen.

A 4-year-old presents with a 2-day history of a barking cough, runny nose, and fever. Exam reveals a red pharynx with no exudate, a temperature of 100.4F, and scattered wheezes throughout both lung fields. What is the recommended treatment? A. antitussive such as dextromethorphan B. ceftriaxone IM (Rocephin) C. nebulized budesonide (Pulmicort) D. nebulized epinephrine

The answer is C A. Antitussives, decongestants, and antibiotics are not recommended for treatment of viral croup. Most croup is viral in origin. B. See A. C. Nebulized budesonide or oral dexamethasone is the recommended treatment for mild to moderate viral croup. D. Nebulized epinephrine is reserved for severe symptoms of croup such as audible stridor, cyanosis, or oxygen saturation below 92%.

A 12-year-old male has been limping for the past 2 months. Exam reveals a moderately obese young male in no acute distress who complains of pain in his left hip region. There is a limitation of internal rotation of the hip with a 2-cm left limb discrepancy as compared to the right. What is the most appropriate diagnostic test? A. AP and lateral radiographs of the left hip B. AP pelvic films C. frog leg view of both hips D. weight-bearing MRI of the left hip

The answer is C. A. See C. B. See C. C. Slipped capital femoral epiphysis (SCFE) typically occurs during an adolescent growth spurt; patients are frequently overweight. Typical presentation is with hip pain. The pain may worsen on ambulation or cause a "waddling" gait. Pain may be felt in the hip but may be referred to the thigh or knee. Radiographs of the child in the frog leg position are usually diagnostic and can be utilized to grade SCFE severity. D. See C.

A 7-year-old male presents with a history of frequent respiratory infections with recurrent pulmonary infiltrates and failure to thrive. His cough is persistent and productive of thick, green sputum. A recent sputum culture was positive for Haemophilus influenzae and Staphylococcus aureus. Which of the following laboratory results is pathognomonic for his condition? A. abdominal flat plate showing small intestinal air-fluid levels and small colon B. chest radiography revealing hyperinflation and small airway obstruction C. elevated sweat chloride levels D. irreversible changes in FVC and FEV1

The answer is C. A. The abdominal flat plate findings described are consistent with meconium ileus, which may be seen in newborns with cystic fibrosis. B. Hyperinflation can also be seen in asthmatics. C. Elevated sweat chloride values are the key to diagnosis of cystic fibrosis. The values for the chloride (and sodium) concentrations in sweat vary with age, but typically a chloride concentration of greater than 70 mEq/L discriminates between patients with cystic fibrosis and those with other lung diseases. D. Pulmonary function testing in patients with cystic fibrosis is highly variable, and abnormal findings are generally irreversible.

A 14-year-old male presents with anterior knee pain and swelling that gets worse after soccer games. The patient reports no trauma to the knee. On exam, knee range of motion is normal but the patient is tender to palpation over the tibial tubercle. What is the most likely diagnosis? A. chondromalacia patella B. medial collateral ligament tear C. Osgood-Schlatter disease D. prepatellar bursitis

The answer is C. A. Chondromalacia patella usually presents with symmetric anterior knee pain that is worse after sitting and going up stairs. B. Medial collateral ligament tear is usually caused by abnormal force applied to the knee and presents with pain and knee instability. C. Osgood-Schlatter disease presents with anterior knee pain and swelling over the tibial tubercle that is associated with activity. It is caused by fragmentation of the tibial tubercle apophysis from chronic tensile stress. D. Prepatellar bursitis in a child is usually caused by trauma that allows hematogenous spread of bacteria into the bursa. It presents with a swollen area anterior to the knee joint and does not interfere with knee range of motion.

A 14-year-old male presents with fever, myalgias, arthralgias, carditis, and polyarthritis. ASO titer is positive. What is the treatment of choice to relieve his fever and arthralgias? A. acetaminophen B. codeine C. ibuprofen D. salicylates

The answer is D A. Acetaminophen or ibuprofen would reduce the fever and pain, but they are not the drugs of choice in rheumatic fever. B. Codeine may be used to relieve severe pain but should be reserved for such cases. C. See A. D. Aspirin is the drug of choice in rheumatic fever; it has consistently shown greater anti-inflammatory properties over other NSAIDs.

A 16-year-old presents to the emergency department with productive cough, hemoptysis, and pleuritic chest pain. Chest x-ray reveals dilated and thickened bronchi that appear as tram tracks and ringlike markings. Which of the following is most likely in this patient's history? A. exposure to asbestos B. family history of sarcoidosis C. history of hyaline membrane disease D. recurrent purulent pulmonary infections

The answer is D A. Asbestos exposure is linked to mesothelioma and pleuritic plaques. B. Sarcoidosis may cause dyspnea but not a productive cough or hemoptysis. C. A history of hyaline membrane disease does not predispose to bronchiectasis. D. Bronchiectasis notoriously causes recurrent pulmonary infections requiring antibiotics. Cystic fibrosis is a common underlying cause.

An 18-year-old male presents for further evaluation of elevated blood pressure, which was recently detected at a health fair. Exam today reveals a blood pressure of 156/99 with an associated left infraclavicular systolic murmur and a prominent suprasternal notch pulsation. What is the most appropriate next step in the evaluation of this patient? A. cardiac catheterization with pressure gradients B. magnetic resonance imaging of the chest C. signal-averaged electrocardiography D. upper and lower extremity blood pressures

The answer is D A. Catheterization may be indicated after MRI or CT to definitively measure the gradients in planning for percutaneous stenting. B. MRI or CT will illustrate the anatomy of the coarctation prior to surgical repair. C. ECG will reveal left ventricular hypertrophy, but this is not diagnostic. D. Coarctation of the aorta commonly presents as systemic hypertension. Elevated hypertension reading in the arms with normotensive lower extremities is characteristic. Prominent infraclavicular murmurs or late systolic ejection murmurs located at the base of the heart and heard best over the spinous processes may also be found.

A 4-year-old girl presents for a routine checkup. She is new to the practice. Physical examination reveals a harsh, holosystolic grade III/VI cardiac murmur at the left sternal border with wide radiation and fixed split S2. There is no change with position or respiration. What is the most likely diagnosis? A. innocent murmur B. aortic regurgitation C. patent ductus arteriosus D. ventricular septal defect

The answer is D A. Innocent murmurs are usually soft (grade I or II), short in duration, and systolic. If a ventricular septal defect is misdiagnosed as an innocent murmur and not corrected, the sequelae of the increased pulmonary artery pressure from the ventricular septal defect can lead to irreversible pul- monary artery hypertension (Eisenmenger's syndrome), resulting in premature death. B. The murmur of aortic regurgitation is a diastolic murmur, not a holosystolic murmur. C. The murmur of a patent ductus arteriosus is continuous, not holosystolic. The murmur accentuates in late systole. D. Ventricular septal defects are typically harsh holosystolic murmurs heard best at the left sternal border. The fixed split S2 has a louder pulmonary component.

A 15-year-old boy presents with a 2-month history of bilateral knee pain. He runs track and attends practice daily. He describes the pain as dull and aching over the anterior knee, increasing with activity. He did not have any specific injury or limp but has noticed some swelling under the knees. On exam, he is afebrile. He has slight swelling and tenderness bilaterally just below the knees. There is full range of motion and negative special tests of the knee. Radiographs are normal. What is the best management at this time? A. knee immobility for 6 to 8 weeks B. referral to physical therapy C. routine referral to orthopaedic surgery D. self-limited activity, ice, and NSAIDs

The answer is D A. Knee immobility may be considered in severe cases where symptomatic care and rest do not control the symptoms. B. Physical therapy can be considered for assistance with isometric and flexibility exercises in moderate to severe cases that are slow to resolve. C. Removal of ossicles from the tibial tuberosity is rarely necessary. D. Osgood-Schlatter disease is characterized by pain over the tibial tubercle in a growing child. It occurs during late childhood or adolescence, especially in athletes, and is likely due to repetitive tensile microtrauma. It is more common in males. Rest, restriction of activities, and, occasionally, a knee immobilizer may be necessary, combined with an isometric and flexibility exercise program.

A 14-year-old female presents for a gymnastics camp physical. She is 60 inches tall and weighs 82 lbs and is Tanner 1 in sexual development. She describes her caloric intake as less than 1,000 kcal/day with hours of weight training and gymnastics practice. Which of the following laboratory studies is indicated? A. insulinlike growth factor 1 B. serum cortisol level C. serum gastrin level D. thyroid function tests

The answer is D A. See D. B. See D. C. See D. D. This patient is at high risk for anorexia nervosa. Her initial workup includes a complete chemistry panel, CBC, ESR, urinalysis, ECG, and thyroid function tests.

A 14-year-old male presents with sore throat and fever, but denies cough. Exam reveals elevated temperature, tender anterior cervical adenopathy, and pharyngeal exudate. What is the most likely diagnosis? A. epiglottitis B. mononucleosis C. peritonsillar abscess D. strep pharyngitis

The answer is D A. Epiglottitis should be suspected when a patient presents with a rapidly developing sore throat or when pain with swallowing is out of proportion to minimal findings on physical exam. It is less common today in children secondary to Hib (Haemophilus influenzae type b) vaccination programs. B. Mononucleosis is suggested by prominent adenopathy and a shaggy, white-gray tonsillar exudate, which may extend into the nasopharynx, especially in young adults. C. Peritonsillar abscess presents with severe sore throat, pain on swallowing, trismus ("lockjaw"), medial deviation of the soft palate, and a muffled ("hot potato") voice. D. These clinical features (fever, tender anterior cervical adenopathy, lack of cough, and pharyngotonsillar exudate) strongly suggest group A B-hemolytic streptococcus.

An 8-year-old boy is being evaluated for short stature. His mother reports that he has gained weight over the last year, but he has little or no energy, sleeps more than normal, and complains of being cold all the time. His growth curve demonstrates that he has fallen from the 50th percentile to the 5th percentile for height, but his weight has increased from the 50th percentile to the 90th percentile for weight. On physical exam, he is obese, has immature facies, thin hair, and slow reflexes. What is the most likely cause of this child's symptoms? A. acromegaly B. Cushing's syndrome C. dwarfism D. hypothyroidism

The answer is D. A. Acromegaly presents in adults with extreme growth, headaches, visual field defects, weakness, soft doughy hands, and amenorrhea in females. It is due to the lack of suppression of growth hormone. Excess growth hormone in children causes gigantism. B. Cushing's syndrome is due to increased production of cortisol. Clinical symptoms typically include central obesity, generalized weight gain, rounding of the face, and a dorsocervical fat pad along with other symptoms of glucocorticoid excess. C. Dwarfism (achondroplasia) is an autosomal recessive disorder with disproportionately short arms and legs, frontal bossing, and midfacial hypoplasia. It is often diagnosed in utero via prenatal ultrasound. D. Deceleration of growth is usually the first clinical manifestation of hypothyroidism in children. Myxedematous changes of the skin, constipation, cold intolerance, decreased energy, and an increased need for sleep develop insidiously. Reflexes may be slowed, especially in the ankles.

A 3year old female presents with rough, erythematous, pruritic lesions on her cheeks and antecubital fossae. The family history is significant for asthma. Which of the following measures would be beneficial in the management of her condition? A. avoid moisturizers B. dehumidifier in bedroom C. frequent bathing D. wear cotton clothing

The answer is D. A. Skin should be kept well moisturized in atopic dermatitis with regular applications of topical emollients. B. Low humidity can exacerbate the symptoms of atopic dermatitis, which is why the condition often worsens in winter. C. Frequent bathing should be avoided in persons with atopic dermatitis. Soap should be used sparingly to avoid irritating the skin. D. Cotton clothing is preferred for persons with atopic dermatitis. Wools and acrylics should be avoided, as they can exacerbate the condition.

A neonate is found to have right ventricular hypertrophy, right ventricular outflow obstruction, and an over-riding aorta via cardiac imaging. What additional finding would you expect to see in this child? A. atrial septal defect B. discrepancy in upper and lower extremity blood pressure C. left bundle branch block D. ventricular septal defect

The answer is D. A. The defect in tetralogy of Fallot is in the ventricle, not the atrium. B. Dissimilar blood pressure readings are pathognomonic of coarctation of the aorta. C. Although ECG findings are not of great significance, usually right bundle branch block occurs in tetralogy of Fallot. D. The finding of ventricular septal defect completes the diagnostic criteria for tetralogy of Fallot.

Bronze baby syndrome occurs due to a. Photochemotherapy b. Wilson's disease c. Hemochromatosis d. Aortic regurgitation

a. Photochemotherapy

Why are there no symptoms of microcephaly and microphthalmia with herpes simplex infection of children.

The infection is acquired through the passage out of the vagina no transmitted in utero.

Why is an MRI of a young child not the best first imaging for MSK complaints?

The need of sedation and it is expensive.

When does the risk of infection begin w/ neutropenic pts?

The risk of infection begins to increase at an ANC below 1000/microL

This is a progression of SJS to full thickness skin detachment.

Toxic epidermal necrosis

Pathogenesis of idiopathic gonadotropin-dependent precocious puberty? Treatment and purpose?

This is a central hypothalamic-pituitary-gonadal axis disorder of excess LH, it is the MCC of central precocious puberty. GnRH agonist therapy to prevent premature epiphyseal plate fusion and maximize adult height potential. Like lupron (lath) leuproLide acetate Or hisretlin

Amikacin, ceftazidime and ciprofloxacin all treat what bacteria??

They treat pseudomonas.

Patient presents with round or oval, scaly, hypopigmented macules. What is the likely diagnosis?

Tinea versicolor

EBV

Tonsillar exudates, diffuse cervical lymphadenopathy, fever, rash with amoxicillin/ampicillin, avoid sports >3 weeks

What is the treatment for diaper dermatitis, commonly caused by candida?

Topical Nystatin, Lotrimin, Diflucan (Fluconazole) PO for 2 weeks Keep area clean, dry and open to air!

What is the treatment for oral candidiasis? What if systemic treatment is needed?

Topical antifungal agents (nyastatin or clotrimazole). If systemic treatment is required = fluconazole or itraconazole. Dentures must be removed and properly cleaned.

Patient presents with gradual onset of testicular pain and scrotal erythema. What has likely happened?

Torsion of the APPENDIX of the testes

TM perforation usu. occurs after penetrating noise or trauma. Where in the TM does it occur?

Typically occurs in the pars tensa, anteriorly or inferiorly.

How does an indirect inguinal hernia form in children?

Typically present in the first year of life but may not appear until pt is older, when the increased intra-abdominal pressure & dilated inguinal ring allow contents to enter cavity.

What imaging study is diagnostic for pyloric stenosis?

US

POD 3-7, Subacute

UTI, surgical site infection, septic thrombophlebitis, anastamotic leakage

What imaging is best for diagnosing intussusception?

Ultrasonography.

This type of hernia is easily reduced, and rarely become incarcerated or strangulated, or even more rarely rupture.

Umbilical hernia

What causes an umbilical hernia?

Umbilical ring not closing

Treatment for peritonsilar abscess

Unasyn (ampicillin-sulbactam): better with gm- bugs Clindamycin: gets MRSA Vancomycin if not getting better b/c it covers resistant MRSA

What is the treatment for SJS?

Uncomplicated cases resolve in 1 mo. Treatment focuses on removing offending agent and maintaiing nutritional and fluid requirements. Child prognosis > adults.

What is the treatment for hypercalcemia?

Underlying cause. Normal saline with furosemide is emergency TOC. In malignancy, bisphosphonates are safe and effective. Calcitonin.

Infants with colic present with crying for no apparent reason that lasts for >/= to 3 hrs/day and occurs in otherwise healthy baby for 3 days/week < 3 mo of age. Though etiology is unknown, what are proposed etiologies?

Unknown. May be associated with: 1. Hunger or swallowed air that has passed into the intestines. 2. High carb containing foods. 3. Food allergy. Typically occurs in infants ~ 10 wks, rarely persists past the age of 4 mos.

How much weight are infants expected to lose in the first 5 days of life? When are they expected to regain their birth weight?

Up to 7% of their birth weight. 10-14 days.

This test in GERD detects grossly abnormal reflux. This test establishes GERD as a cause of symptoms.

Upper GI Series, Bernstein's test

Galactosemia

Uridyl Transferase Deficiency Supplement with soy-based formula (lactose-free) Features: 1. jaundice 2. HSM 3. vomiting, hypoglycemia, seizures, lethargy, irritability 4. poor feeding and failure to thrive with regular milk 5. aminoaciduria, liver failure, MR, risk for E.coli sepsis

Of the causes of galactosemia, which causes hypotonia and deafness?

Uridyl diphosphate galactose-4-epimerase deficiency

What is the most common form on congenital heart disease?

VSD

holosystolic starting with S1; blowing murmur

VSD

What common congenital heart defects can present with CHF and a murmur?

VSD, aortic stenosis, coarctation of the aorta, PDA

What causes atresia of the jejunum or ileum in an infant?

Vascular accident in utero. Women who take cocaine or tobacco during pregnancy put their babies at risk.

9 month black baby with swollen feet and hands. Diagnosis?

Vaso-occlusive phenomena due to sickle cell disease.

Innocent Heart Murmurs

Venous hum - continuous murmur below the right mid-clavicle that goes away while supine, loudest while sitting Still's murmur: grade I-III systolic murmur best heard at the mid-left sternal border, loudest in supine Pulmonic systolic murmur I-II high-pitched systolic murmur best heard at the upper left sternal border, loudest in supine

Lead capillary level is mildly elevated what do you do next?

Venous lead.

"VACTERL"

Vertebral Anal atresia, Cardiac, Tracheoesophageal fistula renal limb.

VACTERL

Vertebral, Anal atresia, Cardiac, TracheoEsophageal, Renal, Limb, Commonly associated abnormalities

What is the treatment for chronic OM?

Very difficult --> likely surgery (mastoirdectomy, myringoplasty and tympanoplasty sytemic abx may be needed

Retrograde flow of urine from bladder into the ureter is termed

Vesicourethral reflux

Fever, lethargy and signs of heart failure after a recent cold. Diagnosis? Prognosis?

Viral myocarditis. Mortality is 75% in newborns and 25% for older infants/children. 33% develop dilated cardiomyopathy.

Patient with conjunctivitis whose complaints are focused on a burning, gritty, itchy feeling most likely have a _________ etiology. Those with a complaint of "eyes stuck shut" most likely have a ___________ etiology.

Viral,Bacterial

X-Ray shows cupping and fraying of the metaphyses of the long bones

Vit D or calcium deficiency = Rickets

This deficiency is rarely seen in the US but is the 3rd MC in the world. Can lead to night blindness, complete blindness, and advanced stages of xerophthalmia (pathologic dryness of conjunctiva and cornea. What deficiency is this?

Vitamin A

This is the 3rd MC vitamin deficiency in the world and can also be seen in pts with disorders associated with fat malabsorption- CF, celiac disease, cholestatic liver disease (biliary cirrhosis, small bowel crohns, pancreatic insufficiency).

Vitamin A

For children hospitalized for measles what medicine is given to them?

Vitamin A.

What are the macrocytic anemias? MCV > 100?

Vitamin B12 deficiency Folate deficiency

Niacin is also known as...

Vitamin B3

This vitamin deficiency occurs most in severely malnourished individuals, drug and alcohol users, or those living in poverty or on diets devoid of fruits and vegetables. Breast milk provides an adequate source for newborns and infants.

Vitamin C

This vitamin deficiency is common in children who are dark skinned and on vegetarian and unusual diets, use anticonvulsant or antiretroviral meds, or with malabsorptive conditions. Addtl. Risk factors: residence in higher latitudes, winter season, and other causes of low sun exposure.

Vitamin D

This vitamin deficiency is common in infants who are dark skinned and exclusively breast-fed beyond 3-6 months of age, particularly if there are additional risk factors such as maternal deficiency of this vitamin during pregnancy.

Vitamin D

Black baby who is exclusively breast fed and stays at home all day has the following findings. Genu varum, enlargement of the costochondral joints and metaphyseal cupping and fraying. Diagnosis?

Vitamin D deficiency rickets.

Presents as an "adolescent girl, w/ excessive menstrual bleeding, recurrent nosebleeds, and pallor" most common inherited bleeding disorder; autosomal dominant; S/S: mucocutaneous bleeding, epistaxis, gingival bleeding. Likely diagnosis?

VwB dz

The essential feature in nephrotic syndrome is: a. Proteinuria b. Hyperlipidemia c. Edema d. Hypoalbuminemia

a. Proteinuria

Patients with lichen planus may present with lacy, reticular white lines covering the lesions. These are called..

Wickham Striae

This is a type of gliobastoma that does NOT show calcification.

Wilm's tumor

This type of childhood tumor arises from the precursor of the normal kidney. Most common malignant renal tumor in childhood. Most children present due to abdominal mass felt by parents.

Wilm's tumor

Wilms tumor association?

Wilms tumor Aniridia Genitourinary anomalies Retardation (intellectual disability) Beckwith-Wiedemann Denys-Drash

This inherited disorder can manifest as chronic hepatitis and is characterized by a deficiency of a-antitrypsin.

Wilson disease

What is the penicillin prophylaxis guideline for rheumatic fever?

Without carditis, at least 5 years or till 21. With carditis at least 10 years or till 21. With persistent heart or valvular disease at least 10 years or until 40.

What is a complication in a neonate who is receiving noninvasive positive-pressure ventilation if they have congenital diaphragmatic hernia?

Worsening respiratory status as air is pumped into the GI tract as well which compresses the lungs.

What is the pathophysiology for hyper-IgM syndrome?

X-linked defect in CD40 ligand. which is responsible for class switching. (default is IgM production).

Duchenne Muscular Dystrophy

X-linked progressive proximal weakness, onset 5 years, Gower's sign, waddling gait

Chronic granulomatous disease

X-linked recessive defect in hydrogen peroxide production in neutrophils, they can phagocytize but not kill bacteria, Catalase positive bacteria: Staph, Asp, Kleb, Serratia,

Lesch-Nyhan Syndrome

X-linked recessive, HGPRT deficiency (for purine metabolism) so increased uric acid deposits in peripheral tissues (gouty arthritis, tophus formation, obstructive nephropathy) NOTE: suspect IF you see a boy with gout PRESENTS around 6 months with hypotonia and persistent vomiting WORSENS with progressive mental retardation, choreoathetosis, spasticity, dysarthric speech CHARAC: dystonia and self-mutilation TX: allopurinol decreases uric acid levels with high fluid intake

Kallman Syndrome

X-linked recessive, can happen to either girls or boys absence of secondary sex characteristics, small genitalia for boys/amenorrhea for girls, and problems with olfaction

Duchenne Muscular Dystrophy (DMD)

X-linked recessive, genetic testing (gold standard) shows deletion of the dystrophin gene on Xp21. serum creatine kinase and aldolase are elevated due to muscle damage fibrosis and fatty infiltration of muscle biopsy support the dx

Wiskott-Aldrich Syndrome

X-linked, eczema, thrombocytopneia (hemorrhage), recurrent infections, risk of lymphoma, tx with IVIG and bone marrow transplant

Wiscott-Aldrich Syndrome

X-linked; thrombocytopenia (<50,000 and small platelets) due to decreased platelet production, eczema, and recurrent infections (often Strep pneumo, Hib, N. mening)

What will you see on XR of pt with Osgood-Schlatter's?

XR needed to r/o other d/os (such as avulsion fracture). May note fragmentation of the tubercle contour.

What is the best imaging tool to diagnose mastoiditis, and what will you see?

XR of the mastoid bone may show soft-tissue swelling, loss of mastoid bone mass and lytic lesions.

Klinefelter's syndrome

XXY, syndrome associated with: 1. behavioral problems (immaturity, insecurity) 2. developmental delay (speech, language, lower verbal/full IQ, MR) 3. physical findings (gynecomastia, hypogonadism/sparse body hair/azospermia, long limbs)

Can the mother of an unborn child, who has capacity, refuse to undergo a Cesarean in the setting of post-term pregnancy complications.

Yes

For children with sickle cell anemia is it advisable to give penicillin prophylaxis?

Yes until at least 5 years of age as these children are at increased risk for strep pneumoniae due to there spleen dysfunction.

Occipital headache in a child. Is this something to be concerned about?

Yes, these are extremely rare and should raise concern for a structural lesion.

Is mammary gland enlargement, leukorrhea and mild uterine bleeding normal in newborns?

Yes, this is known as maternal estrogen effects in newborns.

Are Koplik spots pathognomonic of measles? Why is this important?

Yes. Sometimes these children may present similarly to Kawasaki so look out for this finding, discrepancies in how long they have had a fever and if they have been vaccinated.

What is the typical presentation of HUS?

Young child who has abdominal pain and bloody diarrhea. Patient becomes jaundice and pale with initial lab studies showing anemia, thrombocytopenia and renal insufficiency.

Perioral dermatitis is most common in what age group?

Young women (age 16-45)

Influenza: Neuraminidase inhibitos have a Shorter duration of disease if given w/ in first 48H of sx. What are examples?

Zanamivir, Oseltamivir

Cerebral Palsy

a group of clinical syndromes characterized primarily by nonprogressive motor dysfunction. 3 subtypes: spastic, dyskinetic, ataxic $ caused by prenatal insults to brain development, with premature birth < 32 weeks as the greatest risk factor. NOTE: => 50% have intellectual disability => Spastic Diplegia MC in preterm infants: 1. hypertonia and hyperreflexia in LE 2. both feet pointing down and inward (equinovarus deformity) 3. resistance to passive muscle movement increases with more rapid movement of the affected extremity ("clasp-knife")

Accumulation of fluid in the scrotum around the testes (tunica vaginalis) is called...

a hydrocele

A child has mental age of 7 and chronological age of 7 years. His IQ is: a. 100 b. 80 c. 120 d. 145

a. 100

The age of onset of JR is a. 16 years b. 13 years c. 8 years d. 12 years

a. 16 years

In children most common complication of measles is: a. ASOM b. Orchitis c. Meningitis d. Bronchopneumonia

a. ASOM

False regarding myelomeningocele in children is a. Absent hydrocephalus b. Fecal and urinary incontinence c. Areflexic paraparesis d. Neural tube defects

a. Absent hydrocephalus

Leukemoid reaction is seen in: a. Acute infection b. Erythroleukemia c. Myelomatosis d. Hemorrhage

a. Acute infection

Post- measles bronchopneumonia is treated with: a. Antibiotics and oxygen b. No treatment is necessary c. Steroid d. Only oxygen

a. Antibiotics and oxygen

Obstructive hydrocephalus in children is most commonly caused by a. Aqueductal stenosis b. Arnold-chiari malformations c. Meningitis d. Dandy walker syndrome

a. Aqueductal stenosis

False negative tuberculin reaction does not occur in children in which of the following condition a. Atypical mycobacteria b. 4-6 weeks followign measles c. Patient on corticosteroid therapy d. Very severely malnourished child

a. Atypical mycobacteria

Thalaseemia is because of a. Deficiency of spectrin b. Defect in spleen c. Defect in hemoglobin synthesis d. All of the above

a. Deficiency of spectrin

Asperger's disorder is: a. Developmental delay b. Neuromuscular disease c. Degenerative disorder d. Metabolic disorder

a. Developmental delay

In kwashiorkor the diagnosis is based on which of the following a. Edema with mental changes b. Edema with skin changes c. Bitot's spots and hepatomegaly d. Weight loss and mental changes

a. Edema with mental changes

Not a complication of chickenpox a. Enteritis b. Pneumonia c. Encephalitis d. Reye syndrome

a. Enteritis

Bronchiolitis is treated with: a. Humidified oxygen b. Steroids c. Antibiotics d. All of the above

a. Humidified oxygen

Not a feature of Down's syndrome a. Hypertonicity b. Mental retardation c. Simian crease d. Brushfeild spots

a. Hypertonicity

Zinc deficiency causes a. Hypogonadism b. Impaired glucose tolerance c. Cadiomyopathy d. Tetany

a. Hypogonadism

Most common cause of aplastic anaemia is: a. Idiopathic b. Petroleum products c. Phenylbutazone d. Chloramphenicol

a. Idiopathic

Not seen in Systemic juvenile rheumatoid arthritis a. Iridocyclitis b. High fever c. Rheumatoid rashes d. Hepatosplenomegaly

a. Iridocyclitis

Flag sign is seen in which of the following diseases: a. Kwashiorkor b. Pellagra c. Hypothyroidism d. Marasmus

a. Kwashiorkor

Which of the following is the characteristic feature of fragile X syndrome a. Large testis b. Large nose c. Large eyes d. Long face

a. Large testis

Characteristic feature of Stills disease is a. Maculopapular rash b. Renal involvement c. Neutropenia d. Rheumatoid factor +ve

a. Maculopapular rash

The mechanism of diarrhea in giardiasis is prevention of absorption of fat by: a. Mechanical coating of the walls of intestine b. Production of exotoxins c. Production of endotoxins d. Secondary bacterial infection

a. Mechanical coating of the walls of intestine

A newborn exlclusively fed with goat's milk will develop which of the following diseases as compared to being fed with breast milk a. Megaloblastic anemia b. Microcytic anemia c. Rickets d. Scurvy

a. Megaloblastic anemia

In infantile spinal muscular atrophy is a group of disorders which affect a. Motor neurons b. Pyramidal tracts c. Peripheral nerves d. Neuromuscular junctions

a. Motor neurons

Defective hepatic conjugation is not seen in a. Novobiocin therapy b. Crigler Najjar syndrome c. Gilbert's syndrome d. Proloned bleeding time

a. Novobiocin therapy

In assessment of gestational age is a neonate the following are seen except: a. Ocular b.Breast bud c. Genitalia d. Scrotal skin

a. Ocular

Hyaline membrane in the lung is seen in: a. Respiratory distress syndrome b. Pneumococcal pneumonia c. Acute viral hepatitis d. Pulmonary edema

a. Respiratory distress syndrome

Most common cause of bronchiolitis: a. Respiratory syncytial virus b. Pneumococcus c. Haemophilus d. Enterovirus

a. Respiratory syncytial virus

In newborns, Most common cause of pneumothorax is: a. Resuscitation b. Hyaline membrane disease c. Pneumonia d. Meconium aspiration

a. Resuscitation

False regrading transient tachypnea of newborn (TTN) is a. Reticulonodular patterns are seen b. Fluid in intralobular fissure c. Mild pleural effusion d. Prominent perihilar streaking

a. Reticulonodular patterns are seen

False regarding systemic juvenile rheumatoid arthritis is a. Rheumatoid factor positive b. Leukocytosis c. Hepatosplenomegaly d. Fever

a. Rheumatoid factor positive

Drug of choice initially in juvenile chronic arthritis is: a. Salicylates b. Prednisolone c. Phenylbutazone d. Indomethacin

a. Salicylates

In children which of the following is not used in treatment of asthma a. Sedatives b. Salbutamol c. Adrenaline d. Steroids

a. Sedatives

Charcot arthritis

a.k.a. neurogenic arthropathy = joint destruction resulting from deterioration of proprioception, pain sensation, and temperature sensation

Acute iron poisoning vs. Lead poisoning

abd pain, N/V, hematemesis (corrosive to GI mucosa), hypovolemic shock and anion-gap metabolic acidosis b/c iron is a potent vasodilator => poor perfusion, buildup of lactic acidosis => e.g. epigastric pain, low bicarb, hypotension, cool extremities, compensatory tachypnea GIVE IV fluids and deferoximine; whole bowel irrigation => those who survive acute toxicity are at risk for gastric scarring and pyrloric stenosis => DX: radiopaque tablets on abd X-ray more chronic process: Irritability, poor appetite, headaches and abd pain/V/anemia TX with calcium EDTA, hemodialysis

In acute testicular torsion is the cremasteric reflex present or absent?

absent

Meniere's disease Vs. Otosclerosis

accumulation of fluid in the inner ear that leads to hearing loss, vertigo, and tinnitus bony overgrowth of the stapes footplate leading to conductive hearing loss

Erythema Multiforme tends to happen in the ____ areas of the body such as palms, soles, dorsal aspect of hands & feet

acral

Acute pharyngotonsilitis: What are serious complications of untreated strep?

acute GN, rheumatic fever

Fixed pupil and severe HA with nausea in conjunctivitis are red flags and should make you concerned about..

acute angle closure glaucoma

What is the most common cause of ARDS?

acute gastric aspiration

Weber test: • With unilateral conductive hearing loss, tone is perceived in the _______ ear. • With unilateral sensorineural hearing loss, tone is perceived in the ______ ear. Affected or unaffected?

affected, unaffected

Best for DX intussception?

air/barium contrast enema

What is the only indication for steroids in mono-associated EBV?

airway compromise is the only reason you should give steroids!

TX for pinworm infx?

albendazole pyrantel pamoate (mostly for preg women) highly contagious so all contacts should be treated as well

What is the 2nd most common cause of Hepatitis?

alcohol

You suspect your 9 year old Sallie Mae has aspiration pneumonia. What organism most likely caused it?

anaerobes

Contraindications to rota vaccine

anaphylaxis HX of intussusception (small risk) and uncorrected congenital malformation of the GI tract (Meckel's) SCID

Tuberous Sclerois

angiofibromas, retinal hamartomas, seizures, mental retardation

This treatment in allergic rhinitis is most effective on early phase sx and has little effect on nasal congestion.

antihistamine

What is the treatment for pityriasis rosea?

antihistamines, topical steroids, UVB light. self limiting

TX for septic arthritis:

arthrocentesis and IV antibiotics

What is the allergic triad?

asthma/allergic rhinitis/atopic dermatitis

Juvenile idiopathic arthritis, how long does the arthritis present for?

at least 6 weeks.

Posterior nose bleeds are more common in the elderly due to ___________

aterteriosclerosis

Complication to be cautious for in down syndrome patients?

atlantoaxial instability.

Patient presents with pruritic, hyper-reactive skin, common in flexural creases, food sensitivities common. What is the likely diagnosis?

atopic dermatitis

What is the most common skin dz in children?

atopic dermatitis

Schmid metaphyseal dysplasia

autosomal dominant presents similar to rickets with short stature, leg bowing, and waddling gait Radiographs show irregular long bone mineralization NORMAL LABS (Ca, alk phos, phosphorus)

In which of the following leukemias, chloroma is seen a. CML b. AML c. ALL d. CLL

b. AML

Pincer grasp develops by a. 5-7 b. 9-12 c. 8-9 d. 12-24

b) 9-12

The percentage of calories to be supplied by proteins is: a. >20% b. 10-15% c. 15-20% d. 5-10%

b. 10-15%

Most common cause of ambioguous gentilia in children is a. Mixed gonadal dysgenesis b. 21-hydroxylase deficiency c. klinefelter syndrome d. Testicular feminization

b. 21-hydroxylase deficiency

The height of children in 2-10 years increases a. 10 cm/year b. 6 cm/year c. 2 cm/year d. 4cm/year

b. 6 cm/year

Microcytic hypochromic anemia is not seen in a. Beta thalassemia b. CRF c. Iron deficiency anemia d. Lead poisoning

b. CRF

Which of the following is true regarding neonatal herpes a. Spontaneous recovery occurs b. Caused by HSV II c. Hepatosplenomegaly is diagnostic d. All are correct

b. Caused by HSV II

In vitamin A deficiency, X3A means a. Corneal scar b. Corneal ulcer c. Corneal xerosis d. Conjunctival xerosis

b. Corneal ulcer

Mediastinal widening is seen in a. Prolymphocytic leukemia b. T cell ALL c. CML with blast crisis d. Histocytic lymphoma

b. T cell ALL

Central cyanosis with systolic murmur at birth is seen in which of the following conditions a. Endocardial cushion defects b. TGA c. VSD d. PDA

b. TGA

False regarding childhood ITP is a. Splenectomy for chronic case b. Decreased bone marrow megarkaryocytes c. Preceding viral infection d. Prolonged bleeding time

b. Decreased bone marrow megarkaryocytes

IUGR is not caused by a. CRF b. Diabetes c. Smoking d. Alcohol

b. Diabetes

Which of the following congenital diseases is characterized by reduced production of RBC a. Sickle cell anemia b. Diamond blackfan syndrome c. Polycythemia vera d. Paroxysmal nocturnal hemoglobinuria

b. Diamond blackfan syndrome

Most common cause of aseptic meningitis is a. Arbovirus b. Enterovirus c. Adenovirus d. Herpes virus

b. Enterovirus

Mycoplasma pneumonia is treated with a. Oral penicillin b. Erythromycin c. Co-trimoxazole d. Rifampicin

b. Erythromycin

Inborn error of metabolism associated with mental retardation a. Scheie syndrome b. Homocystinuria b. Pentosuria d. Alkaptonuria

b. Homocystinuria

Poisoning with which of the following can cause unexplained ataxia in a child: a. Salicylate b. Lead c. Mercury d. Arsenic

b. Lead

Most common infratentorial tumor in children is a. Ependymoma b. Medulloblastoma c. Glioma of midline of cerebellum d. Glioma multiforme

b. Medulloblastoma

Which of the following intracranial tumors in children have poor prognosis a. Hemangioma b. Medulloblastoma c. Glioma d. Glioma multiforme

b. Medulloblastoma

NOTE:

benign astrocytomas are the most common supra and infratentorial CNS tumors

Not a characteristic feature in petit mal seizure a. 3Hz pike and wave pattern in EEG b. Onset after the age of 14 years c. Absence of motor activity during seizures d. Short duration of seizure

b. Onset after the age of 14 years

Oligohydraminos is associated with which of the following conditions: a. Esophageal atresia b. Renal agenesis c. Anencephaly d. All of the above

b. Renal agenesis

Infantile diarrhea is not caused by : a. Adenovirus b. Reovirus c. Rota virus d. Caliciviruses

b. Reovirus

Diarrhea in children is commonly caused by a. Pneumococcus b. Rotavirus c. E. coli d. Vibrio cholerae

b. Rotavirus

Which of the following is the most common complication of amebic liver abscess in children a. Secondary infection b. Rupture into peritoneal cavity c. Recurrence d. Rupture into right lung

b. Rupture into peritoneal cavity

A child presented with diarrhea followed by development of rashes and petechiae. Most likely diagnosis is a. Giardia b. Shigella c. Campylobacter d. Vibrio

b. Shigella

A 5-year-old child of normal intelligence presents with feature of hypotonia. There were fasciculation in this tongue and he keeps his body in a frog like position. Most likely diagnosis is a. Limb girdle atrophy b. Spinal muscular atrophy c. GB syndrome d. Down syndrome

b. Spinal muscular atrophy

Which of the following is true regarding Klinefelter syndrome a. Breast adenoma b. Subnormal intelligence c. Pituitary adenoma d. Short stature

b. Subnormal intelligence

Most common chromosomal anomaly is a. Fragile X syndrome b. Trisomy 21 c. Trisomy 18 d. Trisomy 13

b. Trisomy 21

Not a characteristic feature of Lowe's syndrome: a. Defect in CNS and eye b. Undescended testes c. Mental deterioration d. Hypophosphatemic rickets

b. Undescended testes

Not seen in a child with nephrotic syndrome a. Reduced serum calcium b. Uremia c. Hyperlipidemia d. Hypoproteinemia

b. Uremia

Which of the following is true regarding porencephaly a. Neural tube defects b. Vascular lesion due to degenerative vessl disease and head injury c. Dandy-walker syndrome d. Fetal alcohol syndrome

b. Vascular lesion due to degenerative vessel disease and head injury

Not a characteristic feature of infantile pyloric stenosis a. More common in male child b. Vomitting at birth c. Visible peristalsis d. Presence of lump

b. Vomitting at birth

What will you see on CXR in tetralogy of fallot?

boot shaped heart

Child changes rattle from one hand to another at: a. 1 year b. 6 months c. 3 months d. 2 years

b.6 months

A mother is preparing to breastfeed her infant, it would be of value to her, if she knows: a. Grasp reflex b.Rooting reflex c. Moro's reflex d. Cough reflex

b.Rooting reflex

language

babbling begins at 6months 1st word by 1 year 2 word phrases by 2 years

NOTE:

babinski reflex is normal age <1

altered mental status, lethargy, fever, nuchal rigidity, vomiting in child >1 month

bacterial meningitis by strep pneumo and neisseria meningitidis; get LP then IV antibiotics, blood cultures, CBC and CMP TX: ceftriaxone/cefotaxime OR vancomycin NOTE: don't give ceftriaxone to infant < 28 days => hyperbilirubinemia => kernicterus NOTE: dexamethasone can reduce risk of sensorineural hearing loss, esp when its HiB Indications for imaging perior to LP: 1. hx of hydrocephalus or neurosurgical procedures 2. hx of head trauma 3. coma/focal neurological findings

Your sinusitis patient complains of upper molar tooth pain which is accompanied by halitosis..what is the likely etiology?

bacterial`

How is influenza typed?

based on the surface antigens hemagglutinin (H) and neuraminidase (N).

NOTE:

bedwetting is normal until age 5

Open comedones are called _____ heads, while closed comedones are called _____heads

black, white

an OPEN comedone is a ____ head, a CLOSED comedone is a ___ head

black, white

In torsion of the APPENDIX of the testes palpation of the testis reveals a 3-5mm tender indurated mass in the upper pole. It may be visible through the scrotum and is called the ________ sign

blue-dot

What is a side effect of hydroxyurea?

bone marrow suppression.

fever, severe headaches (nocturnal or morning) and focal neurological changes and seizures

brain abscess, with the following risk factors: 1. otitis media, mastoiditis => temporal lobe, cerebellum 2. recurrent frontal or ethmoid sinusitis*** => frontal lobe 3. dental infection => frontal lobe 4. bacteremia from cyanotic heart disease*** or other sites of infection => multiple abscesses along MCA

Is oral candidiasis more common in breast fed infants or non breast fed infants?

breast fed

Infantile Spasms

brief, myoclonic jerks lasting 1-2 seconds, occurring in clusters of 5-10 seizures spread out over 3-5 minutes $ sudden extension of arms, flexion of head $ tuberous sclerosis MCC; also perinatal asphyxia, intraventricular hemorrhage, meningitis

Patients with symptoms of cough, decreased air entry, dyspnea and wheezing are consistent with bronchial or laryngotracheal aspiration?

bronchial

Though 80-90% of patients with hyaline membrane disease will survive with normal lungs by 1 mo of age, what is a serious complication?

bronchopulmonary dysplasia

commonly occurs in neonate. Associated with weakness, fever and diarrhea. Vesicles on face, trunk, buttocks, perineum or extremities. Vesicles enlarge into flaccid, translucent bullae and can be as large as 5cm. Easily rupture and a shiny dry erosion remains. Heals quicker. Is this bullous or nonbullous impetigo?

bullous

Most common cause of meningitis in neonates is: a. Streptococcus b. Haemophilus c. Escherichia coli d. Meningococcus

c. Escherichia coli

In which of the following disorders deficiency of glucouronyltransferase is seen a. Intrahepatic cholestasis b. Rotor syndrome c. Dubin Johnson syndrome d. Crigler- Najjar syndrome

d. Crigler- Najjar syndrome

Which of the following is not seen in congenital rubella syndrome? a. PDA b. Deafness c. Aortic stenosis d. Mental retardation

c) Aortic stenosis

Infant triples weight at the age of __ months a. 5 b. 11 c. 18 d. 24

c. 18

For a 2-year-old child calories as supplied in ICDS diet are- a. 1000 Kcal b. 500-600 Kcal c. 300-400 Kcal d. 200 Kcal

c. 300-400 Kcal

Syphilis attack the fetus after the....... months: a. 2nd b. 7th c. 4th d. 3rd

c. 4th

Which of the following shows the most specific finding in pediatric age with nutrition/malabsorption state a. Malnutrition b. Giardiasis c. Abetalipoproteinemia d. Tropical sprue

c. Abetalipoproteinemia

Which of the following is the most common malignancy in children a. Neuroblastoma b. Wilms tumor c. Lymphoma and leukemia d. Retinoblastoma

c. Lymphoma and leukemia

A 2-year-old HIV positive boy presents with history of croup, fever, cough, diffuse wheezing and creps all over. Examination reveals hyperinflated cest. His management should be started with a. Ganciclovir b. Erythromcycin c. Co-trimoxazole d. Nebulised ribavirin

c. Co-trimoxazole

In children suprasellar calcification suggest a. Meningioma b. Neuroblastoma c. Craniopharyngioma d. Glioma

c. Craniopharyngioma

True about beta thalassemia is a. Decreased alpha chain, decreased beta chain b. Increased beta chain, increased alpha chain c. Decreased beta chain, increased alpha chain d. Increased beta chain, decreased alpha chain

c. Decreased beta chain, increased alpha chain

Unconjugated hyperbilirubinemia is not seen in a a. Hemolytic anemia b. Hypothyroidism c. Dubin johnson syndrome d. Gilbert syndrome

c. Dubin johnson syndrome

In the females, congenital adrenal hyperplasia causes which of the following conditions: a. Infant hercules b. Macrogenitosomia praecox c. Female pseudohermaphroditism d. None

c. Female pseudohermaphroditism

In children most common cause of subdural effusion is a. Meningococcus b. Streptococcus c. Hemophilus influenzae d. Staphylococcus

c. Hemophilus influenzae

Not found in TOF a. Severity of cyanosis is directly related to the severity of pulmonary stenosis b. Ventricular septal defect c. Left ventricular hypertrophy d. Pulmonary stenosis

c. Left ventricular hypertrophy

In rheumatic fever most common valvular lesion is a. Aortic regurgitation b. Aortic stenosis c. Mitral regurgitation d. Mitral stenosis

c. Mitral regurgitation

Transplacental transmission is not seen in a. Toxoplasma b. Rubella c. Mumps d. Syphilis

c. Mumps

A child has anti-mongoloid slant, pulmonary stensois, short stature and undescended testis. The child has a. Down syndrome b. Turner syndrome c. Noonan's syndrome d. Klinefelter syndrome

c. Noonan's syndrome

Which of the following of juvenile rheumatoid arthritis is associated with iridocyclitis? a. Rheumatoid factor +ve b. Systemic c. Pauciarticular d. Polyarticular

c. Pauciarticular

Phenytoin i snot used for which of the following a. Focal cortical epilepsy b. Psychomotor epilepsy c. Petit mal epilepsy d. Cardiac arrhythmia (digitalis induced)

c. Petit mal epilepsy

Not a cause nephrotic syndrome: a. Membranous glomerulonephritis b. Focal glomerulonephritis c. Post streptococcal glomerulonephritis d. Minimal lesion glomerulonephritis

c. Post streptococcal glomerulonephritis

Transient tachypnea of the newborn develops in: a. Term baby by spontaneous vaginal delivery b. Term baby by forceps c. Preterm baby by cesarean section d.Preterm with ventouse

c. Preterm baby by cesarean section

The clinical signs in measles which appears last is: a. Coryza b. Fever c. Rash d. Koplik's spot

c. Rash

A child on 10th day of birth was found to have 10% decrase in weight after excessive breastfeeding. It was passing golden yellow soft stools. Anxious parents report to the pediatrician. The most common advice is a. Start the child on high protein diet b. More frequent breastfeeding c. Reassurance d. Investigate for lactic acidosis

c. Reassurance

Unconjugated hyperbilirubinemia is not seen in a. Crigler-Najjar syndrome b. G6PD deficiency c. Rotor syndrome d. Thalassemia

c. Rotor syndrome

Which of the following is the most common complication of chickenpox a. Diarrhea b. Encephalitis c. Secondary bacterial infection d. Pneumonia

c. Secondary bacterial infection

What is the antidote for lead poisoning?

calcium disodium edetate

Which of the following is true regarding brain tumor in children a. Hemiparesis is present b. Usually causes papilledema c. Usually infratentorial d. Most common tumor in chldren is wilms tumor

c. Usually infratentorial

Hemophagocytic Lymphohistiocytosis (HLH) or Macrophage Activation Syndrome (MAS)

can be caused by malignancy, EBV, ticket borne illnesses where immune systemic gets pathologically activated and starts destroying everything Ferritin > 5000 = diagnostic

Maple Syrup Urine Disease

can't metabolize branched-chain amino acids

Diaper dermatitis w/ a skin fold infection indicates that this fungus is likely the cause

candida

What will you see on CXR in large PDA?

cardiomegaly, left atrial and ventricular enlargement and increased pulmonary congetsion are noted.

What will be seen on prenatal examinations in duodenal atresia?

characteristic double bubble appearance seen on a postnatal abdominal radiograph. Due to dilation of the stomach and the first part of the duodenum. Polyhydramnios- occurs in 50%, hyperechoic bowel, and ascites.

What is the pharmacologic treatment for epiglottitis?

cefotaxime, ceftriaxone, ampicillin/sulbacam, chloramphenicol and steroids

What are the most common causes of hearing loss

cerumen impaction, Eustachian tube dysfunction, increasing age (presbycusis), barotrauma

Patient presents with Weak or absent femoral pulses and delayed femoral pulse when compared with UE's. Upper extremity HTN. A systolic ejection murmur may be heard at the apex. CXR = enlarged aortic knob, notching of the ribs may be noted. What is the likely dx?

coarctation of the aorta

A 2 month old infant presents your clinic with persistent crying typically in the late afternoon for evening > may persist for several hours > face may be flushed, abdomen distended and legs drawn up to the abdomen. Abdominal exam rules out intussusception and strangulated hernia. What are you highly suspicious of?

colic

How do you manage encopresis (involuntary dc of feces)?

conventional medical therapy is commonly the 1st therapy, generally consisting of: demystification and education, colonic disimpaction followed by routine laxative therapy, toilet training)

What special tests can you use in strabismus?

corneal light reflex will reveal misalignment (manifest or heterotropia) and cover/uncover test will reveal latent strabisumus (heterophoria).

What are potential complications of kawasaki disease?

coronary vasculitis and aneursym formation

Non absorbable sugars: lactulose and sorbitol are effective in constipation but may cause..

cramping/bloating

Patient presents w/ 1-5 day prodrome of cough and coryza followed by 3-4 days of brassy, barking cough. Peaks on day 3. What is the likely dx?

croup

Steeple sign on lateral x-ray makes you suspicious of...

croup

Was is the first line surgical treatment for genital warts (condyloma acuminata)? CMDT

cryotherapy

"Undescended testes" is also called

cryptochidism

Bloody mucus in intussuception is termed...

currant jelly stool

Not a feature of necrotizing enterocolitis a. Abdominal distension b. Blood in stool c. Bilious vomitting d. Increased bowel sound

d) Increased bowel sound

False regarding polio is a. Fasciculation seen b. Normal sensorium c. Autonomic involvement d. 70% of cases have muscle paralysis

d. 70% of cases have muscle paralysis

Which of the following is the treatment of choice in infantile myoclonus a. Clonazepam b. Phenytoin c. Phenobarbitone d. ACTH

d. ACTH

Which of the following is marker for neural tube defects? a. Acetyl phosphatidyl esterase b. Acetyl cysteine c. Acetyl glucouronidase d. Acetyl cholinesterase

d. Acetyl cholinesterase

A child has pedal edema ++, facial edema. His blood pressure is 190/110. he has gross hematuria but ascites is absent. He is suffering from a. CRF b. Renal vein thrombosis c. Nephrotic syndrome d. Acute glomerulonephritis

d. Acute glomerulonephritis

Not a characteritic feature of Kwashiorkor: a. Edema b. Flag sign c. Dermatitis d. Alertness

d. Alertness

True regarding AIDS in children is a. Presents with lymphadenopathy b. Pneumontis often important c. Kaposi sarcoma rarely occurs d. All are correct

d. All are correct

The malignancies of childhood include: a. AML b. Wilms' tumor c. ALL d. All of the above

d. All of the above

Not a characteristic feature of thalassemia major: a. Target cells b. Splenomegaly c. Hypochromic microcytic anaemia d. Decreased serum ion

d. Decreased serum ion

In which of the following syndromes nuchal fold thickness is increased a. Paul Bunnel syndrome b. Cri du chat syndrome c. Turner syndrome d. Downs syndrome

d. Downs syndrome

Neonatal jaundice at birth or before 24 hours is commonly due to a. Physiological b. Biliary atresia c. Congenital hyperbilirubinemia d. Erythoblastosis

d. Erythoblastosis

Which of the following is the most common cause of portal hypertension in children? a. Postnecrotic b. Veno-occlusive disease c. Budd-chiari syndrome d. Extrahepatic compresssion

d. Extrahepatic compresssion

Which of the following is the most common organism causing neonatal septicemia in USA? a. E. coli b. Streptococcus viridans c. Staphylococcus aureus d. Group B streptococcus

d. Group B streptococcus

Not seen in nephrotic syndrome a. Hypercholesterolemia b. Massive proteinuria c. Hypoalbuminemia d. Hematuria

d. Hematuria

The etiological agent of rosiola infantum is a. Parvovirus b. EBV c. CMV d. Human herpes virus 6

d. Human herpes virus 6

Not a major criterion of rheumatic fever a. Arthritis b. Subcutaneous nodule c. Carditis d. Increased ASO titre

d. Increased ASO titre

Which of the following is the first constant symtom of brain tumor children a. Personality disturbances b. Headache c. Vomitting d. Increased head size

d. Increased head size

Fetal lung maturity is assessed by a. USG b. Bilirubin contents of amniotic fluid c. Amniocentesis d. L/S ratio

d. L/S ratio

Not seen in neonatal septicemia a. Band neutrophils b. Increased ESR c. Increased CRP d. Leukocytosis

d. Leukocytosis

Wilms tumor commonly metastasize to which of the following organ. a. Liver b. Brain c. Lymph node d. Lung

d. Lung

Steroids are useful in: a. Rapidly progressing glomerulonephritis b. Membranous glomerulonephritis c.Post- streptococcal glomerulonephritis d. Minimal change glomerulonephritis

d. Minimal change glomerulonephritis

Not a feature of Treacher-collins syndrome a. Deafness b. Colobomas of lower eyelid c. Hypoplasia of zygomatic bones d. Mongoloid slant of palpebral fissure

d. Mongoloid slant of palpebral fissure

Cerebrospinal fluid examination of a one day old term male baby reveals a cell count of 10 RBCs/HPF, sugar 45 mg/dl and protein 40mg/dl. Most likely diagnosis is a. Hypoglycemia b. Intracranial bleed c. Meningitis d. None of the above

d. None of the above

Incrase levels of acetyl cholinesterase in amniotic fluid indicate which of the following a. Edward syndrome b. Down syndrome c. Esophageal atresia d. Open neural tube defects

d. Open neural tube defects

In which of the following ossification center appears first a.Upper end of tibia b. Femur c. Talus d. Ossicles

d. Ossicles

In bronchiolitis the treatment of choice is a. Zidovudine b. Vidarabine c. Amatidine d. Ribavirin

d. Ribavirin

Secondary lactose intolerance is seen with which of the following a. Yersinia enterocolitica b. Entamoeba c. Shigella d. Rotavirus diarrhea

d. Rotavirus diarrhea

On 1st day of life, patient presents with bilious vomiting *without abdominal distension*...what are you suspicious of?

duodenal atresia

False regarding sacral myelomeningocele i s a. Lax anal sphincter is seen b. Bladder incontinence c. Hydrocephalus d. Spasticity is a feature of lower limb

d. Spasticity is a feature of lower limb

Which of the following is the earliest sign of puberty in males a. Change in voice b. Moustache c. Pubarche d. Testicular enlargement

d. Testicular enlargement

In girls the first sign of puberty is a. Change in voice b. Moustaches c. Pubarche d. Thelarche

d. Thelarche

Correct sequence of development in puberty in girls is a. Menarche, pubarche, thelarche b. Thelarche, menarche, pubarche c. Pubarche, thelarche, menarche d. Thelarche, pubarche, menarche

d. Thelarche, pubarche, menarche

Blood and mucus in stools are not seen with: a. Shigella shigae b. Escherichia coli c. Entamoeba histolytica d. Vibrio cholerae

d. Vibrio cholerae

Most common complication following meningococcal meningitis is: a. Hydrocephalus b.Arthritis c. Subdural effusion d. Waterhouse- Fredrickson syndrome

d. Waterhouse- Fredrickson syndrome

Polycythemia in a newborn

defined as hematocrit > 65% => hyperviscosity Risk factors: + delayed cord clamping + in-utero hypxoia (maternal HTN, smoking) + maternal diabetes Symptoms: => respiratory distress, neuro symptoms, hypoglycemia TX: if symptomatic, partial exchange transfusion. If not, hydration.

What are the most common causes of syncope in adolescents?

dehydration (MOST COMMON) fear and pain medical conditions

False regarding thumb sucking is a. Must be treated vigorously in first year of life b. Is a sign of insecurity c. Is a source of pleasure d. can lead to malocclusion

d. can lead to malocclusion

HCM murmurs ______ with squat or hand grip & leg elevation and ______ with valsalva.

decrease, increase

Type 2 RTA

decreased absorption of bicarbonate proximally associated with Fanconi syndrome with glucosuria, aminoaciduria, hyperphosphaturia

What will you see in labs of a pt with ITP?

decreased platelet, often severe (less than 50K or even less than 10K)- normal is 150-400K. Rule out pseudothrombocytopenia and DIC. Anitplatelet antibody tests are usu. +

Type 4 RTA

decreased response to aldosterone hyperkalemia

NOTE:

dip in the growth on BMI curve during ages 2-5 is normal due to increase in caloric expenditure

TX for lyme dx?

doxy if > 8 amoxicillin < 8 IV ceftriaxone if mengingitis RMSF always give doxy no matter what

Emollient laxatives: ducosate sodium or mineral oil work to promote stool softening. Which carries a risk for aspiration pneumonitis?

ducosate sodium

In coarctation of the aorta, obstruction is noted in the descending aorta the insertion site of the ...

ducutus arteriosus

Herditary angioedema

due to C1 inhibitor deficiency => elevated C2b and bradykinin which promote edema Rapid onset of : + noninflammatory edema of face, limbs, genitalia + laryngeal edema - life threatening + edema of intestines - colicky abd pain WITH no evidence of uriticaria NOTE: episodes usually follow an infection, dental procedure, and trauma Acquired angioedema => MCC in ACEI use; presents in ages > 30 => LOW C1q levels

Anemia secondary to increased destruction of RBCs will have an elevated or decreased reticulocyte count?

elevated

What does an elevated reticulocyte count mean? What does a depressed reticulocyte count mean?

elevated retic count noted in anemia secondary to increased destruction of RBCs. Decreased retic count and index noted in anemia secondary to decreased production of RBCs.

Jaundice caused by excess production of bilirubin will present with normal or elevated reticulocyte counts? Jaundice with decreased rate of conjugate will present with normal or elevated reticulocyte counts?

elevated, normal

What must be performed if foreign body is aspirated beyond the oropharynx?

endoscopy

What will you see on echo in a patient with ASD?

enlarged right ventricle and flow across the defect.

Gold standard for dx/defect in Heriditary Spherocytosis:

eosin-5-maleimide binding test (flow cytometry) and acidfied glycerol test PATHO: AD, mutation in ankyrin gene => spectrin defiency => RBC membrane defect => spherocytes

Short rapidly progressive febrile illness. Sore throat, pain with swallowing and SOB. PE: Toxic, assumes a forward leaning position, neck-extended with *drooling of secretions.* Likely dx?

epiglottitis

Thumb sign will be present in what disorder?

epiglottitis

NOTE:

erythromycin given to <1 month infant can cause pyloric stenosis

First-line for preventing gonococcal conjunctivitis?

erythromycin ophthalmic ointment...silver nitrate can cause chemical conjunctivits

Convergent, inward misalignment of the eye in strabismus is called...

esotropia

What is the most common form of strabismus?

esotropic strabismus (cross eyes)

When is surgical repair usu. performed in tetralogy of fallot?

first 3-6 months of life

Rare d/o more common in Ashkenazi jews. Both autosomal recessive & dominant. Bleeding not as severe. Sponatenous bleeding and hemarthroses are uncommon. Patients undergoing surgery are @ high risk unless replacement therapy given.

factor XI d/o

In which gender does an incarcerated hernia occur more frequently?

females

How will pts with subluxation of radial head present?

flexed and IR

Ostium Secundum is the most common form of ASD. Where is the defect noted?

fossa ovalis

What is the most common extracranial solid tumor in childhood?

glioblastoma

Bone age < chronological age seen in:

growth hormone deficiency hypothyroidism hypercortisolism constitutional growth delay

Long term complications of bacterial meningitis:

hearing loss loss of cognitive functions (loss of neurons in the dentate gyrus of the hippocampus) seizures spasticity/paresis mental retardation

What defines polycythemia in a neonate. What is the reason why this happens for certain pregnancies?

hematocrit >65%. Either increased erythropoiesis or RBC transfusion (such as delayed cord clamping or twin-twin transfusion) Note: maternal DM, HTN, smoking and IUGR causes increased EPO)

Tay-Sachs disease

hemosamnidase A deficiency, Jews + hyperactive startle and loses eye contact + cherry-red macula, enlarging head circumference, neurodegeneration with severe developmental delay, progressive blindness, and seizures + Death by age 4

Niemann-Pick

hepatomegaly and seizures

In a child with jaundice and splenomegaly, be suspicious of...

hereditary spherocytosis

Incidence of epiglottitis has decreased with the max vaccination for...

hiB

High blood cell turnover due to hemolysis results in:

high LDH, unconjugated hyperbilirubinemia, reticulocytosis, low haptoglobin; can also deplete folic acid stores and cause megaloblastic anemia FOR EXAMPLE in sickle cell anemia or autoimmune hemolysis

false positives with heterophile antibodies test for Mono?

high probability early in disease and with young patients...look at lecture

Standard TX for OCD (either primary or from PANDAS):

high-dose SSRIs and CBT

Congenital megacolon is also known as..

hirschsprung disease

1st exposures to allergies don't always cause drug eruptions because your body makes an antibody (IgE). However, in the second exposure your body makes _____ which causes the allergy symptoms.

histamine

Small for gestational age infants (< 10%) are at risk for:

hypoxia => polycythemia hypoglycemia hypocalcemia perinatal asphyxia hypothermia meconium aspiration

mild chronic hip and knee pain of insidious onset persisting for >1 month plus an antalgic gait with X-Ray showing necrotic bone in boys 4-10 proximal thigh atrophy may be present as the disease progresses, internal rotation and abduction of the hip joint can become markedly limited

idiopathic avascular necrosis of the femoral capital epiphysis a.k.a. Legg-Calve-Perthese disease => managed conservatively with observation and bracing; surgery if femoral head is not contained within the acetabulum NOTE: transient synovitis, presenting after a viral URI, should resolve within 1-4 weeks

Red cell Distribution Width (RDW) values > 20% are suggestive of...

iron deficiency anemia (microcytic anemia) normal values of 12-14% are seen in thalassemia NOTE: commonly caused by excessive intake of cow's milk (> 24 ounches/day)

What is the most common location of intussuception? Is it more common in males or females?

ileocolic. Males 3:1

NOTE:

in CF: hyponatremic, hypochloremic, hypokalemic metabolic alkalosis

Newborns with CF will present with these two disorders.

intestinal obstruction, meconium ileus

What is the most effective medication for overrall control of allergic rhinits?

intranasal corticosteroids

What are complications of hypercalcemia?

kidney stone formation, calcification of blood vessel walls, calcification of organ walls, spontaneous fracture

In tetralogy of fallot, holding the baby in what position increases systemic vascular resistance and allows for temporary reversal of the shunt?

knee/chest

Patients with symptoms of cough, cyanosis, dyspnea and stridor are consistent with bronchial or laryngotracheal aspiration?

laryngotracheal

Release of cytokines and leukotriens influx of inflammatory cells (eosinophils) Can begin 4-6 hours after initial exposure Symptoms = nasal congestion and post nasal drip. Early or late allergic rhinitis?

late

What are the signs and symptoms of lead poisoning?

lead line in gums, papilledema, ocular palsy, wrist drop, foot drop, slurred speech, reflex changes, bradycardia, mental status changes: seizures, delirium, coma

Fanconi Anemia

most common congenital cause of aplastic anemia Diagnosis: chromosomal breaks on genetic analysis along with certain clinical findings + bone marrow failure/aplastic anemia + hypo/hyper pigmented areas + abnormal thumbs

Achondroplasia

most common skeletal dysplasia, AD + prominent forehead, hypoplasia of midface + trident-shaped hands and bilateral short femurs and upper arms Complication: 1. cord compression caused by foramen magnum stenosis => sudden death in infancy 2. obstructive sleep apnea 3. genu varum 4. back pain caused by lumbar lordosis

What is the treatment for a hydrocele?

most resolve by age 2 as the tunica closes or the fluid is reabsorbed. Failure to close by age two requires surgical correction

This type of verruca plantaris manifests as an anthill like lesion.

myrmecia

This type of verrucae is associated with HPV type 1?

myrmecia

Accounts for 70% of all cases of impetigo. 2-4mm erythematous macule that evolves into a vesicle or pustule. Results in a superficial erosion with the "honey-colored crust". Is this bullous or nonbullous impetigo?

non bullous

What is the treatment for ITP? What drugs may worsen ITP?

normally resolves spontaneously. For severe bleeding and counts < 10k, use prednisone for 1-4 weeks. IVIG in conjunction with steroids or IV anti-D. Splenectomy may be needed. Stop all drugs that may be worsening ITP: • Bactrim, quinine, penicillins, furosemide, phenytoin, cimetidine may worsen ITP.

1/2 of cases of foreign body aspiration are due to _____. _____ aspiration is the single most common cause of death.

nuts, hot dogs

Abnormal attachment of the testes is called _______ deformity

o Abnormal attachment is called bell-clapper deformity

What complications is cryptorchidism associated w/?

o Associated with inguinal hernia/torsion/infertility

How do you diagnose and treat erythema infectiosum (5th dz) which is caused by Human Parvovirus B19?

o Diagnosis: Based on clinical findings or presence of IgM antibodies. o Treatment: Usu. self limited.

What is the tx for tinea corporis?

oral antifungals such as Griseofulvin, Itraconazole, Terbinafine and Fluconazole

The cause of perioral dermatitis is unknown but can be aggravated by chronic ________ use

oral steroid

What are complications of untreated sinusitis?

orbital cellulitis

What is the treatment for cryptorchidism?

orchidopexy by 1 year of age - removes undescended testicle and fixes it in scrotum

Cri du chat

partial deletion on short arm of chromosome 5 + slow growth, microcephaly, mental retardation + hypertelorism, classic catlike cry

Nesidioblastosis

persistent low blood sugar due to congenital hyperinsulinemia (B-cell hyperplasia)

Appendicitis: Where is McBurney's point? What is the obturator sign? What is the psoas sign?

perumbilical or epigastric pain, can be localized to RLQ (McBurney's point), obturator sign (pain with flexion and internal rotation of hip), psoas sign (pain on extension of the leg), rebound tenderness

What is the most common cause of newborn jaundice?

physiologic jaundice, prematurity and breast-feeding jaundice.

Enterobius Vermicularis manfiests as...

pinworms

IF you're worried about epiglottidis what test should you get?

plain film (neck X-ray) to demonstrate widening of the retropharyngeal space OR thumbprint sign

What is the treatment for hemophilia A?

plasma derived or recombinant factor concentrates are mainstay of treatment. PREVENT TRAUMA, *avoid aspirin and other antiplatelet medications*. Supplement coagulation factor (factor VIII concentrate, FFP or CRP). Genetic counseling for family.

It is common for patients with duodenal atresiato have a history of excessive amniotic fluid, also called ______

polyhydramnios

Rh and ABO are coombs test _____ incompatibilites. G6pD is a coombs test ____ incompatibility.

positive, negative

Patients with tinea pedis present with blisters, fissures, maceration between toes. What age group is this common in?

post pubertal child

Limited eye movements on exam of a patient with orbital cellulitis is a sign of?

postseptal cellulitis

Isotretinoin is effective in treating nodular and cystic acne but may cause initial flare. What can be used to control the flare?

prednisone

Management of Chronic Granulomatous Disease?

prevent infection with daily TMP-SMX and interferon-gamma 3 times a week Suspect IF: + unusual and recurrent lymphadenitis + hepatomegaly or abscess + unusual infx with catalase + organisms (Staph, Serratia, Aspergillus, Burkholderia) + osteomyelitis at multiple joints

Management of Chediaki-Hidaski dx?

prevent infx with daily TMP-SMX and ascorbic acid

Fanconi's anemia

progressive pancytopenia and macrocytosis, AR + abnormal thumbs + horseshoe kidneys + short stature + microcephaly and ophthalmia, cafe au lait spots, hyper/hypo pigmentation DX: chromosomal breaks on genetics analysis

ENT: What is usu. the etiology of PTA/cellulitis?

quinsy-group A strep

fine motor

reach/grab things at 4 months hand to hand transfer at 6 months rake grasp 6-8months fine pincer grasp 9-11 months

Features of Hyper IgE (Job Syndrome)

recurrent Staph infx of skin, joint, and lungs as well as coarse facial features, dental abnormalities, and bone fx as well as pruirtic dermatitis and eosinophilia with hugely elevated IgE

What is the hallmark of GERD?

recurrent heartburn

Type 1 RTA

reduced secretion of H+ ions in distal tubules, low bicarb, urine pH high >5.5; can cause failure to thrive hypokalemic metabolic acidosis normal anion gap and hyperchloremia with all metabolic acidosis!!!!

Biochemical changes are clinical manifestations of Vitamin D deficiency. How does it present?

reduces intestinal calcium and phosphorus absorption. PTH increases, leading to mobilization of calcium from bone so that serum calcium levels remain normal or are moderately decreased.

Secondary attacks of epiglottitis can be decreased by the prophylactic use of...

rifampin

If pulmonary resistance is greater than sytemic resistance, a ____ to ____ shunt develops.

right to left

What is the most common bacterial pathogen causing pneumonia in infants 1-3 months? What about older children and adolescents?

s. pneumo myco. pneumo

Jonny is a 14 year old boy with a history of influenza that now presents with symptoms of pneumonia. What are the likely causative agents?

s.aureus, s. pneumo, h.infulenza

What bug is most likely to cause gastroenteritis?

salmonella

When should blood lead level be screened?

screen high risk and all Medicaid enrolled or eligible children (age 1-2 years), recent immigrant children on arrival and again at 3-6 months, and high risk children.

When does the AAP recommend screening for anemia?

screening between 9-12 months, and screening at risk children again between 2-5 years.

Patient with a vitamin deficiency presents with ecchymosis, bleeding gums, petechiae, coiled hairs, hyperkeratosis, Sjogren's syndrome, arthralgias, impaired wound healing, weakness, malaise, joint swelling, arthralgias, edema, depression, neuropathy, and vasomotor instability. You know this is ____ caused by vitamin __ deficiency.

scurvy, C

What is the best way to measure iron binding capacity?

serum ferritin, iron, iron-binding capacity (TIBC; the amount of transferrin available to transport iron and percent saturation) the percentage of transferrin saturated with iron).

gross motor

sitting by 6months roll over variable 1 week to 8 months creeping by 6 months crawling by 6-8 months Cruising by 9-12 months independent steps 13 months

Lactose is digested by lactase which is produced in the ___________

small intestine

Patients who have these risk factors are at risk for Acute Bronchiolitis 1. _____________ 2. _______-

smoking exposure, day care attendance

Niemann-Pick vs. Tay Sachs disease

sphingomyelinase VS. B-hexoaminidase A deficiency EPPY: autosomal recessive, Ashkenazi Jews Onset: 2-6 months Clinical features: => BOTH: loss of motor milestones, hypotonia, feeding difficulties, cherry-red macula => hepatosplenomegaly and areflexia VS. hyperreflexia

What physical exam findings EXCLUDES the diagnosis of ITP?

splenomegaly

NOTE:

spondylolisthesis is a developmental disorder characterized by forward slipping of a vertebrae (commonly L5 over S1), resulting in slowly developing chronic back pain and neurologic dysfunction (sensation and urinary incontinence). A palpable "step-off" can be felt if severe. Happens most commonly to preadolescent children.

What seasons is RSV most endemic?, How many people are infected by the age of 1?

spring/fall, 60%

How does chlamydia pneumonia present?

staccato cough, tachypnea, eosionophilia, and bilateral infiltrates with hyperinflation on chest radiograph with or without conjunctivitis TX: erythromycin, azithromycin

Dysphagia, GI, bleeding or weight loss are alarm symptoms in GERD. If present think....

stricture or adenocarcinoma

Sandifer's syndrome

strong correlation with GERD symptoms - spasmodic torticollis and dystonia (weird arching of back)

Name 5 drugs that may cause SJS.

sulfonamides, PCN, phenytoin, phenobarbital, Allopurinol

What is the treatment for DIC, a bleeding d/o characterized by both bleeding and clotting?

supportive & treat underlying cause, heparin (for CLOTTING), depleted factors (FFP, cryo) << for bleeding!

What is the treatment for HSP?

supportive, short course of NSAIDs, severe cases may require steroids for severe abdominal pain or immunosuppression for renal involvement

Though the etiology of pityriasis rosea is unknown, this virus is suspected.

suspect herpes virus type 7

triad of acute onset fever, posterior cervical lymphadenopathy, and pharyngitis as well as fatigue, tender hepatosplenomegaly, mild leukocytosis (atypical lymphocytes)

suspect mono, get heterophile antibodies

IF nephrotic patient suddenly develops flank pain? Belly pain?

suspect renal vein thrombosis 2/2 peeing out ATIII, protein C, and S. Do CT or U/S stat spontaneous bacterial peritonitis

Test for screening CF

sweat chloride test with pilocarpine inotophoresis (need two positives before diagnosis can be made unless baby is already known to have two CF-causing mutations) neonatal screening detects immunoreactive trypsinogen in blood, with confirmatory tests to make final diagnosis NOTE: false positives can occur with metabolic disorders including adrenal insufficiency and hypothyroidism

What happens in the preicteric phase of hepatitis?

symptoms are malaise, nausea, decreased appetite and vague abdominal pain • Viral specific antibodies appear, and serum transaminases elevate

What happens in the icteric phase of hepatitis?

symptoms worsen and jaundice appears • Serum transaminases reach their peak. 10X upper limit. • Urine darkens and stool lightens. • Bilirubin > 3.0 mg/dl

What is the most common cyanotic congenital heart defect?

tetralogy of fallot

solitary, painful lytic long bone lesion with overlying swelling and hypercalcemia in a child (can involve mastoid bone - chronic otorrhea and mass behind the ear)

think Langerhans cell histiocytosis as well as other neoplastic processes (Ewing sarcoma, metastases) NOTE: parathyroid adenoma in >50 yo, and sarcoidosis in 20-35 yo

1.grip myotonia (delayed muscle relaxation - unable to release hand after handshake) 2. facial weakness, foot drop, dysphagia (risk for aspiration pneumonia) 3. cardiac conduction anomalies (and arrhythmias) 4. cataracts, testicular atrophy/infertility, and baldness

think autosomal dominant during teenage years, myotonic muscular dystrophy type 1; expansion of trinucleotide repeat in dystrophia myotonica protein kinase gene causes weakness of skeletal, smooth, and cardiac muscles

cyanosis within 24 hours, single S2 heart sound on auscultation (absent pulmonary component of S2 because the aorta is anterior to the pulm artery), and a narrow mediastinum ("egg on a string") on x-ray

transposition of the great vessels is the most common congenital cyanotic heart disease in the neonatal period WHEREAS tetralogy of fallot is the most common cause after the neonatal period

Acute Aspirin Poisoning

tinnitus, fever, hyperpnea (resp. alkalosis), and metabolic acidosis

What symptoms are associated with partial seizure with secondary generalization?

tonic-clonic manifestations (diffuse muscle aches and elevated CPK)

What is the treatment for tinea versicolor, a skin d/o that presents with oval, scaly hypopigmented macules?

topical Selenium Sulfide- Lotrimin. Oral-Ketoconazole 400mgx1 or 200mg qd x 1wk

In inflamm acne causing papules and pustules, use the same tx as non-flamm (benozyl perxide, retinoids) along with...

topical antibx = clindamycin, erythromycin and oral antibx = tetracycline, erythromycin, doxycycline, minocycline

What is the treatment for allergic conjunctivitis?

topical antihistamine-vasoconstrictor agents Short course of topical or oral steroids (Mast Cell stabilizers: Crolom, Alomide, H1 Antagonist: Livostin, NSAID: Acular)

What is the tx for tinea pedis?

topical medicaitons include butenafine, terbinafine and sertaconazole. Acute vesicular responds to Burrow's wet dressings several times per day with topical antifungal creams

Research supports ___________________ in all infants < 24h of age to elevate for further testing in jaundice.

transcutaneous bilirubin screening

NOTE:

tricuspid atresia is the only cyanotic congenital heart disease with left ventricular hypertrophy cardiomegaly with "snowman" appearance = Total Anomalous Pulmonary Venous Return

If migraines in a child do not abate acetaminophen what else can be offered?

triptans

To diagnose CF, elevated sweat test > 60 mEQ __ times in diagnostic.

two

In infants, is is important to distinguish atopic dermatitis from seborrheic dermatitis. This kind of dermatitis is chronic, positive family hx, happens in extensor surfaces and itches. This kind lasts 6 wks, negative family hx, happens in axilla and diaper area and doesn't itch.

type 1 - atopic type 2- seborrheic

Physiologic Jaundice and its causes

unconjugated hyperbilirubinemia on days 2-4, resolves by weeks 1-2 1. at birth RBC concentration is high (hematocrit of 50-60%) so turnover and bilirubin production is high 2. decreased hepatic UGT levels until 2 weeks of age (especially in Asians) 3. greater enterohepatic recycling due to sterile gut

NOTE:

universal screening for dyslipedemia recommended at ages 9-11 and 17-21

Malrotation with midgut volvulus

usually obstruction involving the duodenum, presenting in neonates < 1 month with vomiting and abd pain with a gasless abdomen on X-ray

NOTE:

vaccinations for medically stable premature infants should be administered by chronological age NOT gestational age (give as normal)

Patient presents with pain at heel and metatarsal heads (pressure points), worse with walking (feels like pebble in shoe. What skin disorder should you be suspicious of?

verruca plantaris

What are the most common offending agents in VIRAL OM? [ most cases of AOM are viral ] What are the most common offending agents in BACTERIAL OM?

viral - RSV, rhinovirus, influenzae and enterovirus bacterial - s. pneumoniae, h. influenzae, moraxella catarrhalis, s. aureus, group A strep

Acute pharyngotonsillitis:1. Rhinorrhea suggests ______ etiology. 2. Pharyngeal exudates suggest ____ or ____ etiology. 3. Vesicle/ulcers =________. 4. Conjunctival conjunction = __________ etiology.

viral, strep or EBV, herpes simplex, adenovirus

Bilateral retinal hemorrhages

virtually pathognomonic for abusive head trauma to infant evaluate with noncontrast CT scan and skeletal survey

What weeks of gestation would steroids be recommended if membranes rupture prematurely?

weeks 24 or 34.

Are male children or female children more affected by GERD? Black or white children?

white males

What is the most common sign of a retinoblastoma?

white pupillary reflex (Leukoria) is most common sign

TX for cat bite

worried about Pasturella, give amoxicillin/clavulanate for 5 days if minor

True or false meconium aspiration syndrome is associated with late term babies?

yes.

What is the holliday sugar method for estimating water needs in the dehydrate child? __ oz per hour for infants. __ oz per hour for toddlers. __ oz per hour for children. To replace ongoing loss __ ml/kg for every loose stool and __ ml/kg for every episode of emesis.

• 1 oz per hour for infants • 2 oz per hour for toddlers • 3 oz per hour for children • To replace ongoing losses, 10mL per kg for every loose stool and 2 ml per kg for every single episode of emesis

How do you diagnose DM I?

• 1. Clinical sx plus random plasma glucose > 200 mg/dl, 2.Fasting plasma glucose greater than 126 mg/dl 3. 2 hr post standard 75-g glucose tolerance test > 200 mg/dl • Impaired fasting glucose refers to a pt with fasting plasma glucose > 110 mg/dl but less than 126 mg/dl. • Impaired glucose

What is the treatment for atypical mycobacterial disease a Pulmonary infection typically occurs in pts with underlying lung disease. Disseminated disease noted in patients with advanced HIV.

• Azithromycin or clarithromycin, rifabutin or rifampin and ethambutol. • Prophylactic treatment w/ azithromycin or clarithromycin when CD4 count < 50

What is the treatment for DM-I?

• Diet: Balance caloric intake with energy expenditure. Must match carbohydrate intake with insulin dosing. Limit cholesterol to < 300 mg/day. Daily protein intake 10 - 20% of total calories. Restrict saturated fats to 10% of total calories. Restrict sodium to less than 2.4g/day. *INSULIN IS REQUIRED*

How does verruca vulgaris progress and what are variants?

• Early: small: pinhead-sized, pale erythematous macules. Larger: irregularly surfaced domed lesions. • Later: larger rough papillary (carpet like) • Variants: digitate, pink to grey. Brownish "verrucous" papules

These flu medications are CONTRAINDICATED in pts < 12?

• Neuraminidase inhibitors (Tamiflu & Relenza) are contraindicated in pts < 12.

What will you see in lab evaluation of A.L. Leukemia?

• Pancytopenia secondary to marrow replacement by tumor cells. • Elevated white cell count in 2/3 of patients. Elevated uric acid and LDH. • Increased blasts noted in bone marrow. 30% to 100% blasts • May have t(9;22) translocation = Philadelphia chromosome =POOR PROGNOSIS

What will you see in the labs of a pt with DIC?

• Prolonged PT, PTT and thrombin time due to coagulation factor consumption. • Thrombocytopenia due to platelet consumption. • Elevated fibrin degradation products (FDPs) or D-dimer due to fibrinolysis. • Fibrinogen level decreases

What is the treatment for verruca plantaris?

• Topical treatments require 2-3 months for eradication. Relieve pressure on wart area with pads. • Soak foot before application of salicylic acid. • Salicylic Acid plasters • Salicylic acid • Other: Canthacur PS, intralesional bleomycin, cryotherapy, surgical dissection.

What is the treatment for hodgkins lymphoma? What is the prognosis?

• Treatment includes radiation for local dz, and chemo for advanced dz. 5 year survival = 90%


Kaugnay na mga set ng pag-aaral

ΒΙΟΛΟΓΙΑ ΓΕΝΙΚΗΣ Β ΛΥΚΕΙΟΥ

View Set

Immunology chapter 5, sections 5-6 through 5-21, and 5-23

View Set

Unit 2 Section 2.5 - 2.9 Vocabulary

View Set

French 2- Bien Dit 1A/1B-Ch. 5, 6 culture review

View Set

NCLEX-PN (Elsevier): Cardiovascular & Respiratory

View Set

Entrepreneurship and Small Business (ESB)

View Set